You are on page 1of 841
eee) ———— Nei) MATHEMATICS GMa S33 Dinesh Khattar ‘The aim of this publication isto supply information taken from scurces believed to be valid and reliable. This is not an attempt to render any type of professional advice or analysis, noris itto be treated as such. While much care has been taken to ensure the veracity and currency of the information presented within, neither the publisher nor its authors bear any responsibility forany ‘damage arising from inadvertent omissions, negligence or inseeuracies (typographical or Factual) that may have found their way into this book. Copyright © 2011 Dorling Kindersley (India) Pvt. Lid Copyright © 2010 Dovting Kindersley (India) Pvt. Lid This book is sold subject to the condition that it shall not, by way of trade or otherwise, be lent, resold, hired out, or otherwise circulated without the publisher's prior written consent in any form of binding cr cover ether than thatin which it is published and without a similar condition including this condition being imposed on the subsequent purchaser and without limiting the rights under copyright reserved above, no part of this publication may be reproduced, stored in or introduced into s retrieval system, or transmitted in any formor by any means (electronic, mechanical, photocopying, recording or otherwise), without the prior written permission of both the copyright owner and the above-mentioned publisher of this book. ISBN 978-81-317-3407-0 First Impression Published by Dorling Kindersley (India) Pt. Ltd, licensees of Pearson Edueation in South Asia Head Office: 7th Floor, Knowledge Boulevard, A-B(A), Sector-62, Noida 201 309, UP, India, Registered Office: 1 Community Cenire. Panchshee! Park, New Delhi 110.017. India Laser typeset by Tantla Composition Services, Chandigarh, Printed in India by Saurabh Printers Pvt, Ltd BERR PEF RBRB EB er pepe ep re BBEBERBS Preface to the Third Edition Preface Eunetions Limits Comtinuity and Differentisbilty Differentiation ‘Applications of Derivatives Indefinite Integration Definite Integral and Area Differential Equations Complex Numbers ‘Sequences and Series Quadratic Equations and Inequations ‘Permutations and Combinations ‘Binomial Theorem Exponential and Logarithmic Series Matrices: Detemninants Logarithms Vector Algebra Coordinates and Straight Lines Pair of Straight Lines es Conic Sections (Parabola, Ellipse and Hyperbola) ‘Three Dimensional Geometry Trigonometric Ratios and Identities ‘Trigonometric Equations Inverse Trigonometric Functions Properties and Solutions of Triangles Heights and Distances Model Test Papers Solved Paper UT-JEE 2010 Contents 9.19.49 11039 A131 9119.45 20,120.21 21.1-21.56 22.1-22.50 Functio REVIEW OF CONCEPTS (One of the most important concepts in mathematics is that ‘ofa ‘function’. The word function is derived from the Latin ‘word mesning operation. Thus, when we double a given positive real number, we think of performing an operation on the number, say x, to obtain a new number 2x. Similarly, when we square a real number x, we get a new number x7, We can, therefore, visualize a function as @ mechanism which produces new numbers out of some given numbers. The concept of function plays an important role in mathematics. FUNCTION OR MAPPING Let Yand ¥betwo non-empty sets. A function f from.Xto 1, written as f:X'— Y's a subset of X * such that notwo cifferent ordered pairs in f have the same firsteniry, that is, if (,y,) € fand (x,y) € fthen y, = y>. Further, for every xe X,there isaye Ysuch that (x,y) ef. IMPORTANT POINT(S) TO NOTE If Ybes non-empty set ¥, where X. Y ).then we say that f isa real valued funetion or in shorta real funetion: function from a non-empty set X this chapter a ‘function’ will mean a “real Features of a mapping fi X - ¥ 1, For each clement x € 2%, there exists a unique element ye! 2. The element y € Y is called the image of x under the mapping f If there isan element in X which has more than one image in ¥, then : Xe Yisnota function. But disinctelements of.’ may be associated tothe same element of Y. 4. If there is an element in X which does not have an image in Y, then f: X— ¥ is not a function. BE cvTion # If. X Yis a function, then there may’ be some elementsin Y, which are not images of elements of X, bout there should not be any x left (element of X) for which there is no element inset ¥ ‘+ IF the graph ofa funetion is plotted and any line parallel to y-axis cuts it at more than one point, then it does not forma funetion. Value of a function The value of a function » = f(x) at-x = a is denoted by f(a). It is obtained by putting x = a in f(x), If for some value of x say x= ay (a) takes the form 0/0, we say that f(a) is indeterminate, + If for some value of x say x ~ a, the denominator vanishes, we say that f(a) is undefined (or does not exist). Domain and range of a function If X— Vibe a function, then the set Xis said to be the domain of fand rangeof set of all image points in Y under the map f “L0~ Leo :fee Fx X} The set ¥ is also called the co-domain of f Clearly, fOCY. 12 Mathematics for IIT—JEE Domain CoDomain Fig. 1.1 In other words, we ean say Domein= All possible values of: for which f(x) exists. ‘Thus, total number of functions from 4 to Ban ns ma times= 1", Intervals in ‘The set ofall numbers lying between two given real numbers, is called an interval in 2. Let a and b be any two real numbers such that 0 ‘© Range is the subset of co-domain. ‘© Range can never exceed co-domain for a given function, ‘ The projection of the graph of y = f(x) on the _waxisis equal to thedomain of f whereas the projection, ‘nthe y-axis is e,ual tothe range of, TRICK(S)FOR PROBLEM SOLVING The total number of functions from set 4 to set B containing m and n elements, respectively, isn”. Explanation Let A= Bpxp oq and B= Waa ‘Then, each element in the domain corresponds to images. f: A——+B ie, x, can take m images. ' : Fig. 1.3 x, can take m images. yy Can take n images. Fig. 15 3. Closed-open interval [a, 6) and open-closed interval (a, 61 Fig. 16 [a,b)= fxixe Reasx and +2, where ~s-= a number less then any real number, +00 a number greaterthan any real number, seo 0,b>0and #1 Exponential functions «= c%is defined forall real values of x, where a> 0. _IMPORTANT POINT(S) TO NOTE. Functions 4.3, {roy ifast |xsy, if0O:ael © log, x> log, > « ie =|.) ifm iseven and Ux" =x, ifis odd. Method to find the range of a function ‘© Find the domain of the function y=f(x). ‘Ifthe domain isan infinite interval, solve the equation y={x) and find x in terns of y to get x = g(0). Find the real values of yfor which x is real. The set of values of» so obisined constitutes the range off Note tha if finite rnamber of values of x are excluded from the domain, find the values of y for these values of x and exclude these values ofy from the range of f found earlier. If the domain is a finite interval, find the least and greatest values of y for values of x in the domain, If ais the least value and 6 the greatest value of yy then the range (f) = [a,b]. Types of functions One-one or injective function A function f: > Yis suid tobe one-one or injective if distinct elements of X have distinct imagesin ¥. £ Fig. 1.7 Inother words, forevery element in X there is not more than one corresponding element in ¥, and for every element in there is not more than one corresponding element in X. Many-one function A function f:X — Y's ssid to be ‘many-one if there exists at least two distinet elements in X whose images re same, f © (=a) (e-6)>0=x b,fora0; b= 1 Method to check whether the function f.X > Y 2 is one-one or many-one pieces faci, itb>t (a) Consider any two points x,y ¢ X. Jao ifacb0;5 1 (©) Irwe get: only then fis one-one, otherwise it is many-one. TRICK(S) FOR PROBLEM SOLVING + Derivative test to check the injectivity If a function is either strictly increasing or strictly decreasing in the whole domain (or equivalently, £@) > 0 or f’(x) <0, ¥ x € 4%), then it is one-one, ‘otherwise it is many-one. © Graphical test fany straight line parallel to.x-axis intersects the graph. of the function at most at one point, then the function is one-one, otherwise itis many-one (ic. it intersects the graph of the function in at least two points). i r L / oo t xx. [Ps a [iwnar Ol je=e0<0 Y issaidto be onto or surjective if every element of ¥ is the image of at least one clement in X under the map f af r : ong - Fig. 1.10 A function f:°— Yisan into functioa if itis not an onto function. x r = tno ° Fig 11 In other words, ifthe function f: > ¥ is such that there is at least one element in ¥ which is not image of any clement in X, then we say that fis a function of 4 into B, ie. fis called into functionif, for some ye Y, there does not existanyxe X such that y=). Method to check whether the function ft X— Y is onto or into ‘© Find the range of the function f ‘= [Prange of f= ¥ (the co-domain), then /'is onto, other- wise KEY POINT(S) TO REMEMBER For an onto function, the range, overlaps or equals co- domain, whereas for an into function, the range does not ‘overlap but fits inside the co-damain TRICK(S) FOR PROBLEM SOLVING ‘+ Number of onto functions (surjections) IX and ¥ are any two finite sets having m andr elements, respectively, where 1 R defined as f(x) =c, vr e R, where c is a constant, is called a constant funetion, lis domain is R and range is singleton set {c} Functions 4.5 The graph of a constant function is a straight line parallel tox-axis as shown inthe figure. Itis above or below the x-axis according as cis positive or negative. Ife= 0, then the straight line coincides with x-axis y @o | fears ero AB Identityfunction The function f: RR defined as f(x) =x, Ve R, iscalled the identity function. Its domainis R and range is also R, The graph of the identity function is a straight line passing through origin and inclined at an angle of 45° with, ovanis, y Foy=x x x 6 r Fig, Li Modulus function or absolute v function f: R + R, defined as x, ifx>0 fixd=|x|=7 0 =x ifx <0 iscalled the absolute value function or modulus function. Its domain is R and its range is (0,23), The graph of the modulus function isas shown in the figure below: Fig. 11s TRICK(S)FOR PROBLEMSOLVING + Foreveryre &,| + Forevery re A)x]= Ye? © [ety Six] +lyl 1.8 Mathematics for IIT-JEE x|+|)"/if x.yhave the same sign, and xy) hx |+|y if syhave opposite signs # |x-y/2/lx|-ly| Lx/—Ly if ssyhave same sign, and] |2[ >| |x| Fy] ] if x.yhave oppesite signs Greatest integer function/step function/floor funetion The function f:R > R defined as f(x) = [x] is, called the greatest integer function, wher, [i = integral part of x or greatest integer not greater tan x or greatest integer less than or equal 10. ie fly = Its domain is & and range is. The graph of the greaiest integer function is as shown below: 1 where nSx 0,then [ bh Signum function The function f: RR defined as, Ix I@)=) x 0 forx=0 iscalled the signum funetion. Its domain is R and range is the set {- 1, 0, 1}. The graph of the signum function is as shown below: y forx #0 1) 0 .-) Fig. 119 Reciprocalfunction The function f: R\{0} — R defined by fix) =~, is called the reciprocal function. Its domain as wellas rangeis R\{0}."The graph of the reciprocal function is as shown below: y Fig. 1.20 Exponential function Let a (+ 1) be a positive real number. Then the function f:R—> R, defined by fix) =a", is called the exponential function, Its domain is R and range is (©, ~). The graph of the exponential function is as shown below: y y Fe)= 08 a> FO)= 08 act Fig. 121 Functions 4.7 aN G mcd suasuwens o d'=etl¥e4. (50) x,(a>0,a#1) +4.be>Oanda.c#1 ga L # logya= provided a# 1, #1 and a, 5>0 jogab Ifa 50°, then.r, >, ifa> Lands, 0,5 Land me R {0} Logarithmic function Let a (+ 1) be @ positive real ‘number. Thea the function f: (0, o9)—> R, defined by f(x) = log, x, iscalled the logarithmic funetion, Its domain is(0, &) and range is R. The graph of the logarithmic function is as, shown below: y y £66) log, = >t. * log,a=1, 1 | provideda>0; a1 —=,ifa>1 ston |cieceet log, x isalso denoted as: nx. *# Iflog, x) > log, ay then x, >x3 if@> Land x; K, defined by Fe=djtajs+a.2 +. capt wherene Nona tian ‘ond, € R,iscalled a polynomial funetion, Ifa, #0, then n iscalled the degree of the polynomial. ‘The domain of a polynomial function is R. a) Rational function A function of the form f(a) where p(x) and q (x) are polynomials over the set of real snumbersand g (x) #0, iscalled arational function. tts domain isRix|q ()=0}. 1.8 Mathematlss for l!7-JEE Trigonometric function Table 11 Funetion. Domain Range R Pht R an] { . Arend nei} R . Ritnz\ne I R 4 (20+ 1)E|me ey Wut) «prams nfarendfeedh Corinto # ys cosees Rijnmine th = uti) Inverse trigonometric function ‘able 1.2 Function Domain Range z -tsxs1 . -1Ses1 # yrcotty -e0 @ yologxts When the variables.x and y occur together in an equation f(s, »)=0. in which ycannot be expressed explicitly in terms fs then y is said to be an implicit function of. Mlustration: () ay tanir+y) @ tay Algebraic and transcendental function A function involving variables x, » raised to any power and signs of BODMAS, is analgebraic function, Mustration ee ay Goa (i) 3)? Fy ar ay 7 A function which is not algebraic is called transcen- dental. In other words, a function involving circular fune- tions, inverse circular functions, logarithmic functions or exponential functions is transcendental Illustration: 5 eae @ yologes {ily pre a2? xl Gill) y= sec*4x + S23 ‘Two ways of defining a function ()Uniformdefinition Ifa functionis defined asy=/ix), x€ (a,b), we sey that itis uniformly defined. Illustration: Gi) y= fl) =sinx, re Ry (il) yA + xe [11] Ifa finetion y= ft0)3@ [a8] (ii) Piecewise definition assumes different forms in different subsets of [a,b], we say that it s piecewise defined, 3 ao Fig. 1.23 Illustration: 1, -Isxr<0 fisetlax, OSxe1 x-l x2 Operations on functions Let fand g be two real functions with domain D, and D,, respectively. Then, 1, The sum function ( f+ g) is defined by CF OS) +g (0, KE Di Dy The domain of + gis D, 0D, 2. The difference function ( fg) isdefined by (Fa) )=f0)—g exe DOD, The domain of f—gis D, > Dy 4. The produst function fis defined by (fe (0)=f0)- B00. re DD, ‘The domain offeisD, 0D, 4. The quotient finetion (4) is defined by g £9 ore DAD MneW=0 (£ rer) \e ‘The domain of Z isD, AD. bx:¢(4)=0} s 5. The scalar multiple function cfis defined by (f)a)=e f(x), ¥xED, The domainof cfis D,. TRICK(S) FOR PROBLEM SOLVING Ifa functions /lx) is such that Explanation We have, f(x) 12) = fSorf (4) > Sd At)-20- 74) () 2 fla) Hr wl) Also, f(x) At) = sors(2) = ro-A(2)-1(2)-10 - AO-7a5 a (On multiplying (1) and (2); we get ; 1: fe 0 2) bQ-]ve- = ({4)- ‘v0 -)-3 3) Since, f(x) is polynomial function, s0, {f(e)—1} and { “2-4 are reciprocal ofeach other ‘Also, xand + are reciprocal of each other ‘Thus (3) ean hold only when SWO-1 esx", wherene R. Composition of functions Let fand g be two real functions with domain D, and D>, respectively. Functions 1.9 Ifrange of f'c domain of g, then composite funtion (gof)is defined by (eof) = 8 UW). D, Also, ifrange of ¢ & domain of then composite fune- tion (og) is defined by (fog) W=flge)),vx€ Dy EE cavvion _g0f exists only if range fc domain g and og exisis only if range g C domain f Properties of composite functions Let /X—9 ¥and g: ¥ + Z be two functions. 1. If both fand g are one-one, then s0 is gay: 2. If both fand g are onto. then gofis also onto. 3. Iffand gare bijective, then gof is also bijective. Explanation Let: A > Band g:B + Che two bijections, Injectivity: Then, gofexists such that gof. A > C. Let x,y betwo arbitrary elementsof A. Then, = = [> gis injective] = xy I fis injective] Therefore, gofis injective, Surjectvity: Letz be an arbitrary element of C. ‘Then,z € C= there exists ye Bsuch thal g(y) =z (- gissurjective} andy ¢ B= there exists.x€ A such that fox) =» [++ fis surjective] Thus, for every 2 € C, there exists x € A such that (gofx)=a(/9)= g0)=2. Therefore, gofis surjective. Thus, gofisa bijective map, 4. gofis one-one, then fis one-one but g may not be one-one. 5. If gofis onto, then g is onto but may not be onto. 6. Itmey happenthat gofmay existand fog may not exist. Moreover, even ifooth gofand fog exist hey may not be equal. But iffog = gof. then eitherf!=gorg™!=f. Inverse functions 1ff:X—> Y be a one-one onto (bijection) function, then the mapping f~': Y + X which associates each element y € Y- with element xe Xsuch that lx)= y, is called the inverse function ofthe function/:X-> ¥. 4.10 Mathemates for IT-JEE ‘We define inverse function f~': ¥ + X by the rule esl ojanvre Xeye Y as shown in the figure below: Method to Find the Inverse of a Function Working Rule Letf: X— Ybe abijective function, * Puyx)=y *# Solve the equation » ~/(x) to obtain x in terms of y. Interchange x and y to obtain the inverse off Aliter: Let g be the inverse of f- Simplify the equation Slgee)) = xtofind gx). IMPORTANT POINT(S) TO NOTE is upointon ph of an invertible funetioy 13.89, Thus, the graphs of fand Properties of inverse functions 1. Inverse of a bijection is also «bijection function. Explanation Let: A> B be « bijection and g: B > A beits inverse. To show: g is a bijection, Injectivity: Let y,,.¥2 € B be such that g(%) =r, and s0)=0 Since g is the inverse of f, therefore, 20,9 =f) = 9, and 0) =) fle) = 9» Now, g()) = 80) 9%) =x) = f(a)) =f) 9} egisinjective. Surjectivity: Letx be an arbitrary element of 4. Then, we A= there exits © B such thats) = y [+ Fis a function from 4 to 2] => there exists ye Bsuch that g(y) =r [+ gis inverse off] ‘Ths, foreach.x€ A, thereeniss y © B Such that 2) =x So, g is surjective, Hence, gis a bijection. 2. Inverse ofa bijection is unique. Explanation Let f: 4 — B bea bijection. If possible, let g: B—> A and hi B >A be twoinversesof f Let e() = x, and hy) =x. Then, a) =x, 3 /ly)=y [ gisinverse off and, IQ) =xy=9flay)=y [ his inverse off] foe) =flxy)3x,=3, |v fis one-one] Thus, gi») =h(y) forall ye B. Hence, a ys 4. Iffand g are two bijections such that (gof) exists. then (of)! =flog”!. Explanation We have, f:4 > Bis abijection g:B > Cisa bijection Again, = (gofy 1: C Aexists f:4 Bis abijection =f: B > Ais abijection Piog': CA exists. Let xe A,ye Bandz © Csuch that Sox) =yand gv)=2. = gof: A> Cis a bijection Al) -Q) 5. If fis increasing , f-" is also increasing and if fis decreasing, f' is also decreasing. 6. Iffiseven, (“lis also even and if fis odé, fis also odd. 7. fog # gof but if, fog ~ gof then either f-! = g or gT=L-Als0, (x ))=(go/ke) =x. (Odd and even functions Odd function A function f(x) is said to be odd if f(-x) == f(s) for every real number xin the domain off Mlustration: Y=f0)=sin xis od Even function A function f (x) is said to be even if, J(2)=f() for every real number inthe domain off Iustration: v=f(x)=x seven. * (x, »)isapointon the graphofan even function ifand only if (x, y) is point on the graph. Fig. 125 ‘* (x,y) is apoint on the graph of an odd function if and only if x, ~») isa point on the graph. y (oy Fig. 126 ‘operties of odd and even functions ‘+ The product of two even functions is an even funetion. ‘* The sum and difference of two even functions is an even function. + The sum and difference of two odd functions is an ode funetion ‘* The product of two odd functions isan even function. ‘= The product of an even and an odd function is an odd function, CAUTION |) It isnot essential that every function is even or odd. It is possible to have some funetions which are neither even norodd, e.g. fla)=37 +x", flx)=log, x flx)=e. ‘* The sum of even and odd function is neither even nor odd function. ero function f(x)=0 is the only function which iseven and odd both, Explanation Uff () is odd as well as even, then we have FO)-fE2)=0 a) and, FO)+fEx=0 (2) On adding (1) and (2), we get f(x) =0, which is theonly function which is simultaneously odd and even. an Functions “TRICK(S) FOR PROBLEM SOLVING ‘© The graph ofan odd function is symmetric aboutor gin and itis placed either in the first and third quad- rant or in the second and fourth quadrant * The graph ofan even function is symmetric about the axis, + To.express a given function f(x) asthe sum ofan even and odd function, we write, fe FOC LEO) where, + [f(s) +f(-1)] isan even function and 3 [/G)-f(—2)] isan odd function. * fF) isan odd function, then f"(x) isan even function, provided f(x) is differentiable on R. © 1/() isaneven function, then /"(x)isanodd function, provided f(x) is differentiable on R. * If fand g are even functions, then fog is also an even function, provided fog is defined. * If fand g are odd functions, then fog is also an odd function, provided fog is defined. * Lis aneven function and g isan odd function, then ‘Jog isan even function. * Iffis an odd function and g is an even function, then {fog'is anevea function. + Fora real domain, even functions are not one-one. + Even functions are many-one functions, because they are symmetric about ya Periodic function ‘A function f(x) is said to be a periodie function of x, pro- vvided there exists.a real number 7>0 such that Sort T)=f(, wre R. ‘The smallest positive real number 7, satisfying the above condition is known asthe period orthe fundamental period of fix). Graphically ifthe graph repeats at fixed interval then the function is said to be periodic and its period is the width of that interval, caution ‘There are certain functions which ae periodic but do not have a fundamental period. For example, the constant function f(x) ~ cis periodic as fo2D=f0) is true for every real number 7, but it does not have a fundamental period since the least positive value of T cannot be obtained 4.42 Mathematics for IIT-JEE Method to check the periodicity of a function = Hfiax +b +aT")=Mlar+b+nT.nel Ly If 7 is the fundamental period of f(x), then Working Rule fxtnT)=fi, Vne Tl Put/(T'+x)=/(8) and solve this equation to find the positive values of T independent ofr. = + Ifno positive value of Tindependent of xis obtained, ence, the fundamental period is 7/ al, which isthe least then f(x) is a non-periadie function. positive value of 7” ‘© Ifpositive values of 7 independent of x are obtained, ‘sin, cos.x, sec x and cosec x are periodic finetions then f(x) is aperiodic function and the lest positive swith period 2x, value of Tis the period of the function f (x) TRICK(S) FOR PROBLEM SOLVING | ‘* Constant function is periodic with no fundamental period. ‘+ tan rand cot x are periodie functions with period z. # | sinx|,|cos.r|,| tan x|,| cot x | seer and cosee x | are periodic functions with period 7, cos"x, sec"x and cosec' x are periodic functions with period 2x when m is odd or when n is, even. * + Hf/6) is petite with period 7; then — and JTay | * ta and cot" x are period functions with prio 1. FO) periodic function wit period Tani g (x)is are also periodic with same period 7. such that range of fc domain of g then ‘IEA G0), ftx) and f(x) are periodic functions with periods Typ T, and T,, respectively. Also, if Ms)~afi(s) + Jf) + ofa), then, period of {f(x)} LCMLof (fj,7.75} if h(x) is an odd function © |ELCM.of tT5 7) i) i an even furetion * IEF (x) is periodic with period 7, then kf (ar + 6), is also BEM or Oriain), Fof 6,12) periodic with period ©, where o, 0,4 Randa, k=O. Qn — at" 3 mpiaauee ‘+ LCM. of rational with rational is possible. ‘oi © LCM. of irrational with irrational is possible. PSU as +) hs peciohe tevin neo PTH We Bae ButL.CM. of rational andirrational is not possible. Hflalx + 1’) +b) =Kf (ax+ 5) 1, Domain of fx)=""Cy,, where Pix) = 19x-9~2x? it nd OA) 5S 4) Donain ofineninedoafix)= V2 ie (@) [4,9] &) [0,9] [6 + 351-64" (©) (1.28) @ 14.58) 6) @ [ll 2. The peri of the function @ (% ses) e es _ fl. when risa rational m 0) (@) none of these @1 (b) 2 4. vajopete te tane!, menqen esr (c) non-periodic (@) none of these BINS Nig hg tig ONO LO. tf ln) is equal to A (@) mfr)—1 ©) ED AM-n ©) @EDAOD +n @ Horn ee os{sin "(sin x + cose} fa) C11] 10) 0) Cee ULL) () (OA) @) [lo) Range of the funetion f(x) = dn (cosx®™* + 1), se(04} (a) (ini te"®),in2) (0) Range ofthe function f(x)= is mfie(e} hea] } (©) (0112) (@)_ none of these Range of the function f defined by f(x) = ] sin(s} (where [.] and {. } respectively denote the greatest integer and the fractional part functions is:) (a) Z, the set of integers (©) N, the set of natural numbers (©) W, the set of whole numbers @) 2.3.4.0} Let f be a real valued function satisfying 2ftxy) =U) + (JO, ¥ xy € Rand (1) =a, where ae | Then, (a—1) ¥ f= @ aa @ a! © a1 @ dt Uf:R 8, ¢: Rs Rbe wo given functions then *fls)=2 min fF 0)— a) |,0} equals () F0)+ 96) i900)-F) (b) FO) + 80) + gO) —F)) (© F@)-g@)+ lea) FO) (@ £0) 20) e)-F) Let (2) bea function defined on [0,1] such that x xe roe aad Then, forall.xe [0,1]. fof ts) is (2) constant () I+x ©) x (a) one of these Ifa function f: R > Rbe such that M6 -PD= ALON +S) *PO) =U wy © Ryan for ® >t 1-2 (6) none o ese 18, 16. 18, 19, Functions The domain of f(x) = feos (sinx) + Jog, (x): where {°} denotes the fractional part, is () 1.2) {b) (0,2n)-[1,x) © (03)-m A function whose graph is symmetrical about he origin is given by (@) f=) (b) ft) cos[logx+ Yl+27)] (©) fe+y)-fO)+f0) Vx VER (d) none of these 143 (@) (0.1) Which of the following functions is (are) injestive rapes)? (a) f)=12+2,¥ € 00,09) (6) fla)=|x+2 re [-2e) (©) fO)= 4-4-5), 4-9) axt 43x-5 td) fy= FO" 3-52 xe (oe) 1 ||] denotes the gremest ix Ix] integer function, then domain of f(x) is @ CLD (b) (-e.1) OCD (@) none of these Let/bea function with domain [- 3, S]and let g(x) ~ | 3x4). Then, the domain of (fog)tx) is 1) 1 w (-34) o [a4] 1) © [ 33] Letf:[-10, 10] > R, where f(2) Let f(x) [€) none of these e inx+|*—| bean ode function. Then, set of values of parameter arisfare (@) (-10,10)-(0} (by (0,10) © [10,9 (@) (100, ‘The range of the function /(x)= sin(sin“ {x}), where {x} is fractional part of x, is fa) (0.0) (b) (0,1) © CLD (a) none of these. I Period of function f(x) = 5 (sin3x + |sin 3x) + {sin 3x]), where [-] denotes the greatest integer function, is 4.14 Mathematics for tIT-JEE 20. 2. 2, 23. 24, 28. 26. 5 2n @ + > an OF we lef: Phe defined by f(x) [x + H], where [Jdenotes the greatest integer function, then f(x) is equal t0 @ x1 © bt 1 1 © @ ix-1] rel If/G) is continuous function Vr Randthe range of fx) a Vaxe R. Then, the least positive integral value of a is @ 2 3 @6 @ 5 0) (2, VB) and geo) = [ ip 0y~ [sin] sre tentang fa) 0,1) b) [0,4} fc) {0,1} (@)_ {0} The domain ott fr} a1+?: where denotes the greatest integer function, is ) C.-V3)U Th.) @ CVU) Iffor a real number x, [x] denotes the greatest integer Jess than or equal to then For any 2 © N [| n+?) Ce] [oat] + j+]72*|+ + C2 ays 16 fn (b) m1 © ntl (@) +2 (@) Co, -2)U [1 ©) (,-3)U TL) {2logx+2) [ 2RE222) andl) = tf then ange of gx) for theexistence of (g(x) is w (63/4 (8) none of these Ifa) = tog.» Range off) =[1 + sin a+[aesing] [3-05] + [xsi] Wx [0, nis, (where [+] denotesthe greatest integer function) n=2_n(n+l)| 2 2. 28, 29. oe 32, 2B. ) na + 4} 0, then the domain of the function FO)=log {px +p+ gu? H(gt rr} is jw Roy werent w ent (d) none of these 2 Ifa and b are natural numbers and se) sin Je 3x4 cos J+ 7x is periodie with finite fundamental period, then period of fix)is: @) x > (9 an{ (a3 »Aio3) (@ xf feP=3 +) in’ The function f(x) = ar 7» Where [x] denotes the 42. 2 integral part of x, i (a) an odd funetion (c) neither odd nor even (d) both odd and even functon (b) an even function 40. a2, 4. 45, 46. F(x)= Tog, ¢logy 2/1 + Functions 1.45 Ifthe graph ofy= ax + bx? + cx +dissymmetic about the line +=, then the value of a + kis we b) c @ -5 © (©) c~bd (8) one of these Let fbe a function satisfying fee +y) =flx) £0) for ally © RAES(1) = 3, then > f(r) is equal to. i Sp @ 3 -1) o) @ #3 @) pone of these The range ofthe function y #(63) o Thedomain of definition of the function o [4 (@) tone of these F(x) = In x} + Jx=2x), where { } denotes the fractional part, is (@) {0} UE=9) © her (0) (lye) (none of these The domain ofthe function F()= In(1 —2 | cos.x p+ Fig © [- (@) none of these The dorrain of definition of the function 1 sin (A 00 fa) (10, 0°) (e) (10, 10°) (b) (10, 10°) (@) one of these ‘The domain of definition ofthe funetion 1 f(x) = —=———. where {. } denotes the area fractional partis, 446 Mathematics for IT-JEE 47. 49. 50. # Pabe «| 3-5 © ( 3 [-1. 21 is om, fa) Peet ET (by Sax ©) Fem x— IP! (A) none of these LetfsR > where x)= 2228 The values a+6x-8F of @, for which fto be onto, are @ 214) (b) 2,14) © 214] @ 2.14) Domain of definition of the function foo- nein? ss a (@) Gr, Qn Ine T (b) (2x, (2n* Irn 1 (OR (d) none of these 35. 56. 87. 58. 59. ou. 2. The domain of the function Fe-binstoos( 5) is (a) (1.2) (6) R—(1,2) (© R02, (4) none of these x-Lx] The range ofthe funstion, @ (0.3) ole ‘The period of (b) [o | w (0 4 a) =sinx + tan 2 + sin + tam +. fe) - z 3 x x +tan > is +sin (le (©) nox-periodie (b) 2x (d) none of these Ifthe domain for y=f(x) is [-3, 2] then the domain of 960) =f1) Es] is (a) (-2,3) (b) [2,3] (©) (2,3) i) 23) 1 (x} and fx] represent factional and integral part of = 2000 6 4 7} ete value of [x] +S EAT @ x (b) 2000 (©) 0 (4d) none of these Let f(x) bea function such that feta =1+ f2fx)= (2G) ,Wxe Rthenfis a periodic function with period fa (b) 2A, (ce) 44 {d) 6A Lett be fos dete y 7) = 5 an gta) i Then, ( fog) (x) isequal to (a) x (b) 2x (ce) 3x {d) 4x iy(ay= J3)x[— a =2 and gtx) = sin, then domain ofefiiton ffou(d @ {ortn3}ner In ur nes ome BE) ner o | ae 2m | 6s. 66. 67. 68, 69. a Grane © {ame+ 22}, ner ca femed] anes ant] fis) tiem UFSMEN™=) onthe rmgeotf(x)is mn( sing)" +1 @ FLY tb) 10.1) © HL @ (1,01) If f:(0,x)->R bedefined by f(x)= ¥ ({1 + sin kx]), i ‘where [x] denotes the integral part of, then the range of fOis @ tant} (b) n-ne} (©) inne ly (d) none of these Letfix) = (x + 1)?=1,#2—1. Then, the set $= fife) =P iis @ f 34h 3-48 (©) 10,10) © 0-1 @ 6 ‘A function F(x) satisfies the functional equation 2F(x) + FUL) = 2x4 for all real x, P(e) must be (@ xe b) 1-4 @ +2 @) 24x41 Consider @ function /(m) defined for all n € N. The fanotion satisfies the following two conditions @ f0)*/2)+/G)* «1060 Gi) FO) pip) Fla 1) + fln+2) +... to} where 0

0 ——(@) neither even nor odd Let f: 8 ~ {2} + R be a function satisfying 2f(x) + (222) 2100+ 80 re R= {2}, then fe) = a 60(2x +29) (a) 16~ 40,- = x2 32x42) (b) 100x+80- = x2 40 toes 30CE+29 © a (2) none of these Letg: > R be given by g(x)~3 + 4x. If") ~g0g0 «- .0g(a), then g(x) (where g(x) denotes inverse of 8") is equal to (@ @-1)+ate (b) +14" 1 (6) Get 24 (d) (4% 1jr+4* Leif p) 142-3 f0)3F@)P-(FOP TAY xe R, where p>0. Then, fxs periodic with period. @p ) (©) 4p (d) none of these it Y s(x + ka) =0, where a> 0, then the period of & semis @a ) Oda Oct (@) none of these Ify= logge and $= (3,27), theset onto which theset $ is mapped is (@) 0.3) © aa © 0.9 @ 0.2 ‘The values of x for which the functions f(x) =x~3 and 9 (0) =4—xsatisy the inequality (x) +9(x) }<|/0)| +40(0)| are @ Ba (©) (-2,00)-[3.4] b) mye) (d) none of these % 5 8 Iffisan even function defined onthe interval [- 5,3], 9% then the real values of x satisfying the equation sins 5 aw x+2 -1tS W345 @ = = 10, o> 2 @ none of these 2 rte etn = fo Tote foeu(l [J denotes the greatest integer function, (@) Domain()=[2.%) (6) Domain( f)= (2.09) u. (©) Range(f)= {| @ ringei/o= {7} ‘The distinct linear funtion which maps [- 1, 1] onto {0,218 @ x- xt @ -x+1 (@) -2-1 Let fx) be defined forall x > @ and be continuous. , rn Let fix) satisfy /(2) = Six) —f() forall x,y and ‘fle)=1.Then (@) fixisbounded — (b) At) as x0 (© xf()-0a8x30 (@) fO)=I0R¥ “The function f:(— ,—1] + (0,51 defined by (0) = ook ig B (@) one-one (6) many-one ss (©) into (@) onto Let f RR be defined by f(a) 142+ 1008 + 5 sin x.then fis @) one-one (b) onto, (© many-one (@) inc, 1 LeifiR-+Rbea function defnedby a), then the graph of f(x) lies in which quadrant” @t ) 0 ml ww 1efem)~fe= Ali Z Jeesene Finan —H fois lex @) tog{ 1=* 18. @) va( 22) &) wn (@) tan? © ma(}2) Which of the following functions have period 2? CO) sn( Zt) (2) sin (cos x) (a) tx} +eosmx (©) sinxt fx) ‘The values of x for which the domain of defivition of 1 (s-11+]7—#1-6 denoies the greatest integer par, is not defined are the function, f(x) = + where [.] @ 1) ® 08) fe) {2,3,4,5,6} (@) 1, UTS] HyO= Te where [] denotes preatest integer function, then (a) f(x) is an odd function ifx= nt (b)_/() ivaneven function ifx# em (e) f(x) is anodd function if.x #0 (€) flx)isan even function if x= nm FRR bedefinedby f(s) (a) fis one-one {b) is onto 1 Fep= boger= fe +1 @) F1C)= g(x ys Letf(x)=sin (oglu) + togtsin”'[x]), where [ Jdenotes the greatest integer function. Then, (a) domain offis[1,2) (b) domain offs 1.3) +0 (e) range of fis {ioe} (@) rangeof fis {0} If the function f [1, 2) + |], =) is defined by (0) = 240) then (@) fis one-one () fis onto 1+ fis atop; 2 fiz dows = 2) = @s'ai= Levin) Then, oad (@) foo+f(l-a=1 0) f@)+f0-n=—1 16. Functions 1.49 Let2 be. positive integer with (1) = 1! +214 31+ ..+ nn and P(x) and Q(x) be polynomials in x uch that Sn 2)=P aft 1)* Ol) fa) forall 1, then @ Peo=xt3 (b) O)=-x-2 . (@ Pay=—x-2 @ Oaxe3 4 ws 10, n. The period of the function, 3" #74 [I+ sn) | where [| denotes the greatest integer function, is .. ‘The number of integral values ofa, for which 3 may. bo the period of the function cos nsx sin is ‘The value of w€ I for which the function (0) = 5 has 4r.as its period is... si) F(2x+3)+ f(2x+7)=2, V.ve R, then the period of f(s) is. ff (3) is an odd periodic function with period 2, then FA) equals... Let f(a, ») be a pericdie function satisfying F(s,9)=f(2s+2y, 272s) forall my. Hale) =/2 0), then g(x) isa period function with period.. Iffix) = (2 cose — 1) (2e08 2v— 1) (2 cos?*x- 1) .. (2cos2”"'x-1), where 2 1, then for'¥ ke uz sae }: Fe The valve ot 2]e[ba co] [ba fy 2) *[2 * 100 |*|2 * i060. 5 ty a> 2*io0, ‘The period of the function f(x} satisfying the condition FO=W+ P+ =HO+ +049). 3 3 1 neviveet[S]e[2¢ch]+ E mail E | [i * lis Sah Beae lo Ify = 2[x] + 3 and y~3 [x2] +5, then the value of [e+ylis... 12, 13. 14, 15, 16. 17 18, 19, ‘The number of surjections from 4 to B where A= (1,2, 3.4} and B= fa, Bh is. fF) bea polynomial function satisfying Je) 1} roy (F}mrernes, then f(6)= Let f(x) be a funetion such that f(x — 1) +f(r+ 1)= % V3fl0), Ve R.IERS)= 100,then YF (S412) = rt Let/:N >. be a function such that x-pe0- [5] -vf LE] v0 where [.] denotes the greatest integer function and 1900 ,..). Butin the case of functions where modulus is involved, the LCM. rule gives the period of the funetion but it may not be the fundamental period. For example, according to the L.C.M. rule, Period of] sinx |+| osx |=L.CM.0f(%,2)= 7, but it is not the fundamental period since bin{ s+ 2) +bos{ +2) 2) at which shows thatthe fundamental period is osx || sine ‘Thus, the period of| sin px |+| cos gx | ~ -temor( ma J itoee Pa 5 The function f(x} = k| cos.x | + A> | sinx + o(k) has period = if is equalto @ ©3 6 Theperiod of the function (b) 2 (@) none of these Fs)= 30+ 3-[3r+ 3}+ sin BE, where [x] denotes the greatest integer 0and a1 Sin'xand cos |r are defined for-1) 013) (© R-(,3] (@) none of these 14. _ If [x] denotes the integral part of x, thenthe domain of the unetion/ (x)= si [2 ~ 3} *log,log, (a? — 30 +S)lis & (b) [+8] 0 (6 0 (eb (d) none of these 1S. The domain of the function «y= Fema =D 5, JO" VP ac @ 2.5) © 26 © 2, (@) none of hese 16, Ifthe domain of f(x) is [-1, 2], then domain of f({3] =" +4), where [-] denotes the greatestinteger function, is @ E17) © C17) © (V3,-Nutvi v7) @ (48, -0 U3, V3] Considera one-one onto function y=f (x) havingdomain A and cange B, ie. f:A—> B. Then, there exists a function g suchthat~ g()) which therefore has domain B and range 4. ie. g:B— Asuchthat/(x}=ye av) =x, Vxe A and ye B. gis said to be the inverse off: 17, If f(x) satisfies x+ | fix) |=2f\x), then f"(x) satisfies (a) 3x+)f'@)|=27"@) (b) xf = 3 ea) (©) f'@)-|x |= 2x (@) 2-11) =F) 124 Mathematics for IIT-JEE 18. ff()=x| x) then f= @ isi (©) (seni) © -vixi (@) none of these 19, Theinverse of the function f(s) = atta lige (LEE 1 lex a) + tog,| ) Ahog, | #* os esd 5 a(S] © te, (@) none of these 20, Ifthe function f:{1,%) —>[1,) isdefined by f(x) = 2D then fax) is a ey” © al eas] (@) not defined PASSAGE VI ‘A mapping fi A —> B is said to be one-one or injective if every element in 4 has a distinct image in B orhas no image atall and every elementin Bhas a distinct pre-image ind or thas no pre-image at all. ie. fe) =F a) x, = Hy for every x,,x,€ A Graphically, a functions is one-one iff none ofthe lines, «drawn parallelto the x-axis or parallel to the y-axis meets the ‘graph of the function at more than one point. ‘A mapping A — B is said to be many-one if every element in 4 has a distinct image in B or has no image atall, but there is atleast one element in B which has more than © Lie FoR] one corresponding pre-image in 4. Graphically, a fanction is, many-one iff none of the lines drawn parallel to the y-axis meets the graph of the function at more than one point but theres atleast one line parallel to x-axis which intersects the curve at more than one point. By calculus, a function /(a) is one-one iff") = 0, .£€ domein, otherwise not A mapping f A + B is said to be onto if for every clement in B there sat least one corresponding elementin A. Here, range of f= co-domain, ic., /(A)~B. A mapping f: A > Bis said tobe into ifthere exists at Jeast one element in B having no presimage in A. 2, If: R> Ris defined by f(x) =3x~ 3] x|, then fis (@) one-one onto ——_(b)_ one-one into (©) many-one onto (al). many-one into 2 Lea pkva={ sr<3} bea function such that Fb) = tan \o? + x +b), where k is constant. The ‘minimum value of & for which fis an onto function, is @ (b) 0 (© Va (@) none of these 23. IL fayex + 3x’ hdc asine + beosx, Vive Risa ‘one-one function, then the greatest value of a” + 6? is fa) 1 (b) 2 © V2 (@) none of these 24. Letf(x)=3?—4x—5, then (a) f isone-oneon (b) fisone-one on (-=,2] (©) f isone-oneon [2, =) (@) none of these Instructions: fn che following questions un Assertion (A) is sven followed by a Reoson (R). Mark your responses from the following options: (a). Assertion(A is True and Reason(R) True; Reason(R) isa correct explanation for Assertion(A) (b)_Assertion(A} is True, Reason(R) is True; Reason(R) is not a correct explanation for Assertion(A} (6) Assertion(A} is True, Reason(R) is False (a) Asscrtion(A) is False, Reason(R) is Truc A. Assertion 13 /(2)—/(0ix)~ nat x> 0, then the area of the region bounded by fie"), axis, the lines ¢=— I and.x= Tis0, Reason: fx)=Mnx,x> 0. 2. Assertion + Iff(x): (o> 0) ,then +Va Reason: faye f(l-ay=1 Vx 3. Assertion : The range of the funtion f(x) = g(x) + go) where go) = (=x? 4x —3 and ‘al hei B emus | hoy= io( {sin -)) is (0,21 Reason: Maximumand minimum values of both g and rare attained at and I, respectively. Functions 4.25 Assertion + Iffx] denotes the integral partofx,then | 6, Assertion : Suppose, f(x) = (x + 1)? forx 2-1. If domain of the function f (x)= gix) +(x), g(x) is the function whose graph is the q reflection of the graph of f(x) with where 20) = Gp GonG=D respect to the line y = x, then g(x) = ea vk -1,x20. ane a)= in [ = Jist.2)-0) Reson + giiisthe inven off) Reason: Domain ofi(x)'s[0,2) 7. Assertion : A polynomial function (x) satisfies the 1 1 i {= = Assertion + If f()= 24,1, then ee (2) oe Ad} 1 1f/(10)= 1001, then f(20)= 8001 (ofafofof)(x)=f (x) Reason: f(x)=x"+1, wherene J Reason: joftx)=/@) _ QUESTIONS WITH ONE CORRECT ANSWER. (a) 5. ) 6 (bi 7. ib) 8. (b) 0) 30 WO 5b 16 m tb) 21.0 20 2B. i 24 @) (b) 29. (b) = 30. (31. fb) 32.) +(e) 37. 0) 38. 3%.) 4.) (0) 45. (b) 46. 7. fb) 48.) (0) 53. 54. 5S.) 56.) (a) 61. @) 62 3.) GH.) . ta) 6. 70. @ 7.) 72. HY (0) QUESTIONS WITH MORE THAN ONE CORRECT ANSWER (bc) A cd) 5. (ad 6. (a,b) 7. (c,d) 8. (a,b,c) ed 1 @ Dd 1%. oO 14 @DO 1S a8) 16D) ey PILL INTHE BLANKS (2) 4 5. 0) 6 (12) 7 8. (50) : 3) 12) 18, BID 44. (19000) 15.1904 or 1994) (501) 19.06) zs COLUMN MATCHING 1. 1 (b), it > (a), Il @), Va) 2. 1 Cd), I (€), I @), VW > (b) 3. 1 (©), > (b), Ill > (a), V > @) 4. 1 (d), Il (C), Il > @), VW > () S. 1 tb), ll = (c), Il > (a), IV > (a) 6. 1 (©), Il > (a), Ill > (b), IV > (d) % 1-9 (0) th > (b), Ill + (@), IV > @) 1 @ 2 2 ©) A 6) 2) 1 @ 1. 6) 12. @) 17. @ 18. fB) 19. ©) 20. fb) 5 (a) 6 fa) 2a a B@ Wd 15.@ 16 © 2 2 ee 2 (). Forevery rational number 7, wehave, 1, when x isa rational AT = 10. when x is aah but there is no least positive value of 7 for which f(T +x) = f(x) because there are infinite number of rational numbers bebvech any two rational nunibers. Therefore,/() ie a periodic furction having no fundamental perio. 4(c). Inthe sum, f(1) +f(2)+f,G) +... +4(m), | occurs times, occurs (n—I)times, Loccurs(n—2)timesandso on. 2 AAR D+AE) +l) I Hn Lb Ds 5 40-2) fx} € (0,1) in {x} € [0,sin 1) buts (x)is defined if'sin {x} #0 ie sin {x} ( sin1” 9 (d). We have, Jes) = 2min {| fle) —g 6,0} s £ £()> g®) 2/3) - 8D) S) (4) meso: eee a) ~ 8) 1-8), + f0)=/@)-8)-|8 0-0) | S12)? BO) f(x) g(a) ww-lE, ae ffs) = te sS@)EQ I- fl) FEO xeQ, xeQ _ hg x€Q, reQ lem L-xeQ, 0 1-(l-x), 1-x€Q, 1€Q {e398 x reO £: REO), x6 [0,1] 11 (c). Wehave, Lo-FO) FO) + fv) +fl0)- ~) Put + =fy)=0 then £0) =f0)+0+f(0)-1 2) « (A)+ 2 +f(A)-1 => 1=2(A)+P-1 p= a2 Hence, flo) = 13 (c). A function whose graph is symmetrical about the origin must be odd. (G°+3*) is aneven function Since cos x is an even function and log (x + V+? )is an odd function, -. cos (log (x + Vix? )) is aa even function. Ufte+ =f) =O) 4 YE R, then f(x) must be odd, 414 (b). The function f(x) = 2 +2, xe (=, ) is not injectiveas /(1)=/(-1) but 14-1 ‘The function fix) =(x~4) (x-5),x.¢ (= =) is notone- oneas (4) =/(5) buts #5 axte3e-s “The fintion fs) = 3E=5, faction 7 ay Sat injectivensfi1)=/(-I)but #1 Forthe function f(2)= [3 #2 | xe [= let fe) 2 (-=, 29) isalso not So, fis an injection, 16 (0). (og) ) “lg C= SU |3x+4 |) Since the domiin of fis [3,5], 3S (3x44 [<5 3x44 [<5 S-SS3x+4s5 1 -9S3xS 1 9-35a58 5 vomits 24] 24 (a). For any xe R, wehave ay Phx, where ne 0¢k<1 Functions 1.27 =, gt 0 Clearly, g(x) > 0 forall x. Sof Ig (x)] = 1, forall.x. 31 (b). Wehave,g(a)= 1+ VE and {lg (a))=3+2Vk +x ol) Also, fle @il=/0+Vz) 2) From(1) and @), we get IU+AE)=3 +298 ta, Le It yF=yorx=(y-1? Sf) 3420-4 1F 42y-24P-Iytl=24y2 J) =242 32 (a). For/to be defined, we must have 1 1 log) [t+g= |<-1 = 1+ bara ) possibleonly if $= > 1,ie.,0 (2 ly =2 which is Hence, the domain of the given function is {n00 Forf(v) tobe defined De r(pt gy Hg tAetr>0 = prcet ltaneet treet D0 = &HDer+gren>0 = xen Landre [Since ¢?—4pr=0 eo atxs ch pt tgetrs eo p+ at pet gxtr=Oandatre -2, po meine rfen-nf-2}] 38 (b). Since the function is periodic, a”~ 3 and 6 +7 should be perfect squares, which is possible only if@ inwhich case flr) qxtr>0] in x+ cos 4, whose period is 2x 39(b). Wehave, f(x) = = far ne wl"? Case L.whenx=nt (ne 2) sin!®! (Cnr) Sex "f(x isan even function. 40 (a). Graph is symmetric sboutx= tif (kx) =/(k+2) = alka + (k—aP + olka) kta + bk tO teks x) Hd => ax! (Gai? + 40k +2e}e=0 Itistrue for all « if a= 0 and 6ak? +4bk+20=0 Baty. 43 (c). For the fn operation to be defined, we have {x} >0 Plotting the curves y= (x} andy = 0, we cansee that the values of x satisfying the above inequality, are xeR-T 0) Forthe square root operation to be defined, we have ix) S22 ie, Plotting the curvesy= {x} and y=x/2as shownabove, we can see that the values of x satisfying the above inequality, are xe O)ULL9) 2) ‘The domain of definition is the intersection of (1) and (2) which gives € (1, %9)— 1° ‘44 ,(c). For the /x operation to be defined, we have 1-2 cosx|>0 ie, joosxi< + ~() For the inverse cos operation to be defined, we have ax -Is— ee] +2)G@]-2)20 => -2S [x] <0 or fx]22 =2sx<1 or x22 2) The union of the intervals (1) and (2) gives the domain as, £20u 2,3) 48(c). wehave FO)= (10802) + (logo 2) (loge 2 0.0016x) + 36 = lees 22° Grogs 29 flotos rlogs102)} +36 VIE +5NO44)436 [puting loge = fen? vime36 = fsa a 412) Functions 1.29 For the square root operation to be defined, we have (A+3)02+12)20 , «AF3E0 (2 2 + 12 isa positive quantity) y — W0Gygx2-3 fe, x8(02)?=5*= 125 ‘Also, x0 for logto be defined Hence, the required domain of definition is (0, 125] 49 (d). Wehave, xe (0,2) Osr<1 Isx<2 ie Isx[-1,2]is onto, then eA t2x +1 BP 241 Solving the left-hand inequality, we have (a+2y2 +220 whichis tie Vx Rifa2—2 0) Solving the right-hand inequality, we have (a2) + 6r-150 whichis true for all € R if coefficient of” <0 and D <0 S2,WreR ie, a<2and36+4(a-2)<0 ie a<2andas-7 ie as-7 2) Hence, from (1) and (2) the permissible values of a are ttiven by ,-MU[- 2,20) ‘52 (a). Wehave, fix)=xt i) ee N Thus, we have SO)=IF1=2,— fQ)=2-1=1 SQ)A-3+1-4, (44-13 ‘and soon, s(S)=5+1 LO The groph of fc) consists of the points (I, 2), (2, 1), .4),(4,3},(5,6),(6,5).- Thus, ifia, 8)isa pointon the graph, then (2, ais also a point on the graph, Hence the inverse of Sis fiselt je, P@axt DL xeN ax? +6; + 6x- 8x7 ie, (ort 8px? + OC —yhe— (oy +8)=0 According to the given condition, y takes all real values, for real x. nother words, the above quadratic equation in x should have real roots for every realy ic, D20,VyeR ie, 36(1-y)*+4(@r+8) (a+) 20,VyER ie, @rBayy2+ (Pt AO (9+ BA)ZO.V ye R ie, Ds and coefficient of? >0 ie, (0 +467 $4(0 + Bay? and 9+ 8a>0 ie, +455 219+ Ba) and @>—9/8 ie, a 160+2850 and a>-918 2Sasi4 and @>-98 2sasl4 Hence.f: R— Ris onto for2$ ars 14 53(d). Lay 54 (b]. The fumetion/ » “sin is defined for 2sl) 6g, 212L ex) +H is a positive quantity [tet in, 2 )x|+120 ((x|~120, whieh is true ¥ ve & a0) “The function Sin is defined for, sinx 20 is, QmsxsQnt+hm ner -2) Intersestion of inequalities (1) and (2) gives the domain as (Onn, Qn+ In}, ne 1 55 (b). (sin.x]is always defined ) ints daued evceyhenc nen when = Isx<2 Hence, domain R-[1,2) x-[x] T+ x-[x =e) 4) 96 (c). Here, y= i+ ‘Thus, domain = teh So, from y= we have, Tris ty be) = bet baad yh x Osta) <1 50, 0S <1 = Osy< Hens ange =[ 58 (c). Forg(x)=/\[x||tobe defined, we must have 35) fe] |s2 => 0S|[x]/<2 2sb1s2 = -.26x<3 [by definition of greatest intoger function} Hence, domain of g(x) is (- 2, 3). [os] x|20¥x] = las|x|Sa>—asxsa} 59 (a). We know that (x + ix} asre Integer 2900 2oot cya Bt fet & + GI 2 3000 w+ > 5000 st £3 . upto 2000¢ines| 2000 2000 bl 2000 Blt ts} 60 (b). Forthe function to be defined, 2fOI-FO)20 = F@)LF@-2150 = Osf(x)s2 0) and from the given function, FetAZ1 = fHZt From (1) and (2), we have 1s fgs2 Again, wehave { f(x + A)—1}?= 2 flr) 4x) FGF AHIP=1+ Rf) = (feet )=1P=1- (f0)- 1)? of) Replacing x by.x-+A, we get AGH2—HP= 1 [fees Subtracting (3) from (4), we get 1S (e+20)— 1? =f) - 1)? > fee2=/e) = _fisperiodie with period22, 64 lc). f(x) = ¥asfsinexd = r+ [sin x] + [sin 2e]+ + [sina] ..) Case: When dox# 5 fork= 1,2,3, 0 Since 0 fe.z)=".P or f(y=% Q) From (1)and (2), we have xP =P = Lap or gape » 7O(a). Given, Sox +9) 4/9) =2/0/0) a) Interchange xandy in(1), we get SOF 0-2=2F OF) ~@) From (1)and 2), flx-y Putting y= 2x, we get f(x) 132 Mathematics for IT-JEE 71 (a). We have, 3 ,(2x+29 ) + 50x40, ay 2x42 Replacing xby ——>— in the given functional equation we gel (ee oP) Sr sof 2222). 40.) x-2 | Using (2) in(1), we get se=? fos 7222) 60+50x+40 = Pfeo-fi= 29-00-75 222 | = ffo- 0-50-72) = 9) 16-abe~ea{ 212) 72 (b). Here, a) = = (og) @)= alg (x)} = a3 + 4x) = “+4(3+4y) = +r > Bix) = 4-1) +e On generalizing, we have 446 ‘Then, for finding inverse, g"(x) = =) ye = +14" = +1-9ye" = 7) = (e+ 1 -47)4" 73 (b). We have, See ph= 1+ (2-3 f@)43 (FOP - FR)? = Seetp) =F (14 (=F? = fletp)-1=[1-4F0)~1)9)"" = geet p) =(1- tat} tt) where g(a) =/(3)1 and gl + p)= fix p)~ | = sg 2p) [1 = gee py (2) > a +2—) =(1 11 YHA from(1) and (2), we have > ats 2p) =[1- (1 ~ fata} 3! > geet 2p) [1-1 + (20? = gtr 2p) = [1971 = G+ 2p) = a6) which shows: (r+ 2p)—1=f62)—1 a, fO+2p) =f6) Hence, f(x) is periodic with period 2p, TA (b). Given f(x) +/+ a)+...+ fle+an)=0 Al) Replace x by x + a, we get PO a) =fEF 2a) wtf xt aN> =O 2) Subtracting (2) from(1), we get SO)=f era(n+Ih=0 = £(Q)is periodic with period a(n 1) 75 (c). Since y = log,» is an increasing function, so S is mapped onto te set (loz,3,log,27) =(1,3) 76 (0). We have f(x)+ ¢x)=x-3 +4—x= |, sothat isd | +] p11 Furthermore, a fe-3 fee, wre a teed a-x ifes4 wet 0 aos 7-2x if x<3 = WO)|tidol=4 1 if3ses4 2x-7 if x>4 We need those points for which the L.H.S. is greater than 1. Clearly, we can exclude values ofr between 3 and 4. Now, for values of x less than 3, 7 ~ 2x is greater than 1, and for values ofr greater than 4, 2x — 7 is greater than 1. Therefore, the given inequality is true for values of x siven by (2,50)-[3,4)}. xt x2 Oy ‘Since f(x) is an even function defined on [- 5, 5}, os SED=SOWVXE [-5,5] xl x2 Si) ozexti-o spe ae 22) From (1) and(2),the values of x are -14¥5 and ~34V5 2 2 3 (b,€). Let the required function bef (x)= ax* b Wa> 0, thenf(-1)=Oandf(1)=2 + -atb=Oanda+b=2 = a=land Ia<0,then f-1 = -a+b=2anda+b=0 Vand b= +1 oor f@y=-x+) if (a) = log x, we see that f (3) =fe-r0) Clearly, f(x} is not bounded | w ) Also, xf (x)=x logx0asx0 T (c,d). Since f(~x) =f) => (2) isan even function, its graph will be symmetrical about y-axis. Jog x ~92028x9 0 Also, /( ) =f) =(potv) = negative i.e, the graph of (2) completely les below the a-axis, and is also symmetric about y-axis (as discussed above). ‘The graph of (x) lies in Ill and TV quadsants, 8 (a,b,c). When x, =-1andx,=1, 1 T+) then f(-1)-/(1)= A = s=0, Jue y which s satisfied when fo) = tan"! ( i) When x, =.) = 0, then FO-fO)= 4 }-ro-ri0-0 1-0 When x, =—1 and.x, = 0, then SD-LO)= Funclions 1.33 lex Tex which is satisfied when fix) = Ct and, f= ve] lox (a,b) (a) The period of cos mei 2 =2, and period of tx} isl Hence, period of the given function is LCM. of (1,2)=2 oy sig (Zi) =e 3 » &+7)-b gives a value of T independent ofx only if Tis an integer. In that ease, the above equation reduces to b+ 7-bx)=20 ie, T=2n Hence, period of f(x) is the smallest positive value of 7, ie,2, (©) We have period of sin.x = 2nand period of {x} = 1 Hence, period of the given functions is L.CM. of (2m, 1) which does notexist since 2mis an irrational number. Hence, the function is not peri (@) Letussove sin {cos (x + 7)} =sin cos x} ie..cos (e+ T= nm+(- 1Y"'cosx,me Putting = 0, gives cos (x + 7) which gives T= 27as the smallest positive value. Forno other value of 7 can a value of T be found independent of x. Hence, the required fandamental period is 2x. 40 (2, by). The function is defined forall real values of'x ‘except those which satisfy the equation, [ls-1|] +[\7-x]-6=0 mo) Case (1 £8) TF whieh is stritty increasing as 2 e**>0 forall x. Thus, fis one-one (©) Ontos Lety=/) => y= £2 where yis stitly monotonic Hence, range off) =(S- =).f)) = nngeot f()=(- =.) So, range of /(x)=co-domain Hence, /(s) is one-one and onto (©) To find) sy~ = since, ef is always positive, so, neglecting negative sign. Hence, fx) =logte+ fx? +1) 13 (a, ¢]. We have, F(3)= sin" og [x]) + loatsinr fx]) say Lt tx) =sin "(log [x)) 2) and, Hx) =logtsin~'fx)) 3) Now for gix); =1Slog[x]<1_fassin”! @exists when-1< 8< 1} and, [x]>0 {as log] exists when [x] > 0} = — L0 = Blsl2 a xe (1,3) (4) Again, from (3), we have ‘(x)= log (sin-"[x) exists whens sin (x]>Oand~1s[)<1 = be} Oand-Tefe]e1 = O Domiinof f@)is[1,2) Now, forrange, weknow, f(@)= sin (loglx))+log(sin "[x)) wherex € [1,2)=9[e]=1 Range of f(x) = sin-!(log 1) + log(sin-! 1) aint ® sir'(o+be (2) = log(n!2) = Range ot7c~ ioe 3} 14(a,b, c). (a) one-one: (2x- 1): log, Functions 1.35 For f(x) to be one-one, it shouldbe strictly increasing or | Adding (1) and (2), we et strictly decreasing. # " So, f'@)>0 FOF => BF xn 1)>0, where 2x?-#> 0 forall: 6 943 349%) | 1 | > H1>0 or wt | | fe) fU-x}= (3) Thus, for given domain 1, =), £62) always increasing. 12 3 98, Hence, fis one-one Now, putting «= 7555 Toa¢? Ta08" “79967 (b) onto: As (x) is strictly increasing (3), weget => Rangef Qe [US my oe cr Fergie ts gba)e 2) = Rangect/(a)=Covdomsinotftayimssisents. |, pf 2), (1994) ier annem fo ‘As fis one-one and onto, -' can be obtained. — AZ) (3-1 Let y=fix) 1996. 1996 = = O-x-log,y ss 997), pf 9) 1 . LENE Hog: y = a (=). , 2 998 998 => S| Jt S| |] #1 or F| cq 1+ 144i 1996. 1996. 1996, 2 = fig- Aer ‘Adding all the above expressions, ve get [asy>0,Vre D] 1 2 1995 (a) * (ae) * +1(i5e] 15(a,0). f= 0) Wish [Oro = (alee 90745 ob and, vi = 99145 = 9973 = 16(a,b). Webaves(n+2)-f(n+ 1) 9 > S0-aey 2 ae =(n+2)!=(nt2) (041)! a =O DLS DFO] pig 0) POAC DfO4-CH42F0) 3649) P(x)=x43.and Q(x)=—x-2 > }. (4) We have, p & ‘{2)=f00+2)=f0) SOx+3)+fOxt+T=2 Al) | and, 2 Replacexbyx+1, f(2x+5)+f(2x+9)=2 ..2) | Now, £2)=-£0) ‘Now, replace by x+ 2, = 2f0)=0 ie, f(0)=0 SOs+N) +f Oxt 11-2 ~@) | on £8 ~f2+2)-fQ-F0)-0 From(1)=(3),we get/(2x+3)—f (e+ 11) =0 Ths, s@) =0. ie, fQx+3)=/2et IaT=4, |. (12) sla,y)=f2r4+29.2)-20) 0) 5. (0) Since f(x) is an odd periodic function with period 2 = f(Q(2x + 2y) +2(2y— 2x), 22y— 2x) —22x+ 2)) C3) = =f) and f+ 2)=f(x) fom (1), 1.36 Mathematics for IIT-JEE = fiey)=f8y~%&) 2) (8(-Bx), — 88y)) (using (2)) = feey=f-64x,-64y) mel =f\(-644-64x), (-64\-64y) (using) = ftw = fe Replace x by 2). = fax, =/2",2,0) = fev, = ga)=gl2+x) [+ evengx)=f2x,0)] Hence, (2) is periodic function with peried 12. 8. (80) We have, n oh ctitnca9 100 ue =0ifns49 Again, 1," 21 ifn2s0 2° 10 Esl The desired sum =S0. ifs0 pi-4 3 ase finction} i fel 3=11 Hence, [e+ y]=[44/+ I1J=[15+=18 42. (14) Number of surjections from 4 to B 2 = Yer 2¢,09% a CIP 2c (tse DP? 26, 2) 2416 4 :. Number of onto mappings from 4 to B= 14 43. (217) weanevers{t)-s09-7(2) x ¥ (fe) =4x" +1; but f(4)=65 = 21 S #64 S eed [e4t0) $0, f@)=r+1 Hence, f(6)=6" + 1=217 14, (10,000) We have, Pe +f EH) = V3 FO) AD Putting x~x~ 1, we get, Peo +fe+2) = VB f+ 1) 2) Again, putting x=x+1, we get fort +fO+3) = V3f@+2) 8) ‘Adding (1) and (3), we get Se +2feer DA L+3) = BUO-LE+2] = 53 feet DI] [using (2)] = SV) AF (+3) = BF e+ 1)-2F 4 VD = SO-W+f +3) =fO41) 4) Putting r= 2+ 2,we get fet 1) tfe+5) =fle+3) 3) ‘Adding (4) amd (3), we get Fe-V+/(+ 8) =0 = Flx-M=—ftx+5) = £(0) =-f+6) (6) ‘ 1G:+6)==f(x+12) 1) Using (6) and (7), we get, fe) =-ES0+ 129] = fo) =flx+ 12) ‘Thus, f(x) is periodic with period 12 99 ‘Now, D6 + 12r) =f) 1 fG 412.1) 0 + SH12.2)+/(54 12.3)>..4/(5 + 1299) = L9+/G)+..+ (8) 700 times [of6+19=F00) =100«f(5) =100* 100 [as (5) = 100, given] 0,000 15, (1904 or 1994) Since, 1900 J (1990) =1994 ~B) From (2) and @), we have F(1990) = 1904 0r1994 16, (4) Given: f+ y)~ bay =f a)* 2? Replace y by —, then FO) +b? =f) 2 = Lo) =f(0) +k? wl) Now, Sf) =f()+k-2=2 > f(Q=-k+4 and, £0) =f(0)+ 4k-8=8 = f(0)=— 4k +16 which gives k= 4and/(0)=0 Thus, from (1), f(a) ~27 gor) -4 Functions 137 47. (1155) Wehave D(x)= sea(2) ms 2os(2) Now, period of f(x) 3) is7*55=385 reriogot g(a) /{)is11%21=231 + Period of D(x)= L.C.M. of (385, 231)= 1155 18, (501) Lety= Therefore, 2 2yt— = nyt 2 = = los(»+0h? +1) (Since > 0) $1) =660)= tog{x +f +1) gO 1) (eta oar 8| Soo = '0e| Soar + So = loge! =501 inthe given equation, we have F()-2 19, (26) Putting x = (fy = FM-DFM-D=t => f()=2Zorf(l)=1. If f(1)= | then putting y= lin the given equation, we have f(x) =2 fe) +1-2=9f (x)= 1 forall.xwhich isnot true as f (3) = 10, Thus, f(1)= 2 Putting y= I/xin the given equation, we get LA). F(R) SON +S (Ua) + f(I)-2 fey +s(lis) Therefore, fx)=x"+ 1 or f(x)=—x"+1 If f(a)=—x"+ 1 then 10=/(3)=—3"+1 = 3" =—9, which is not possible. Thus, f(x) =x"+1,5010=f(3)=3"+ 1 = 3950-2 Therefore, f(x) =32+1 = IS) = 91 =26 5 .(b) /(x)is defined if| sinx | +sin.x>0 = sinx> 0 2nn0,1+ + >0,2¢0 ea +) x 1 => logyg| 1+ | <- al +1>0,2%0 of 1.38 Mathematics for (IT-JEE eal!) oe nieso = ip(i) eco oles yes ea gon O-190Oandx>0 = (log,x—3) Q—log, x)> 0 andx>0 = (log,x—2) (log, x-3)< 0 and x> 0 + 2logyx<3andz>0 = Per

Oand b— = 2 #Dorpositive integer, x0 = edepeo(a) ss which i true for ll real x. : Domain( f) Let, p= log, (3x? -4x+5)=9 "= 3x— 4x +5 3937 -4r+ (5-0) =0 (eo) For xto bereal, 16-1250") 20-9 e244 we eh2 u 3/2108, Range of f [ = x0 which is true V xe Now, wehave a x-Z}< 22 sin( 5) 2 1s2 sin(x—4) +35 fi aston no $)e3} tags je, OSys? i eissdleorevaren I, (b) The function is defined for values of x such that 2 log jz (16sin*x +1)>0 Also, we have 2+ log g (I6sin’x +1) <2 + log g (16 sir?x+1)20) Together, we have 0<2— log jg (16 sin’x+1)s2 ie, -e0 re! sep x<1 s+ Domain of f=(-=. 1) 40(c). Since (2+ tJisanimeger, «sin (af2+1))=0 Spey Sndats? +1) Era Hence, Range of f= R= {0} ° 11 (b), Wehave, fig =14+ cos? . (cos) + sins (2 cos sins) cost x _cosx (cos? x+2sin?x) _ 1+sin? x cos x ‘cos? x = f'@70. 1.40 Mathematics for IIT-JEE © (is increasing function, ont” costx 3 sinx smi, (14 SR) - 3 Range = (- =, 6) 14 (d), Forf(4) tobe defined, () 2e-3]=-1,0,1 = -1s2P-3<2 528205 3 ered slsx<5 Igx? SxS-lors2t © Jecde- flac 27 V2 7542 5 Is @ Base+5>0 = x ~Q) 3) log, @7-Sx+5)>0 ° > e-sst5<(3) = 2o545<1 = Bist4 S3andx#1,2,3 2 Domain ofg (2) =(-=,3)~ (1.2.3) 4 Domain of A(x): o 3ot* 2 (3) -siS Js Thus, from (1), 2) and (3), we have Domain ofhtx)=10,2) Domain of f=[0,2)- {0} <0 68 (a). Since, g(x)is function whose graph isthe reflection ofthe graph of f(s in the line y= = gx) is theinverse off) by definition <2 3 283r-2<4 ha)= sin l= -1s = -283x-2<0 osx? 1) ity 8) S10) Now, et y=f(2) = (c+ 1, Wx2-1 {given} = G+1)= 2 Jp,920 as aE os B52) 5 0s30-2~2 z a = elthers=-14 fy or r=-1- fp = 2s3x<4 = f= fy-1 Notpossible, «7 2-1 (given), reef z"S5 (2) Pleas ve -1=96).x20 QUESTIONS FROM PREVIOUS YEARS’ lIT-JEE 4. IE/(4)=cos [r2}z+c0s [-n2Jx, where [x] stands forthe greatest integer function, then 1 Ify=fe @ x-/0) © s0=3 @ 48) 1 ) fe=1 (©) y increases with x forx <1 2 (@ F isa rational fintion of = Feast Pr (§- det 2. If Sis the set of all real x such that ae is 5: Fora pasidve iomger, tee positive, then S contains 3 @ (=-3) 140) (16) 80 90420020) (1 +see 46) ..(1-+se¢ 2" 6) then ut 1 28 @ {4 x © O(a) ©) oa} ws(E)- (©) none of these» 3. Letg(x) bea function defined on {-1, 1]. Ifthe area of oa(Z)-1 oA(%)- ‘the equilateral triangle with wo of is vertices at (0, 0) and fe, g(e)]is tie tinction ge 6. Ifand Yare two non-empty sets, where the function dfx, g(x] is J3/4, then 1 tion g(x) FX ¥is defined such that @) g@=tyi- — &) gay= Ji-? AO= YF) :xe C} forc ox and ("(D)= {:/(6)€ D} for © g)=-fi-F —@ sie 4 Or eee sr forany Ac Yand Bc ¥then 10. 12. 1B. @ f'tay=A #1) © FB) FEB) onlyitf(=a only iB f(x) x, iff xis rational the fe if xis irational oye f if xis rational (@) one-one and (b) neither one-one nor onto (©) many one and onto (@ one-one and onto Iff(x) = sinx + cos x, g(x) =x? invertible in the domain. b) [ @ [3] 1, then gx) is x aa © [- i | (@) (0.x) Domain of definition ofthe function fo @ [- (- Range of the function f(x) @ (he) @) G17) © (1.73) @ (1,715) 110,59) [0,ee)andy C= =, then Fis (a) one-one and onto (b) one-one but not onio (©) onto but not one-one (@) neither one-one nor onto Let function f: B+ Re defined by f(x) =2x+sin x for xe R.Then, fis (a) one-to-one and onto (b) one-to-one but not onto (©) onto but not one-to-one (@) neither one-to-one nor cnto ‘Suppose, (x) =(x+1)? forx2-1. [f2(2)is the function whose graph is reflection of the graph of f(x) with respect to the line y~ x, then g(s) equals (@) -Vx -1,220 — (b) tel Crt? 14, 15. 16. 11 18, 19, 20. 21. () fe+l, x2-1 Let f(x) = (1 + b?)x7 + 2be + 1 and let m(b) be the minimum value of f(x). As 6 varies, the range of m (6) is, © pt ® [od] @ an » Let B= (1,2,3,4} and F= {1,2}. Then, the number of ‘onto functions from E to F is (@) Ve-1.x20 @ 4 © 6 or @8 Let f(x)= ae 2-1. Then, for what value of ais, Sea? @ 2 (bo) -V2 @1 @a4 Uf: (1, &) > [2,) is given by f(x) =x +, then, SG equals © etx? 4 wo = 2 1 wea? =4 oS @ 1+vF-4 Let fi (O,e9) +R and Fla) = f° fle) IEF) = (1 +x), thenf(4) equals @) 54 () 7 @©4 td) 2 Hf) = 3x5, then f(x) 1 ats @) it given by | oes ©) is given by (©) oes not exist because fis not one-one (4) does not exist because fis not onto Let g(x) = 1 +x—[r] and i=l, x<0 ), then for all x,/[e(x)]is equal to (b) isgiven by @x ®t Of) © 9) For real x,the function eaene will assume ll real values, provided @ a>b>e © acoce © erexb © asesd n. 23. 25, 26. 27. 28. 29. Haff (s))=| sin x [and f())= (sin Jr then forallxe (0,1) (@) e<1tx (©) log,(1+2)x @ logsx>z Ifthe function /:[1, =) > [1,) is defined by f(x) = 26), then fx) is tye oy ©) $0+ {TF Hog; 3) (@) not defined ‘The domainof definition of the function y(x), given by the equation 2* +2”=2is (a) O” stands for “ ‘Thus, we are led to the following definition of limit ‘We-say that lim f(x)=1, ifthe value of f(x) gets closer and closer to the number /as.x gets nearer and nearer to a bout not equal to a. caution i For finding lim f(x), we study the behaviour of the function fin the neighbourhood of @ and not ata. Thus, the function /may or may not be defined atx = a. Lim Right-hand Limit We say that right-hand limit of f (x) as -xtends to ‘a’ exists and is equal to /, ifas.x approaches ‘a’ through values greater than ‘a’, the values of f(x) approach fa definite unique real number, and we write lim, f (x)=1, oF fla+0) Working Rule lim, f (x) + Putx=a+hinf(x)toget fim f(a+) To evaluate «© Take the limit ash 0. Left-hand Limit We say that left-hand limit of f(x) as x tends to ‘a’ exists and is equal to /, ifas x approaches ‘a’ through values less than ‘a’, the values of f(x) approach a definite unique real number /, and we write lim_ (x)=, oF f(a-0)= ty. Working Rule Toevaluate lim f(x). # Putx=a—hin f(x) to get tim f(a~) «© Take the limit ash 0. EE cauTion EE A numberis said tobe a limiting valve only ifitis finite and real, otherwise we say thatthe limit does not exist Indeterminate Forms Ifaunique value cannot be assigned to /(a),then f(x) is said to be indeterminate at x=, o Most general ofallindeterminate forms is, others being = 9 shih is indterminate ® Sheeg ry = 0 snd heate (e248) (b) 0D hich dete id he (b) m7 170 7 0 Which is indeterminate and hence is : which is indeterminate and hence is @ -=20 log 1 = «0 which s indeterminate and © Lety=0" => log y=0- log 0=0 x20 which s indeterminate and hence is 0° Oo Lety=o* => logy=0- log «= 0 co which is indeterminate and hence is =, Algebra of Limits If lim f(x)=/ and lim g(x) =m, then following results are true A. tim (£00) + 8G] = lim f (2) + Him g(x) =htm. 2 fim [F()~ BG] = lim f(x)— lim g(x) =l=m, 3. fim ke f(x) = A> lim f (x)= ki, where k is a constant 40 him (£¢2)-g())= Him F (0) + tim g(a im La} are 6 tim (Jog) (x) = lim s1e6o=/( Jn et) =fmy. Inparticular, @ Tim tog g(2) = es 20) log m tim st) tim 80) = er sincyeor [tin £09] '. forall ne N ‘8 Sandwich Theorem (or Squeeze Principle) Iff gand hare functions such that f(x) Sg (x) $ h (2) forall x in some neighbourhood of the point a (except possibly at x= a) and if lim f(x) =1= lim h(x), then lim g(x) =! ORTANT POINT(S) TO NOTE Himits © 9. If lim f(x) =fythen lim | f(x)| =| (EEE cauTON ES! ‘The converse of the above result may not be true, |A lim flxy=t ie, tim) foo Evaluation of Limits ‘The problems on limits can be divided into the following categories: Limits Algebraic Trigonometric—_—Exponentialand Limits Limits Logarithmic Limits Fig. 2.1 Algebr: The following methods are useful for evaluating limits of algebraic functions: Method of Factorization I(x) and (x) are polynomi- alsandg ()#0,then weave inns ef), el? 9@ se g(x) Tim (3) g(a) Now, iff(a)= 0= g (a), then (x — a) isa factor of both F(x) and g (2). We cancel this common factor (« ~ a) from both the numerator and denominator and again, putx=ain the given expression. If we get a meaningful number then that number is the limit ofthe given expression, otherwise ‘we repeat this process till we get a meaningful number, MethodofRationalization This method is usefulwhere radical signs (i.e., expressions of the form Va +b) are involved either in the numerator or in the denominator or both. The numerator or (and) the denominator (as required) is (are) rationalized and limit taken after cancelling out the common factors. Standard Formula tim ~~ = na”! sso x-a ne Q, the setof rational numbers. Limit of an Algebraic Function when x — © In order to find the limit of a function of the type: 2 as $e, wheref(a)andg (x)are algebraic functions Bt tis ‘onvenient to divide all the terms of f() and g (x) by the highest power of xin cumeratr and denominator both and use the following standard limits 1 @ lims-o tim 9, tps slim = Gi lim Fp ~0,ifp>0. 4 “TRICK(S) FOR PROBLEM SOLVING ©, ifa>l maiare 1 ifa=t abe 0, if-tg Some Useful Summations pede ag ent OE 14 2634 no @ BPP P s+ te VOY 6 2 Gi) En =13 428-439. +0 [2229] (iy) Zar" = a+ artar +... ¢ay t= provided r< 1, Trigonometric Limits For finding the limits oftrigonometric functions, weuse trigo- nomettic transformations and simplify. The following results usefil: sin ss - Oe © eet i tim = Go) im = Limite 23 e sinx® oN 0 an ima f(a 1), where a # 0, on taking 5 Sia tls 2" 730 xe O0 ii) im, as ©, (0) im c+3!"= ti (+4) a Uh = (i) Jim, (1+ ai) (i) im ®E -am>0 wey 24 Mathematics for IT-JEE iy fin 98 TF) ee, (a> 0,041) ) im (+2) - ren () lim (+ Gs) Him + FeO =e. ‘Some Useful Expansions (ie Gi) tog, +3) =: Gv) log, 2 (parents slogat ERED 5, tow n( (vi) (4x)=1 ++ x? 4. toe, -10 and tim g(x) =B.then Jim LF Cas =? 3 HE Sim £2) = 1 and fin g(x} = then iB reopen 4 Evaluation of Limits Using Hospital's Rule Besides the methods given above to evaluate limits, there is yet another method for finding limits, usually known as L Hospitals rule as given below for indeterminate forms: 0 (2) i, = in) -0 8 im LO EO, prose the limit on the ee eG on Bt) RHS exists ii) (=) ) form: He lim f (3)== and. fim, g(x) ==, then tim LO. = tim LO), provided the limit on the fae g(a) 50g") RES. exists, Here, f” is derivative of f: Note that sometimes, we have to repeat the process if the formis © or = rms Por = asin TRICK(S) FOR PROBLEM SOLVING f&) ‘= Hospital's Rule is applicable oaly when: a) os becomes ofthe form 7 oF =. 0 + Huheformisnot 5 plifythegiven expression till it reduces to the form : or LHospital’s rule. '* For applying L’Hospital’s rule differentiate the numerator and denominator separately. and then use GE caution L Hospital’srule cannot be applied in every problem. Bxtsi 2 i ig ig 2 Pn 2) Here, if we apply L’ Hosp’ rule, we get Bxtsindx | 3+2cos2x jim jim —* OSS sou 3=200s2x roe 3x-sin2x Now, beth the numerator and denomin because lim cos2.x does not exist vor are undefined Wecan find the above limit as 342 3x+sin2x in tine Se Sra sind ae =Aysince tim Sin2x ein OE (a) [2x|>v3 (b) j2x|/< v3 (©) [2x|203 @ l2x1sv5 & tim | cos ~ cos * cos ~ ...cos + |= pal (o guhg Sg eee ) x sinx © Ginx 7a oo (@) none of these % 10. ot @t OG 3 Let x)= {22} whee (4 denotesthe fational partofx. Then, lim, f(s)is 1 1 Ms (b) 3 1 @t (4) none of these ©5 ® If [x] denotes the integral part of x, then tim 4) Sa rom LE x 2 @o > x x oF os 61,8, .1 B 1 fim |tand+4 tan 4 tn 244+ A tan s(t funds dim S + jan$} 1 Os (©) 2eo26 fo) gn 200128 (@) none of these Limits 25 PROBLEMS FOR PRACTICE 1 @ 5 (©) does not exist (@ none of these tim 1080+) xo a} where (+) denotes fractional part ofx, is @ 1 (b) log? (©) does not exist Let {x} denote the fractional part of x. Then, (@) none of these isequal to ro tan x)” (b) 0 (@) none of these 1,2,.04mand ifa, 1y>0.fO9) is equal to @) fifo ) fo)-F0) «ld (b) foy+F0 (@) none of these tim E21. where [-] denotes greatest integer eA sin| x a sentences function, is @o 1 (©) does not exist (@)- none of thes The vahe of fin Tis eo (a) 1 (b) 1 0 (d) none of these tin | 3222], where denotes he west iene function, is 23. mM. 28. 26. 27. 28. 29. (b) 1 (d) none of these where [+] denot (a) = sin} (©) sin est integer function, is @) sin2 (a) sia Ifthe rih term, ¢ of a series is given by 7 then fim XN is @l (b) a 2 \ oO 5 (2) none of these The value of im n fa) nt ©) GD! (b) @o fim (14x) 1482) 424) Pla his equal to @ ) (©) Ix x1 lim (a) no value of a (b) all values of (©) only negative values of (4) only positive values of 0, (ninteger), for 34344 ctn(nsl) lim is equal to @) I (b) 1 1 @ + ad) none of th ) 3 (dy these 4 1 e052 Wa min fe2+4x+5,'€ Rj andb= lim “SS oye then the value of SY ab?" is ra (a) a (b) 27t-1 : 4-2" atty ce) = (d) none of these & py « 30. 31. 32. 33. 37. im HORCL+ x42") + log (I+ is equal to eal) seex—cosx @ 1 (b) -1 ©o de Trevaheot im YEA? wm ns Yee @t ) 0 © @ = vx +r + sie fim The vale of fig FREES is 2 OR b) V3 1 OF (@ none of these im = isequal to (ea + RE) z 1 @ ar © sr © 24%, {d) none of these In a circle of radius r, an isosceles triangle ABC is inscribed with AB = AC. Ifthe AABC has perimeter =a dair-t+ Pr | andaread = yP3ir =H? , where his the altitude from. to 8C, thea tim, 4 is htt equal to U 128, (@) 128" o) (8) none of these (0) =i (@) none of these = 00829) i, saa 10 xSUinGinkkt) 2 a os Ds: © 0 (@ none of these Li c08 (ex? + be + a) (x09? ax + bx+c=0, is equal to lim tle + where @ is a root of 38. Limits 27 yy Bde 2a ® o (d) none of these 2 2. 44, 1 &) — ma" (a) cost 8 in cos 8 sin‘ O in sin @ (b) cost 8 in cos + sin® Bn sin 8 (©) cost 6 in sin 0 sin* 8 hn cos 8 (d) none of these Is tn (iz) . Sol x (a) el? (by 1? @ eM (@) none of these las wy | fae tb) fe el? @ ot and j denotes integral part, is equal to (a) a+b (b) a+ b-1 © a-b @) a-b-1 2B Mathematics for IIT-JEE ts hm ADE + edt fm) sa. lin tim, fe aS eache A Te a ++ mp ee @ x () 2 LAU PDE So edt ee Ur ot wo (a) none of these ™ @ 0 o1 $6, lim n?(x!”—x!"*"), x50 is equal to 1 we oa M5 @ 0 Oe 2 © ins (@) none of these B 55, The value of tim Ff smet J denotes the x an af am Leet sar. 3, greatest integer, is @ 0 @1 ir sory n tim {1+ LOP _ om sin ¥ @- @ ; @e we @e (@) none of these 6. The value of fim |x, where ['] denotes the Rey reatest integer, is 48. Ifyax+ sthen im = isequal to oo 1 @-4 (4) does not exist ‘ 87, Ifa =1anda,=m(1+4,_,), ¥ 22, then @t a @o (@) none of these weft lod} ed] 8.x (60s x)" 4. a | a) 0 (bbe ae cesar!) | oe (@) does not exist f@) 0 (b) 1 | none of these 2 (@) none of 1 a ign fea) = tan x 0h Thevaluc of son ia (tanx)—tanx +1 @ 2 (b) 1 ©) 0 (d) none of these 59. eo? x gles Ye 51. lim (iors 28ers @ wae _— nor © y {d) none of these fa) Oo) = ; i! (@) oF the rt © (@) none of these beat xa Sal # a She) ee t i @o ) x @o (b) tan! * a © > (a) none of these OF (@) none of these a 4 tim ([2si0z), [tans 53. Thevahcot lim [sinseossjwnere(-Jdsnoes | 61- Thevalueet fin || ]*| "| wher) the greatest integer function, is denotes the greatest integer function, is @ 2 ) 2 @n (b) 2n+1 1 @- (© 2-1 (@) none of these 62. o. xo| sinx tnx. integer function, is (@ 0 @2 |i ( al? +x! ax)" tim (108 (1+ x) ~ log2) (3.4 ~ 3x) Fl (74) = (14 3x) sine @ 2t0g4 xe © 2 t0g2 2 fim O=2M=2)- =i[0-3) (1-2). @ n! 2 in| = |, whee [1 dome he pene ®t (@ Coes not exist sthen lim f(x) = >) -1 (@) does not exist Oa (@) none of these 4 ©) 3 top4 ze (@ none of these 69. 10. n Limts 29 (2n)! ay (2) none of these in 3) n 1 An if costar =A forany &2 1 Yeos tar AE foray ‘ = , +x and @ Zor) ten | ee is @l () 1 1 1 © 5 © -5 If and B are the roots of the quadratic equation ef1_t efl_d felts) » bites © (al (@) none ofthese _|* Jeny | x20 Visteot } , x<0 1e value of cot” | tim f(x) | is rennet (3 @o 1 © (@) none of these If ti +a,b,c€ R(0} exists andhasnon- #90 sin (x°) zero value, then (@) abcawinAP. —(b) a,e,barein AP. (©) a,c,bareinG.P, (a) none of these L lim_ lim (1+ cos?" nlzx) isequal to @ 2 @o 1f tim 241 xt @) a=1 xt a-b 1 ) 1 (@ none of these © 6-1 r= Ie tim U2) +8" > then Solsd=oe™ (a) a--1 (b) a=1 © b=03)!9 (@ 33 tr im £04282) 0 e 240 5 Mathematics for IT-JEE (b) () cate rb +e) a (a eteetate (a) (a+ bee) (6) abe 298-243! 81°94 1 fig BET acta a, 5, = ¥ then’ ino lat 2= 2 Hogan) COS h S20 TOs» C0822) tin ( nO isoqalto (@ eifm=1 (b) Lifn>1 (©) cifaOfor all me Nythen lim s(n) is equal 10 IE f(@) is polynomial ststying FLOFO)=F)+/0) + f(xy) — 2 for all real x and y and f(2) = 5, then Jim f(x) @ CO) 0 @ o. \ 9 4 7 1 5 1 O2HOO wOoGOOe Copyrighted mat 1 —sinx + Insinx 29 133 S42" Stee 10? 10 10" Nim n+? The matching grid: We know that if lim fix) =/and lim gix)=m(#0), then tim f60) £5 BO m LD. SgG) ~ im 20) However, if lim g(x) =O= lim f(x), we cannotsay anything definite about he existence of im Bey Thou 0-90 in some cases this limit exists. Any expression of the type 3 or = is termed as an indeterminate form. Many other ‘expressions like 29 ~00, I 0°,0 x 1 6 Amertion : tin S704" soy 2 | Reason: tim #0420" <9 | % mor 8 | . In(l+x)—x _ 1 | fig MOD rs | r tli PVFEN = e, wh 7. Assertion : lim ((/(2)]+ 2°) e where fo)= 822 and [+], {} denote integral | and fractional parts, respectively @ 20 3 a) 40 3 0 eo ne 8 oo 12 16 2a 326 We 0 16 @ @ 6% 626 me 206 «woh 2309 2 o @ 2.) oa 2) 3.) 8.) @ 3% 3.0 3. a 3% 6 382 4 3%) 40.0 @ 2% 4 @ 4b 45. @ 46 7) 48. © 50.) «= S15) 53. HSA 5S. a) 56, HY Ck © 6 @ %0 8&0 68 @ 01@ 7. @) LOH 2409 3 60 4 oO 3% 6 6 @bO 7% ba is 1 169 (0,11 @), Ilo bb), Veo (a) 2.1 (a), Ne (0), I > (b), Vio (2) nae i 244 Mathematics for IIT-JEE 1.) 2 2 @) a0 3.) 6 nw a ©) %@ 1 6) 1%. &) 1 @ 2 id) 3. @) a @ 5. 6 @) 2 @ 8 @) 2 0 nol Now, S= Da ‘= ie JL = Bar? + (0? = 20— dr Py rst = se Hffeee2f-aofbneeencn en} alt 2 +(0 = 2n- fae + of mon] Solving and rearranging, we have, = Lat 11m =19n? + 6n} Fry 3a), The (r+ 1)th term ofthe series is har e+0 DE Ll Saye. = tim +] Sex thay H+ 142434... 2-r) ters} BAG) Hats sen (Se | = fase HD gO =nn=r+d) Since Pr-1s (Be 0 for 2 and, Similarly, if isin the right neighbourhood of a, Then, x—4,0 for Dyan j 2.46 Mathematics for IT-JEE x4, AL 21 fori=m+l.n “= a) and, 4 1 for #=1,2, xa, Now, fim (4,43 0. 4y) = 1"! and, lim (Ajay 4p) = me Hence, fim (4,4... 4,) does not exist. 418 (b). We know that lim 22 or lim — 5 1¢ J Svsnz lim [io] + tim [59222] = 100+98=198. saol ding] ” x0, “4 i atal 2b rel rtrd 2 “2Lr@=Del (reread Eiir-s0+0). ts Fo-DF1 =O-F+0] where, f= —_) jh snoe Ga+inei] ” 2 Ve al 4 3 a al TE 25(0). in [tiene] st 7 | ta [ $2710" tn? | ee 1 ae [ees s| sy [Os seenson, 0] eel rye weg og? 163 +. 4 oe) | clos: 2-3.m <1 | 26(0). tim +) +2) 44.202 | tim CIDE A437 0434). me 1=x 2y0422,04e4 2 = tim (EExd txt) ) ne I-x = lim “ wr | lim — = lm is tm Mena? nt | [Using L’Hospital’s rule repeatedly} | -0 | Case IT. n is anegative integer. .m, where m isa positive integer] 1= 082 _ j.,, 2sin? 6 0 6? a Zar oer aes abla ets ta | Also, b= lim 0 30¢0). ty, bats xa? )+eg(l=x42°) 20 (1-cos? x)/cosx. = tim B+? +24) 0 sinxtanx log(+x7(1+x7)) x0 (Lt?) -1(« tim B+), ) ax (=) 2422 P04) se 33(b). tim 282 = Saas ae xz) xV2xz—x? * 30 Ux Wer~4x VE Vay tim OME 0 [lemaeVE] ve] 1 Limits 2.47 hydhr- ALi | 94 (0). tim A= tim —*VP = fete! daw | Paina + Pir} tim EE 108 i/o hdr] aja _ 0g [ rh +m] oP a 8(2Var)) 1287 ais tagcat ig tee mney T2642) [putting x- 1/3 = 2} = jim = 77° (1 ~-cosz+ v3sinz) In sin3x) tim 12082 © 290 (sin3x) = tm 2522 290 (3x) 9 1-08 (cx? +r +a) ‘37 (a). li Oe (= xa)? sole 1008 (ex? +bx ta) (ex? +e +)? = ii wile (et eheea? Osa £082 ing (ay? +by+ 0? BV ie Foor {putting ex’ + 6x+a=zand x= 1h] 2 -a)"ly ~ BF Piy-ay [If@, Bareroots of ax + bx +e "4 ye then ax? + bx +c =a(x—a) (x- Bl] 2.48 Mathematics for IT-JEE [Putting 2” +x= "and 2" +x= 9") ! 1 weet pet Tim (£280) = in6)* ~c0520 4 x4 41 ( +4 (siny'* 4 (eos! @ ~sint fim ©220)* = (end)?** (oe sintO) fat 7 {Punting x4 = y and cos 2@= cost @-sin* 6] = ino) = tim owt of eae =I] setae] yo y =cos‘ 8 in cos @~sin’ Binsin @ ‘tecaxt 1a (totes) roteeosx im ofl 4200) in (ESSE) i, 1 Now, fim in ox Hence, limit doesnot exist. 3 fen 28a) In (dbz r+ 7-3)2x- ta 39(b). tin, in= I 2 on 2 , [raeer=! co nt in erases * |e im GOSE-! cos (5467 +264 +6) Gx-5)® + 4Gx- 5)” wg (2 2 ) tN 8 Gra) Herce, the required limit is e"!, [ueeobe eat 7 43(c)._ lim = lim | — =i eure Ts geod = , 4c POPE 2 ii Gx — 5)! 443%-5? “| | 23x) yo a tay 40(c). ti, +9" +) Limits 2.19 47 (b). We have, us = tim lim, [rexeL] yao ao) x ; Anfl+ xe g(x) wt 242 os [rai LB ts yoo? 4 90 x Hence, the required limitis e'?. =o tim “l+x+s@) asinx ban. 90 a ae in [2] re should also approsch zero for a finite limit to exist [exo S22 ort Ear | Thus, we have, * * lim g(x) =0 rite 5 1 ue OE 51 m0 wae Te ‘Now, using L’Hospital’s rule, the above equation reduces to =(a-1)+b tim +8) =atb-l suoT+x+t g(a) (x)= 1+ a+ Ba] +4 fae] | H8O~3>g0)-2 4B (a Lae 14243447 Hence, we have, is il +e00) Nom melee; lim LQ] lime > E428 + Set at me ole =o IOS T2434 tn et) = lim elt = 80 1+ Gx-+Ge-D+. = lim el #09 = eo | 2 i ve ve = x 48 (a). Lety=x+ art in nei fa). Lety=x 7 aS [ex-Isielexv xe R] Thus, we have, 2 TH Oss ‘Now, we have, fin x= sang lita Hence, by Sandwich Theorem, we have lim f(x) =x 46 (c). mr? (a 2 1 ( 1 = lin 2 een aor Bi livar cad 5 49 (0). Weave, 1 tim 2082 = C6082)" = fine aD <1 —c08 + Ta (cos) ed (cos.xF =1einx 1=Inx - inon| Se] L-dof = tim SOT [Putting cos x —1 =] Sondro-t 3 4 pa fend fe-pe-2) , = km 2 50 (a). We have, (tan xy" ~tan x soon In lan 3) = tan xT lim, ona) Him (IY +2" +t)" snl To EF] n(0+0+..+1)=n (eee) (F412) (1) i (rp ee(rs5) = lim cot [Putting tan.x =i) cat! a cot" 4] "43 53 (b). Here, sin x +cos x= Bisa v4) Fors 4h. sin{ x=) ors Eth V2 + but greater than ~ J [2an(2] -2 0 lies meall a Aso, fore > on sB ial w+) 9-8, bu , i ( ‘| A, but ‘greater than — J famliet]-2 From (1)and (2), we get Tim [sin.x + osx} = Tim 2) 54 (d). ( ($] somra= eee mint = lim “Ine 55 (a). Weknow, which tends to one a8 7 oo but always remains less than ea = 56 (b).. Here, 0 bxJ=n sine Here, nasx 0 but less than tan 3 nasx—0 but more than Jennsx0e |) Also, me r-t4[ aa You have either reached a page that is unavailable for viewing or reached your viewing limit for this book. Limits 2.23 68 (c). Since maximum value of cos"! = = inf @ teste 2 = lim, 2 ot js possible ifand only ifeach tim + 8-2, =e xtx tm (a= 204 90 z+? 14h 4004) " 3 lim 30 x(l+x) Lelevoty “aie — So Tee BO letet (0.2) means tems containing 7,2, = fim (iy eRT=TS = RT 69 (2). + ax? + bx +¢=Ohas roots eran f, therefore ity Tim, f(a) = fim St lee Oiene?t bet an Obes roots an 5 ASO Ale - tan? i? = ferbet}a(e2 [5] ae =" S & ss lim f(t) does not exist, ~ cos (or? +e +a) ~ Now, lim, 2 fen 2 5in? x ot 2a} 710. fi ay 2 ue aig? tte = lim xatbre( SX lim, 2 = 50 x in(s°) “al d-ax? ‘The above limit is non-zero ifa + b—c=0 4 (a,b). We know that | cos0{< 1 forall @ afl) if |cosn! mxI<1, tell* a)? So,if |cosn! rx|<1, = 2 tim —0e8e!*CI/?) Jim_ lim (1 +005?” n!ax) =(1+0)= 39004 (1=5°)el/* (=1/ x?) andit cost nx|=1 (Using ‘Hosp srl} lim, lim (1-+00s?ntex) = tim Tim (1+12") a: ABR ees ea an men Leasyeen eet? ‘ Pomc al 14a) +8e"" (00 ae dita aha 8 (bye). We have2= ig (E fom) Hence a= 1 and 5 =(~3)!2 2.24 Mathematics for IIT-JEE wt elt 5(c.d). lim eo - (2 pied comet I Patina ee ) = lim e? } ya Now, we have, Man gg | ee yoy 3 = tim > Inf 1 ( a) +b" re" ge Behe Sy soy t ] ah 4b 40" [rin =Ina~Inb* ine= In (abe) Hence, the required limit is e!” "9 = abe 7 (a, b,c). Using the expansion, we have, ‘Now, above limit would exist if least power in numerator is greater than or equal to least power in denominator ‘Le, coefficient of x and x? must be zeroand coefficient of x should be 2, 4(8).. Required limit = fim 282°@*=)-9"E% -Y 40" a) Pd GE =1) 1243)" = G7) = 9" +) = tim rae si GB -)O*-NET-D) soe log 3-log 9-log 27=log 3-2log 3: Slog 3 ~6(log3))~Kilog3)> (given) k=6 lim cos{ er +n a colnet) ~sosafeie cee] tin cos{ nae 3 20). = lim sin( ne or ager (27 423 -6.27)/2 318). tim eh geet Qt =a" (2?? -2) ner- mn im 242) Q?— = =) = lim (24? +2) (24 -2)=@ +2); (4-2)=8 4410), tiny Bees 08S 150 Foc, CON} Incosx in «Si Tse) Theos) fim 08x =in (60x) +0 In cos(x/2) In cos(x/2) 2 = tim { tons Lin cos), = [mises P| cose=t cos(e2)-1__eus-1 “a(t +e0s(2)=1) cos(2)=1 7 =2¥x+1 1) in y2-2y41 hg [Putting Yy =y:a82 1p 1] 12 “im OU) =i rt PIG-DG DE GF eyeDE 9 2.25 lim f(n+ 1) =k (say) 6 (3). Let lim fin) were sms 4 {re9+ 25 1 9 = tim f(a) = tim fort) = tim L] f+ PIO ~ etry tg Z iis) = ke}{ke2 |< -90rk=3 2 k es tim f(a) =3 7 (40). Putting x=2 and y=1,we have SOFM=FA+FM+F2)-2 = SM=8HTH=S0) Putting y= 1/« in the given equation, we have FF Oe) =f) +f ix) +f) 2 =f) +f) Therefore, f(x) Burs=/(2) |. which is not possible. = m=2. Hence, lim f(x) = Him (2? +) =10 ML (b) (%— 2ith factor of the series is mol antl nel wonsh 23 w+ntl i . nanniaeD 7 1M. (8) im (-=#]/ 226 Mathematics for IIT-JEE 2A. (a) Letsinx=f, thenas.x— 02,1 given limit tim Bat gg Leben int “hols helnh hoi =141/h [Using L’ Hospital rule) nim SE] 25 finite quantity ‘sin? (1!) always lies between 0 and 1. Also, since 1-#> 0, cn!-#-ycoasin yoo} | =0 4 (0) Let = tim S122asinx (2) Oh 0 . = im 260528 tacos an 2a 20 3x? 2 2(a+xWa? {Using L’ Hospital’ rule] aaa We require 2 cos2x + acos x= 0 forx=0as denominator n iszer0. . rouse ny et 0’) (0% i” = fn Aen coset Hence, 3 . tim —" wa (°) Nycocreat _ Mo zeons 0 Eafe tn (282) (2 0 6 6 6 (b). lim (I+ ax? +bx +e)" 2(e). tim Ja? — 2? cor 4 [A= * (G-2 form) tin tafe saeco] 0 m2 IM) (Bm) | iste gate “2 0, Bare roots of ar? + bx +0=0] Limits 227 9(€). We can see that 13°5...2n—1) 246 ..2n = Os im t, 0< lim (Bees, + {nx} on E According to Sandwich Theorem or Squeeze Principle fig, LEE corer 11 (b). Wehave, ‘Thus, we have, Pex-1s[Px] tan a 2 ~ oa z Loos ld = lim ~cotx +] o-1 (@) none of these (cosx~1 (cosx =e") Fa 3, tim LOA#2+H)- FQ) 40-70 The integer n for which tim soo) a given that ’(2)=6 and isa finitenon-zero number is €@)_ does not exist isequal to -3/2 1 Og e 9 : ) 3 @4 © isequalto3/2 ——(@) sequal to3 10, Let/: RR be such that /(1) =3 and/”(1)=6. Then, 4.0 Forxe R, lim He ee tite (a NES J equals @e so FO oet i @ | w = we @e 5 by He ee =0, where nis non- zero real member, then ais equalto cos? x) @o © ntl Ai. The value of Jim, = © fa)! ) +1 an @ neh wo (@)_ none of these was 2. Iff(a)=2.f'(a)= 1, g(a) =~-1.¢'(a)=2, then the value & Fors>0, tim| ins! 4(L)"" lig of tim £2) fa) gla) f@) p at = x0 xa = @0 o 4 1 ©1 @2 @ © 5 @ s (@ none of these 2.30 Mathematics for IIT-JEE 13, tim = 8082—D | xt sin( 1) 42 mle 17. im x = (2) exists and it equals V2 sel) ° @- (b) exists and it equals —/2 | @ (©) does not exist because x1 +0 ; ot (@ none of these (@ does not exist because Le-hand limit isnotequal | Fo-3 torighi-hard limit 18. 1/00)" 9,/9)=4,ther lin, 4 es equals... @4 () 4 Tar equals oo @2 19, ABC is an isosceles triangle inscribed in a circle of 124 14, im -}. a (@ 1+V5 (bo) -145 radius r. If AB = AC and h is the altitude from @ 48 @ ne 4 to BC thon the triangle ABC has perimeter P= 1(2hr—H? +VIhr) and jim “=... 18,166 6)=—J25=2 then fim C= GO . pant OPS @t 4 4 wt i 4 oR 3 Og © © V4 (@) none of these a | feos? eat ik TERS { HRA MBNADELED soy | og 8 . 2 otherwise (20 Ry same xt +1 x#0,2 @1 ) +1 ae)=) 4 x=0 then tim gi f(a) is. | @o (@)_none of these 3s oxez son CLE 2A) —2InC1-+ hi) _ @o oA a ie ©. (@) none of there @1 4 ©o (@)_ none of these 1 © 20 2 @) 4 @ 5 @ 6. b) 7 @ & @ 20 060@ We 2 @ o wa BO we 17) 18.) 1.) 20, 21. Differentia REVIEW OF CONCEPTS Continuity In this chapter, the concept ofa continuous function is defined in terms of limits. Most of the results in calculus are not true unless We are dealing with functions that are continuous. We may intuitively think of continuous functions as those functions whose graphs we can draw without liffing the pencil, A formal definition of continuity follows: Continuity of a function at a point A function f () is said to be continuous at an interior point x =< of its domain if tim f(x) = f(a). In other words, @ function f(x) is said to be continuous at a point x = a provided left-hand limit, rightchand limit and value of the function are equal IMPORTANT POINT(S) TO NOTE function f(x) is continuous at a pointy Continuity of a function on an interval Continuity on an open interval A function f(x) is said \© be continuous on an open interval (a, 6) if it is continuous at each point of (a,b). Continuity on a closed interval A function f(x) is said to be continuous on a closed interval [a, 6] if (® FG) is continuous from right at x Jim, Se) =F) (i) f@) is comtinuows from left atx = b, ie, im SO- = 70) (iii) ,/@) is continuous at each point of the open interval (ab). Continuity at end points of interval For continuity of f(x) at the end points of an interval [a, b} we musthave © Jim (a+) =f(@atx=a # fim fO—M) =JO)atx=b. Discontinuity of afunction A function f(x), which is not continuous at a point x= a, is said to be discontinuous at that point, TRICK(S) FOR PROBLEM SOLVING ‘The discontinuity may arise cue to any of the following situations: © Jim, fle—m) Jim, f(a> Mie, LHL and REL. exis, but are not equal. lim fe—h)= lim f(a+h)#f(a) ie, LHL. and R.HLenist and are equal, but are different from f(a). # (isnot defined. ‘© At least one of the limits lim (q ~ h) or lim ‘At least one of the limits lim, f(a ~ h) or im, (a +h) docsnot existor atleast one of these limite is i Ot Some useful results on continuous functions 1. If fand g are continuous at x= a, then (@) +g is continuous at () f= 2 is continuous at (©) fais continuous at (@ figs continuous at x= a, provided g (a) #0. © Mf is continuous at x = o, where k is any real constant 3.2. Mathematics for T-JEE (1) LF CaM*iscontinnousat xa, provided | f(y"" is defined on an interval containing ¢, and m and n are integers. Iffis continuous st « and g is continuous at (a), then gof is continuous at a. 3. IE,fis continuous at x= a and g is discontinuous at xa, then fg and f~ g are discontinuous at x~ a, ‘whereas fa may be continuous at x= a. 4. Iffiscontinuous at x =a and f(a) #0, then there exists an open interval (a — 8, a + 6) such that-7 x= (a—8, +B), Fle) has the same sign as f(a), 5. Intermediate Value Theorem {a)_Iffis a continuous function defined on [a, 6] such that f(a) - f(b) <0, then there exists at least one solution of the equation f(x}=0 in the open interval (aby (b) If fisa continuous function defined on (a, 6] and k isany eal number between (a) and f(b), then there exists at least one solution of the equation f(x) = k 6. Ifa function fis continuous on @ closed interval [a,b] then itis bounded on [a,b] ie. there exists real mumbers ‘and K such that ES f(x) SK forall x¢ [a,b]. 7. Every polynomial is continuous at every point of the real line, 8, Every rational function is continuous at every point where its denominator is different from zero, 9 Logarithmic functions, exponential functions. trigonometric functions, inverse circular functions and alpsolute value funetions are continuous intheir domain. Types of discontinuity Removable discontinuity If fim f(a —) and lim F(ah) exist and are equal, bu arent equal tof (a, then the function f(x) is ssid to have a removable discontinuity al x =a, However, by suitably defining the function at v= a, (x) can be made continuous atx = a. Discontinuity of the first kind If fim f(a ~ ) and Jim, f(a) exist but are not equal, thenthe function f(x) is said to have a discontinuity of the first kind at x =a. We also say that fix) has jump diseontinsity at.°~ a We define nF) fam, Fo) ~ jump oF the function atv a. ° IF Bim, f(a) exis bai mot equal t/a) hen the function fe) is sad to have a discontinuity ofthe firstkind fromthe et atx =a. Sinn, lah) exstsbut fb notequal t/a, then the fanetion /(x) is said to have a discontinuity of the first kind from therightat.x= a Discontinuity ofthesecondkind If at least one of the limits tim fia—h) oF lim fla h)doesnotexistor atleast pono fae ‘one of these limits is © or ~, then the funetion f(x) is said to have a discontinuity of the second kind atx =a. We also say that /(2) has an infinite discontinuity at x~ a IE im, f(a 1) does not exist or sequal to. = ore pow then the function /(x) is said to have a discontinuity of the second kind from the left at x = o. Discontinuity of the second kind from the right is similarly defined. Geometrical meaning of continuity 1. A function /(x) will be continuous at a point x= a, if thete is no break or cut or gap in the graph of the function » = f(x) atthe point (a, f(a) Otherwise, it is discontinuous at that point. A function f(x) will be continuous on the closed interval [a, 5} ifthe graph of the function y= f(s) is an unbroken line (curved or straight) from the point (a,f{(a)) to the point (b, (5). y 5 sta), BS ly@ te él i Ol a-hx-armath x Fie 3.1 f(dhas acontinuoes griphate= a 1 Fla-W) Of xea-hy=axcath (i> 0) Fig, 32/4) hus @ discontinuous waph at x= a Differentiability of a function The function ftx)is differentiable ata point P, ifand only it there exists a unique tangent atthe point P. In other words F(a) is differentiable at a point P if and only if the curve ‘does not have P as a corner point. Le. the function is not differentiable nt those points on which function has jumps {or holes) and sharp edges Consider the function f(x) =|) Let us draw the graph of this Funetion which shows that/{x)is not differentiable at x= asf) has sharp edge at x= 0, Differentiability of a function at a point Let f bea function defined on an interval 1 CR. We sey that J is differentiable at an interior point c€ I provided f08)- FO. lim We denote this limit by /(c), called the derivativeoffat e enistsand is finite, I TRICK(S) FOR PROBLEM SOLVING In view of the definition of limit of a function fone may observe that/"(c) exists, provided L(x) = 0) LE)= fle ‘The limit on thelefi, denoted by L.f"(c), is called the lef- sand derivative of fat cand the limit on the right, denoted by Rf (c),is called the right-hand derivaiive of fat Thus,/ (a) is differentiable ats~cif Lf’(c)-R/"(0. "(= tim Differentiability of a function on an interval ‘A function f(x) is said to be differentiable on an open interval (a, 0) iff) differentiable atevery point ofthis erval a.) Tes differentiable on a closed interval [a 6] i iis differentiable on the open interval (a, 5) and the limits ‘eye tim L02D=L(e) R. = lit ON tai LO-H)=S(6) (b-h)-b and, L£@= Jim exis. ae Some important results on differentiability 1. Every polynomial function, exponential function and ‘constant function is differentiable at each point of the realline. 2. Logarithmic functions, trigonometric functions and inverse tigorometric functions are differentiable in thei domain. 3. The sum, difference, product and quotient of two differentiable functions is differentiable 4. The composition of differentiable functions is a differentiable function. 33 5. Ifa function is not differentiable bat is continuous at a point, it geometrically impliesthere is asharp comer oF kink at that point, 6. Iff(x) and g (x) both are not differentiable at point, then the sum function f(x) + g(x) and the product function £(8) g(x) can stil be differentiable at that point. | _TRICK(S) FOR PROBLEM SOLVING ‘= Ifa function/ (x)is differentiable ala pointx—a, then itis continuous at x Explanation Let function be differentiable at. Ther, tin LYLE - » (e) 0 In onder to prove that fis continuousat x=, itis enough to show that Him £() (©). Now, lim f()= tin (So JO \o-o+ 0) on (ee = 7” wo}ene LO LO tim (xc) + fle} f(e)x0+flo) + tim f(x) *fle) *# If /(x) is only continuous at a point xa, there is no guarantee that (x) isdifferentiable there. ‘+ I£/(9) is not differentiable at x~a then it may ormay not be continuous at.x = a Explanation The function (x) =x is continuous at x =0 but itis not differentiable at x =0, as shown in the figure given below, y fo) =I) Fig 3.4 Clearly, we have sharp edge (corner) at x=0. 34 Mathematics for I © If {() is not continuous at x differentiable atx = a, JEE a, then it is not * Iflef-hand derivative and right-hand derivative of) atx= aare nite (they may or may not be equal) then £2) is continuous atx =a, PROBLEMS FOR PRACTICE 1 3. 5 6. D1} denoves greatestinteger function), then f(x) is (@) discontinuous at some x (6) continuous at all x, but the derivative f(s) does not exist for some.x 1 (© F(x) exists forall x, but/“G@) does not exist @ FS" (exists forall x Let R > Rbea differentiable function satisfying /y) Slx—y)=f(a)¥ x, Rand f"(0)=p,f’6)=4, then SS)is, @ Pq © pa 8 © @4 The function f(x)= r+) x20 is 0 ve (@) discontinuous at only one point (©) discontinuous exactly at two points (©) continuous everywhere g (6) none of these LeUs)= (gga F 8 W2- His comtinsousat = ni then fia @e (b) 1 @ 0 (d)_ none of these Sex) = cos (|x| + [xD then fis (where [] denotes ‘greatest integral function) 10. (©) continuous at «= 12 () continuous at. (©) differentiable in (~1, 0) (@) differentiable in (0,1) sitll teal a4 =I, x=0 u. f= denotes the ‘greatest integer function), then (a) f(2) is continuous at x= 0 (b) tim, fix) = © lim fo)=1 ) none of these min. tx,x°}, ¥20 HEf@)= min. (2x, (x7 =I}, x<0 ‘Then, number of points in the interval [-2,2] where is non-differentiable, is @ 1 2 3 @4 The Dirichlet function, defined as 1ifx is rational Se)= { (a) continuous for all real x (b) continuous only at some values of x (¢) discontinuous for all real x (d) discontinuous only at some values of x Letf: RR be a function such that g( 242) - £020) 3 a Then, (a) f(x)is a quadratic function (b) /(~)is continuous but not differentiable (o) f(x) isdifferentiable inR (@) f(@)is bounded in R Let f(x +») =f) S0) for allx, ye Rand f(s) = 1 x6(@) log? where im, 6 (2) =1. Then, f(a) is equal to i fa) log 2/ {o) log2 Oif x is irrational” +f 0) =0 and f"(0) = 3. (©) log fe)? (d)_none of these ‘x, when x is rational 1x, when x is irrational” (a) f(x)is continuous for all real x (b) f(x)is discontinuous for all real.x (© fla)is continuous only atx= 1/2 (d) f(x) is discontinuous only at x= 1/2. itye= 12. 13. TH 15. 16. a 19, Left =f) fO)and/Q)=1 +318 IG Cuter | lim, g G)=aand lim, G ()=6. Then, (x) KY ts), Wile k is equal tb @F (b) 1+ab © a (@) none of these The points where the function f(x) = = 1S x53, where [1] denotes the greatest integer function, isnot differentiable, are @ x=-1,0,1,23 © (©) 3=0,1,23 @ 1,0,2 1,0,1,2 foy= tste[s + S]e[x +2]wre R.Then,nurber of points of discontinuity of (x)in 1, T)is/are @s (b) 4 ©7 (d) none of these Leta hiotion fn Rest he equations e3)= ‘f()+f(9) forall x,y. Ifthe function f(x) is continuous atx=0, then (@ £G)= 0 continuous forall x (©) Je) is continua tor al positive eal (©) f()is continuous for all (@) none of these The funtion (x)= [x 20s (2£=")e where denotes the greatest integer function, is discontinuous at @ allx (b) all integer points © 0x (@)_xwhich isnot aninteger. The function (x)= [x}*~ [x] (where[ x] is the greatest integer less than or equal to x), is discontinuous at @ ) © © all integers all integers except 0 and 1 all integers except all integers except | w discontinuous at the points @ (©) x=1,0 (b) x= (@)_ none of these If fz)= Say |x If, where a;e R, then ae (@ Gis continuousat x= forall age & (©) /()is differentiable atx~ | forall a,€ R (©) SGdis differentiable atx= 1. provided a,, . (@ Gis continuousat x= 1, provideda,, = 0 bl+it-xh | 20, a. 2. 2, m4. Continuity and Differentiabilty 9.5 If £9 = fa] sin a} greatest integer function, then the points of discontinuity of fin the domain are @Z & Z\(0} © RELO (@ none of these Iffis differentiable function satisfying f(0) =O and if gQ= fo then value, that should be assigned to (0), sothat gis continuous at‘0" is @ 1 0 ©) FO @FO ‘The value of ((0)so that the function eos” (1 = {x}?) sin - {3) baby denotes fractional part of x) becomes continuous at x=0is (& |. where [J denotes the x #O( {x} x x oz oF oF (@) none of these Ifthe function f() defined as fsinsvemeyeer , exco Fea) « » eso ein nel x BP pghe + OSHS is continuous at x= 0, then @ a=gb=1— (B azi,b=e I © a=, (© none of these a Ifthe function f (= 92844 2) 2 ig continuous G24 bx) atx=0,thenthe value of 2 is 5 @ 240) © 0) © Sno (none of these Let: R’ — R satisfies the equation f(y) =e? -*-? ef) * FW) Vy RUS ()=e, then sf (x)= (ae tog |x| (Detox |x] (©) og |x| @ none of these Letf(x)= [n+p sin x},x¢ (0, 2), n¢ Zand psa prime number, where [- ] denotes the greatest integer funetion. Then, the number of points where f(x) is not differentiable, are 36 27. 28, 29, 30, 3h “SECTION w dc i Mathematics ‘or IIT-JEE @ 6 © 2-1 ‘The finetion y= fx), defined parametric as x=2t-|1-1 \andy=2" (@) cont (©) continuous for x ¢ A and differentiable for re R-(2}. (©) continuous for. € Rand differentiable fore R =t-1.2} () tone of these (b) 2-1) (@) none of these jwous and di If fx) is a continuous function for all real values of x satisfying x7 + (f(x) —2)x + 2V3-3-\3 fix) = 0, Vore R, then the value of F153 Jis @ 8 (b) 1- v3 © 11-3) (a) 203 -1) ‘The jump of the function at the pointof discontinuity ie.,x= 1 of the function 2 x foy= fig MECHAM, webb @) sint-log3 ——(b) sin +log3 (©) sin +log3—(d)_ none of these xel 7 The funetion 0) = } gsi 4 o (@) is continuous (b) tas removable discontinuity (©) tas jump discontinuity (@) kas infinite discontinuity Let R be a real valued functi I7)—0) || x—y PUeye R. Then, the finetion hes | sods is (@) continuous Vre & such that Let {f” bea continuous function on &. If then f(O)is {@) not unique &) (©) data sufficient to find f10) (6) data insuificient find f(0) t xP eas. 5#0 thenate—O,ftavis OQ 0 Hye 3. 33. 3. 35, (b) discontinuous at s = 0 only (6) discontinuous at all integral points (a) HO)=0 fy isa continuous function from R to R and (f(a ‘a forsome a = R,then the equation (x)= xhas (a) no solution (b) exactly one solution ()_ at most one solution (d) at least one solution Let be a continuous function on R suck that f(14n) 2 = (sine" Jo" + > Then, the valueof 0) 8 re 1 (b) us fay m5 (0 (d) none of these Let f be a continuous and differentiable function in (a, 6). tim, f(x) and lim f(x) 4-9. If P44 PQ) 2-1 forex, then (a) basa (b) b-a2n (c) b-a=m (dq) none of these Let f bea differentiable function such that f(x + y)= fix) +f) + Bey ~ 1 for all seal x and y. Uf 1°(0)=cos a, then xe R (a) f)=0 & seo (c) f()>0 @) fiax A function f:R — R, where R isthe set of real numbers See ae [ety )_£O)+ 709+ £10) (=) 3 for all x,y in R. Ifthe function f is differentiable at x= O,thinfs (eyo (©) cubic (b) quadratic (@) biquadratie "QUESTIONS WITH MORE THAN ONE CORRECT ANSWER (a) continucus ifp >0 (©) slifferentable ifp> 1 (©) continuous ifp > 1 (@) differentiable ifp>0 Lal fy #9 aumo, la xe Letg ()= fi), wheres (0) (a) gis differentiable but gis not continucus (b)_gisdifferentiable while fis not (©) both fand g ore differentiable (d) g isdifferentiable and g’ is continuous 10. fx /2]-[1/2] SO= “1S 2is discontinuous at (where [-] denotes greatest integer function) 1 f@) 0 ) 5 3 1 5 Let (x)= sgn (cos 2x2 sinx+ 3), where sgn ()is the signum function, thea f(a) (8) is continuous over is domain (b) has a missing point discontinsity (©) has isolated point discontinuity (@ itremovable discontinuity [of iletnem, 1 0, Isles then, (where ['] denotes greatest integer function (@ fisdifferentiable everywhere (©) Fis continuous everywhere © fisperiodic (@) fis not an odd function Iff() = | [2] x Lin -1 Sx $ 2, where [] denotes the ‘greatest integer function, then, f(x) is (@ continuous at x= 0 (©) discontinuous at (©) notdiffereniable at (@) differentiable atr=2 The function f(x) = max. {(1 - x), (1 +x), 2}, Xe (0,0), 18 () continuous at all points (b) differentiable atall points (©) differentiable at all points except at x= 1 and ot @ continuous at all points except at x= 1 and => 1 where its dacontinunos. ‘The function f (x) = (x), where (x) denotes the smallest ioernae (a) continuous at integral points (b) ‘continuous at non-integral points (©) contzoous at ihegra ponte (@) discontinuous at non-integral points ise La sey=} Tal ax + xi4) (b) f(z) isnot continuous at x =0 (©) f() iscontinuous atx = 0, . (d)_f(@) has infinite points of discontinuity Veg Laf@=e@) © Tap) where gis the derivative ate of g and is a continuous function, then lim f(x) exists os (a) g(2)isa polynomial (b)_ g@)~= (©) geo? (d)_-g(2) = A(x), where f(x) is a polynomial Ifthe function f(x), defined as a(I=xsinx) +b cosx +5 ee eee f= 3 eel is continuous at = 0, then (@) a=-1 (b) b=-4 © (a) log? <0 sx=0 Lff(x+y)=2f (a) -f(y) forall, y, where f"(0)=3 and £(4)~2, then f (4) isequal to... Let/(x) be a function satisfying the condition FC-x)=f 0, forall real x Iff"(O) exists, thenits value § 16/0) = froos4 a, then the number of points of 2 discontinuity off (x)in the interval (0, x) is. I f(a) = san (2) and g(x) = cos x ~ sinx, then the number of points of dscontimity of fog in (0,2 ae. 8. f(a) -Esinx ~cos x], where [+ ] denotes the greatest integer function, then the number of points of discontinuity of f(x) in 0, 2a are. uegey~ [>- 2] +t} E], where (J denotes the greatest integer function, then the number of points of discontinuity of f(x) in (-1, 0) are... sseo-sere( He ) forall,ye RGy A) and tin, £28 =2, then f‘(1)=... If f(x +y7)= f(a) + LOOP forall x, ye Rand f'(0)20, then (10) Continuity anc Difterentiablity 3.9 1. Let fand g be differentiable functions satisfying £(a)=2, g(a)= b and fog =I (identity function). ‘Then s()= @ = as fsinzytet , —Zez2 0 6 Letf@= + then at.x=2 Srtl © £@) is continuous ©) f(x) isnot continuous © f(a) isdifferentiable (© FG) isnotdifferentiable x82 1S be | Let fbe a function defined on an interval discontinuous at a point p € J, then we say that @ Fhasaremovabe discontinuity atpif lim f(x) exists Dut is not equal to f(p). Continuity and Diferentiabilty 3:11 (@) f has a discontinuity of the first kind at p if lim /(2) and lim , f(x) existbutare unequal. (i) has a discontinuity ofthe second kind atp ifneither 10. t cof fim f(x) and _ tim f(a) exists. apee rape ‘The function f(x) = are tan —* has @) discontinuity of the first kind atx = 5 () discontinuity of the second kind at x= 5 (© removable discontinuity atx 5 (@) continuous at x= 5. wide The fneton f(a) = + Where w= tan 3, has (@) discontinuity of the first kind at x= a+ Gane 1 (®) discontinuity ofthe second kind at x= mes F, nel (©) removable discontinuity atx= m+ ; wnet @ continuous atx= me = ne 1 The function (+) =, where e-1, x50 etl, O 0. Thus, for limitto exist the ‘numerator must also —> 0. Thus, we have (128ay!"*=2 gives a=2 Now, wehave, am (128a + ar) —2 2) oye ea Oe 2/0 10 3 Sa a 2 (as0 +2477" = lim a 178 22 thay 5 gives be ao Hence, we have, a_ 6 22S 9 5 LO) 25(a). Given that; Foy HP (CFF FON Wave R Puttingx=y= 1, weget PMH FEO => Sf) =0 = Sixt hy f(x) a0 fe Avo . x = lim a 7 AL afro) et f ime’ tt a ‘sowed et ne — te epayre irk po “i i Foo(e-Hee” my wAL a is 7 (=f)=0) (+4) ro} = fy fe) pey eo fer and fin = xO [oray=e} = i eF- feet 4 fea vale Integrating both sides wrt, 'x' we get log |x |+e= an on J(s) =e" flog |x| +e} Since f0)=0=e~0 ‘Thus, FG) =e log |x] 26(c). Hee, J(x)= [0+ p sin.x] is not differentiable at those points witeren +p sin x is integer. Asp is e prime number, = nt psinxisan integer ifsin. rip not n-sin 1 whereOsrsp-1 ? n~sin"' where 0 1, tim 8242) =" sin x 1 log(2+x) sinx = lim — T ta x ~ lim f(@) =logQ+1)=lg3and tim f(x) =-sin1 jump of discontinuity at. =| tim, f(a) im_ foo) sal a 30 (a). tim, f(x) = tim sor 0 eM lim f(a) = tim —— ror rao th] Therefore, f(x) is continuous atx=1 34 (a). Sinee|f(x)-FO)|S|x-y? xy je L0)| c)2-y1 —y Taking limites yx, we get tim | L2=L0) | « tim jx —y| Te es = [tim L=L09) cI tin (xy) por oxsy | [ye = Uf @)|s0S//'@)/=0 2 f(C)=03/(8)=6 (constant) nay = [fide = fedy = ex +d, where dis constant of integra x) of a linear function of x which is continuous for allxe R. | F@) 20} 32 (A). 1f/(a)=0, then obviously, x =< isthe solution Let/(a)> a. Since g(x) =f x) —2 therefore g(a) 0 and ef (@))=f (1a) -Fla)=a-fa)<0 Since g is continuous from R to R so at least for one c€ (af(@g(c)=0 ‘Similarly, weean argue for f(a) =) 34(b). Wehave f(x) + °(x)2-1 Saree. forxe (0,6) a) a stheponel __£e) 2» Zeer foo)en = ett 200 am() => Wa)= tare lf(x) + x, is a non-deere inthe interval (a, 8) = Lim A(x) $ lim hte) ing function => lim (an f+) fim (an! Sixy+x) => =sas—"45 greeny Hence, b—a2 35(c). Wehave, iy L(R+ A) S19) S'@)= jim. b = fin, Lode ey 2h — te = jim {20+ 201) 0 h ‘Now, substituting = y=0 in the given functional relation, we get, FO) =f(0)+F(0)+0-1=4f|0)=1 4g) = LO=FO yap $5) 20+ jim ZA a 26 +710) = #0) =2xte0sa Integrating, f(x) =22 +xe0s a+ C Here, x =Oand/(0)~ 1 = Iisa quadratic inx with discriminant D=cos'a-4<0 and coefficient of x?=1>0 f>0 TxeR 36 (a). Since f(x) is differentiable atx=0 = jim, Oe) caisay tt) @9)= lim LEt0= fla) Nowe affe)= lhn: h p(eeah ae 0 = f= fim 5 6x) = tn L804 FGM JO~ FBO) FO i TOI Is 3h = tim L2=L0 => ses lin = “0 = [from(1)] (say) +b, which is linear 2 (a,b). Continuity at LAL. im f(0-1) = lim Hy? cos =0itp>0 L HAL. im f(0+ A) = im? cos L RHE pede t 0 A =oifp=0 and, f(0)=0. ++ fle) is continuous atx=0 ifp>0 LF'0)= tim in 287A coat 8 = fim in?“ cost =itp>0, an, 8 /10)= fg LO*H=LO conto ig iB ove | ape: lim A’ cos+=0 ifp>1 nso Ce f(a) is differentiable atx=Oitp> 1. 1 3 (a, b). We have, g (x) | x For x 0, na a A(t se)? (5 1 & is not continuous at x =0 as cos — is not continuous =0. at x = 0. Also, fis not differentiable at lex, xsd 8 (a,c). fo) = 2, -l lim ar? +6 = lim — ay eoix ~ atbel “ Now, forf(x) to be differentiable at r= 1, we must have jin £92 LO _ jig, LO~LY soleorst rly xo = = fim a(x+1) = tim = a= -2 te i 3 Putting a= —+ in (1), we get b= = ting a= —> in (1), we get b= 5 41a, bya), Wehave, kim, /(0—H)= im, [-Asin (-0)] = Jim, thsin xh} = 0, Him £(0# 1) = fey (sin wh] ~ 0. and f(0) 0 FG) is continuous at x = 0. It can be easily seen that f(x) is continuous in 1, 0). f(x) is not differentiable at x = 1 but it is differentiable in (—1, 1). 12 (a, d). We have, 15° = im, - =hGeWh + 4) -1 pone” mo (7) aa You have either reached a page that is unavailable for viewing or reached your viewing limit for this book. aa You have either reached a page that is unavailable for viewing or reached your viewing limit for this book. for NT-JEE 4A -12x7 48x, -1<1<0 ee 0, O lim =0=0=0 wx Now, im (1 de)" = lim [+ ayy =i So, e/=3=9d= Hence, 02,3 —4,¢=0 and d=log,3 2(0). Since / (0) exists, + RF OHLS'0) jin, LOAN=L) ig JO-N=LO > wo ro tim LD LO oh = tim LO=LO a0 Le fC M=F001 LO=£) _ 55 ragye = 2pm AOELO -o-22/'0)=0 (rf Oexists] = F'0=0 3(0). Wehave,f(x) = froest dra" (ay=xe0st Clearly, f(a) exists andis finite inthe interval (0,2). ‘Therefore, (x) is differentiable in the interval (0, 2), Hence, f(x) is continuous in the interval (0, 1). (2). Wehave, f(x) — sgn (3) andg(x)~ 00s x ~ sinx. Then, _fog (x) = sgn (cos x~ sin x)= sgn (ve(« + 3] In the interval xe (0,2n], we have Jog 2) =-1,c0s ( = = ie, O+2], xe [-1,0) ie, Se -Isr<29 140+0=-1, x=0 Hence, fis discontinuous at 3 points, T(t). Wehave, /0)+/0)= AZ) « Putting weget /0)-0 Putting weget, f(+x)+f(a)=/(0) ss fY=S) 2) Also, A aera fe ) =2 (Given) Continuity and Ditferentiabilty 3.23 Now, F6) = jig LOWS) = im fashefen fusing 2) xth-x 7 =Gt NC = [im rt : at) [using(1)) h Ae l+e ao +h) = £0)= lim) [ J i 4 ae = £0)= jim, exh i f q ae = £0)= lim Sine ‘(ats) wax -2 e Fiete pte LO) (ms “2 Integrating both sides IG)=2tm Me +2 Sime J0)=0 = c=0 Thus, S(x)=2 tant x Hence, sss pa 8(0.0F 10). Given FY) -fO+ FOP ay and, F1@)20 -Q) Replacing x,y by 0, we get {0 =f(0)+/0 3f@=0 ~G) Ate, 10. fig LO+0= LO G “sf “ 3.24 Mathematics for IIT-JEE of, L=0,1,-Lasf(0)20 Thus, 6) = fim F* =F sxe (ey ey ef Om 6) no Foo+{ s(a? oy Fix) SF (e)= lim fusing (1)] se = f=) [asf7(0)=0, using (5)) Integrating both sides, Fo) =0 or ete Asf(0)=0,wehave f(e)=0 or x ‘Thus, f(1D) =9 or 10 11 (c) Wehave, fog=1 = (fog) (x)= xforallx = SleW=xaf'[e@-g'@=1 = s[e@)-s'@=1>f'[s@) 1 g(a) 2 bs 8'(@=2) = ror} + #(@)=9) W. (a) We have, fim, f(O-A) = Jim, 11+ | sin 19 (pra! = jim, (+ sin yom fim, [C1 sin Ainge et, ssgorne ee im, | and /(0)=b For fo be continuous at = 0, we must have _ e gabe fim FO-H)= Jim f+) =O P= 29 =5 and b= e*3, 21. (b) Since f(2) is continuous in [0, 1], therefore, 1. (d) Since fis continuous on (2, 5], therefore assumes a least once, every values between f(2) and f(5), But it is given that f(x) takes only rational values for all x and there are irrational values also between f(2) and f(5), this is possible only if f(r) has a constant rational value at all points between x = 2 and x = $. Since (4) = 8 2 fG-N=8 MI {a) Let g (x) = 2° — 3, then g (x) is an increasing function on the interval (I, 2). Since g (1) = ~ 2and.g (2) = 5, therefore between ~ 2 and 5 there are 6 points where £(8) is discontinuous (as (x? - 3] is discontinuous at the points where x? ~'3 is an integer). nr a f(z) ~ (3 * 2 e088] is discontinuous at those points where 3 + 2 cosx is an integer. WW.te) 353+ 2000x558 ore ( | Now, 3+ 2 cox = 3 ifcoss = 0, So, x= =", 4 | 29 (not possibie) 3+ 2cose=4 ifcosx= So, x has two values = and 3 3+2cosx=5 ifcosx =|. So, x= 0 0mn/(¥) Bio, a2 tan (+2) 5 | Ths, by intermediate value theorem, there is ax ga e- eras (0) such that (x)= 0. Similarly, argue for I, I and IV. LO ans wim [sse(Z+#)+2]-[on +9] £0 © £f"(0)= Jin, = fi, i : + {@)isnot differentiableatx=0 — tim [sink Also, ifx <0 or2 then] x|20 wo All|) =21x 1+ forall, eee RF) = fn, FO LO. | Gof) isnot differentisbleatx= 1/2. a | = m0) = tn =Alsine (k= 1) —UHIsinek = Jim, Bo 4 3 (a) £70) = sin i ° ivi fies yalvaticde 40 fig (DEM Dsin abo wo =h and, ENO) = ie fim (DAT =1)sin zh = lim aso 2. f(x |) is differentiable atx=0, a -1)*-(k-1) x, 4 (a). Differentiability atx (2 0) = tin, LO=M= LO 2@). Leo '(2) Lf") = jim, eh m(e YA =e) | wie + fin eam A Sas no (elit ety “Bone een) = Jim nao = tim BINA ZIM I i Since Lf'(0) #Rf "(O), -. (2) is not differentiable at nao | x-0 But since £f"(0) and Rf" (0) are finite, therefore f(x) is continuous at x= 0. Hence, f(x) Is continuous everywhere but not differen- eo ‘ tlableatx=0. 326 Mathematics for IIT-JEE 5(b, d). We have, fey = = og at= = leg a" = eee ghee af 0- i) =F 0) LPO) jim, FOP ELO «i ao a, =loga = him nook aloes soy= tim LOtM=LO BF fg SinceL f"(O=RF"(), ‘/(G)isdifferentiableatx=0, ince every differentiable function is continuous, therefore, f(@) is continuous at. 6(a,d). We have, fore>2 5 Poy = fS+I-1) de 0 1 . = forienasfis-ena o : [Sincex>2] Seren _fitaeex, ites? Je { Se+l , ifx<2 f(2 +h) -h(2) Weave, 712) fin LOA) =M2) Ow ‘ 144(24 m+ Oy = lim ——_2 = 7 2 teste? on -a— onh =f sd, 170)» im L2=B=L2) 5Q-A)-1-11 3 woe 2+ f(e)isnot differentiable atx=2 Since R/“(2) and Lf"(2) are finite, therefore f(x) is continuous at x= 2 7 (a). We have, _— (-i)-5 lim f(S—A) = Sim tan! nao ano and jim 544) Since Jim, /¢S— Aye fim, /(S +h), terefore, fx) has discontinuity of the first kind at.x=5, 8 (C). The function w = tan x is discontinuous at wn 7/2, nel 1- The function f(s) z is continuous at every u € R. Hence, (x) is continuous on xe R-jnnte ner 2 Now, we have, nes, the points x= mt © n-¢ Jhave removable discontinuity. 9 (a, b). The function GD, x20 (+), O 2°, So, f(x) =x Ix =—1, then. x= x, So,f(@) = f= 1 x. So, f(x) Ife =, then x =22. $0, f(x) = x We >1, then x <3. $0, f(a) = 23 Thus, f@) =x, xs-1 $G) =x, -1O ie, FECLOUUL) Thus, we have, [-1, xe. )UG~) fogs)=| 0, xe {10,1} 1 xe(-,-DUG,0) | which is continuous everywhere except atx € {-1,0, 1} Also, gof =f?) 1-CD, x<0 a-), x0 a-P), x0 ie, goft) which is continuous everywhere. 8 (a). We know, = fim ZEt4-/@) F'0) = fia h t(x(1+2)- rea = tim =. had h soo f{148)-s09 = fil)= lim 20 i [givenf(xy)=fee) FOO] porfiet (tes (A)f-10 aE [given f(1 +x) 1 +x (1 +200))] se {i+¥(te (%))-1} h = S's) = kim, = Fey lim a) [= im, #90] [using (1)] = 4 [06 (94)] 9 (a). Wehave,y~ #4 | |andx=2¢-|¢| Whens20 Also, when r<0 xe2etr=de and y=P-P=0 = yo D forall x<0 atx Hence, f(a) = 42% '**° whichis clearly 0. x<0 ‘continuous for all x. Continuity and Differentiabilty 3.29 QUESTIONS FROM PREVIOUS YEARS’ IIT-JEE — If@)= ; +1 then on the interval [0,7] (a) tan [f(x)] and 1/f(x) are both continuous (©) tan[/(@)] and L/f{x) are both discontinuous (©) tan [f(@)] and f(x) are both continuous © tan[/(@) is continuous but 1//(x) is not Let /: R— R be any function, Define g: R > R by 2G)=Iftx) | forall x. Then, gis (@ ontoif fis onto (0) one-one iffis one-one (© continuous fis continuous (@) differentiableif is differentiable Which of the following functions is differentiable at x=0? (@) cos(|x|)+|x| — (b) 008(|x|)-1x] © sin(|x|}+|x] — @ sin(]x|)-|x| The domain of the derivative ofthe function tan 'y, — if|xfst JO VMGat-n, tle @ 2-00) &) R-} © @ R41} Let f be twice differentiable function satisfying SM)=1I@)=4,3)=! © S@)-2vxeR) ) £'@)=5=f"G), forsome xe (1,3) (© There exists at least one x € (1, 3) such that F@)=2 (@ none of these ‘The function f(x) = [x]? [22] (where [x] athe greatest integer less than or equal tox), is discontinuous et (@) allintegers (©) all imegers except and 1 (©) allintegers except (@ allintegers except 1 The function f(x) = (22-1) |x2-3x+2 |+cos(|x Dis not differentiable at ®) -1 &) 0 © @2 Let [x] denote the greatest integer less than or equal tox. ff) = [x sin 1x}, then f(a) is (@) continuous atx=0 (©) continuous in(-1,0) 10. a. R 2B. 14, 15. (©) differentiable atx=1 (@) differentiablein (1,1) 1ef()=min. (1,2, 2}, then (@) £(@)is continuous everywhere (b) /(@)is continuous and differentiable everywhere (©) /(@)is not differentiable attwo points (@) fx) is not differentiable at one point Let /(a) = [|| “Ih, then points where fla), is not differentiable is/are @ 0,41 (&) #1 @o @i ‘The left hand derivative of (x)= [] sin (x) atx=, ‘kan integer is @ CE-De CDM De © Citer @ Cex The function pe = nl 2) bx) is not defined assigned to fat x: @ a-b © tnatind =0. The value which should be = 0 so that it iscontinuous atx=0, is (e) a+b (@) none of these Fora rel aumber ety] dence the greatest integer Jess than or equal to y. Then, the function tan[a(x-1)] FOV staP (@) discontinuous at some x (©) continuous a al x, bu the derivative (x) does tot xis foe come (© FG) exists for all, but the derivative f"(x) does not exist for somex © S@exists for alix ‘Theset ofall points where the function f(x)= is differentiable, om © .0UO«) (©) none of these TG) = x(x + Je +1), then @) Alx)iscointinuous but not differentiable at x= 0 (©) (is differentiable atx=0 (© f(2)isnot differentiable atx =0 (@) none of these is +x] © 1-) @ Ox) 3.30 Mathematics for IIT-JEE 16. 1, 18, wD. 20. The function Ix-3), eel 2 fo) | 32, BO opis 42°74 (a) continuous atx = 1 (0) differentiableatx=1 (© discontinuous at x (@) differentiableat x=3 1 The function fx)= tses( yet denotesthe 2 greatest integer function, is discontinuous at (@ allx (©) nox (b) all integer points (@)_xwhich is not an integer Let [-] denotes the greatest integer function and fe)= [tan? x), then (@) fim, f(a) does not exist (b) f(x) is continuous at x= 0 (© fla)isnot differentiable atx =0 @ £@=1 Ifx+|y |=2y, then y asa function of xis (@) defined forall real (©) continuous atx =0 (©) differentiable forall x Be pee (@ such hat = 5 forx <0 The following functions are continuous on (0, 7”) @ tanx © [rsmta a. 2, B. 24, hosxs%t © asin2x exer 4 xsinx,0< est © x x Faint. Fer Let f: R + R be a function defined by ‘fls)= max fx, x3) The set ofall points where f(s) is not differentiable is @ Chit © 1) Iffis a differentiable function satisfying 43) =0 forall n2 1,n€ Jy then @) S@)=0,x€ (0,1) (&) £(0)=0=f0) © f0)=Obutj’(O)not necessarily zero @ [f@)1s 1x 1] Let h(x)=min {x,2} forevery real numbers, then (@) hiscontinuous forall (b) Aisdifferentiable forallx © W'@)=1,forallx> 1 (@) hisnot differentiable at two values of x ‘The function f(x)=1+| sin: |is (@) continuous no where (®) continuous everywhere (©) differentiable atx=0 (@ notdifferentiable at x=0 (0) {1,0} @ 01) Led Bad 3&9 4 BO 20) 1% 1. 0 7 @ 1% 19%) 20 w 5. bad 6 @ 8. (b) 3 @ 14 @ 1% @ 16 @ 2.) 22). REVIEW OF CONCEPTS In this chapter, the concept of limit is epplied to introduce the idea of derivative which is one ofthe two central concepts in the branch of mathematics called calculus and hasa variety of applications, including curve sketching, analysis of rates, ff change and the optimization of functions. Issac Newton (1642-1727) and G.W, Leibnitz (1646-1717) are the two prominent names in the history of mathematics that share the credit forinventing calculus, Both of them independentiy invented calculus around the seventeenth century. The notion of derivative wes introduced to solve problems of physics concerning motion of an object. The velocity of an object is the measure of the rate of change of distance with respect to time. Acceleration is a measure of therate of change of velocity with respect totime. This rate of change is precisely given by the notion of derivative. The derivative is also used to find the equation of tangent to a curve ata specific point, ‘The concept of derivative has been applied to other fields as well. A biologist uses it 1o determine the rate of ‘growth of bacteria in ¢ culture, An economist uses it to solve optimization problems of a company. An electrical engineer uses it to describe the change in current in an electric circuit. Inthis chapter, we shall study in detail the meaning and the process of differentiation. We shall also discuss a number of formulae and special techniques to find the derivatives of functions. Derivative or differential coefficient of a function Lety=/lx) bea function defined on a certain interval and.x ’be< fixed interior point ofthis interval. Let dx (read: delta) ‘or ibe an increment of x such thatx + dx (orx-+A)lies within the stated interval. Let dy or f(x + A) ~ fix) be the corresponding increment of y = f(x). Then, the inerement Lieb) - SO) op foo+ = Fla) ox Differentiati 8y gy Se+8x)~ fl) 5 ox tends toa definite finite quantity as dx (or, h) tends to ze10, from either side, then this finite quantity is called the derivative or the differential coefficient of f(x) weet. ‘at the is a functionof &c(or,h). Ithisratio coorl b point x" and is denoted by f“(x) o1 gor rs a 49 = jig Let 8) =F) Th fer ia ae or im LEM LO) ah provided this limit exists. In this case, we say that /(x) is derivable wart. x at the fixed point x. The process of finding the derivative isknown, as differentiation. TREES cauTion ‘8 oe regarded as the quotient of dy by dr. 2 , isasymbol representing dal ) ‘nd is notto be IMPORTANT POINT(S) TO NOTE Derivative at a Point The value of f“(x) obtained by putting x = a, is called the derivative of f(x) at x = a and it is denoted by f(a) or ta] 42° Mathematics for IIT-JEE Standard Derivatives Algebraic Functions @ Se) =m! Sper" nyo £40 AL) gent © {2} ~ © 46-4 a = BTO- pS Wee Exponential Functions @ Ze) 4 4 yen «et 4 gop E (FFF) ©) Sea) =e'ox.0 Sait toga) 0) Logarithmic Functions @ So, 9=4 Zoe. 10) = FA ey 1 © Ste. = 545 wha F(o5, J@)= Fora” ‘Trigonometric Functions. a @s in) = cos. x i a gins) = cos! (4) Ee) a © Lleosx) é = -sin( feo Lyre Geese) = sin FO) LC @ Ltanx)= see Ltn( 700) = see (s009)£ 70 (6) Zecots) = ~coses? «9 Leor(se9) = ~cosec”, ero (e) Swen = sec x tan x 4 : 4 secre) = sof spins) L708 (0 Zeosee) = -coseex eat ¢ =" pa eel) ~eosee(f(2)) cot( f@)) FS) Inverse Circular Functions © gow) =a Fi 00) = nme S pesto = (Hay? © Fees) =~ Seow! LQ) =~ me 4 0 Ye@y d “1 1 a LZ oee"! 4) = —_L——4 pn) & VOW @P 1 {fe >t (© L¢cosec!x) = -| — z >1 & lal? 1 1 FOWL? de ge Ecosse" fix) = 4 jon: let Rules for Differentiation 1, The derivative of a constant functionis zero, ic., d x One 2 The derivative of constant times a function is constant ‘times the derivative of the function, ie, 1 @ 4 Fe fOHe FAs). | 3. The derivative of the sum or difference of two functions | is the sum or difference of their derivatives, i.e, | d d d | ZU OH OES ee). Product Rule of Differentiation The derivative of the | product of two functions | 4 Yy@-2@) =: it | FIC) 2) 0) FO} +20) FLO} = (first function) x (derivative of second function) + (second function) * (derivative of first function) Quotient Rule of Differentiation The derivative of the quotient of two functions a)“ ys) to} | de de | {g@? | (denom.x derivative of num.) | ~(auen.xderivative ofdenam) —_| Differentiation 43 Derivative of Implicit Functions ‘The derivative of an implicit function, given by the relation ‘JG, ))=0in whichy isnot expressible explicitly in termsof x, ‘an be found by the following steps Step L. Differentiate each term of the equation f(x, y) = 0 a ; 4 (ryan @s wart. x, keeping in mind that 767) = 2v5> dx 42 EO and so on. Step 2. Collect the terms containing 4 on one side and jing 2 a the terms not involving © om the other side. terms not oe Divide by coefficient of © to get 2 asa fune- Step 3. a a tion of x or yor both, ‘Shorter Method for Finding the Derivative of an Implicit Function ‘© Take ll the terms of the function to be differentiated to the left hand side and put left hand side equal to or derivate of O(a) ware. y treating x as constant * ie of (xy) weit xtroating y as constant 6049). ge & - (denominator)? | Derivative of a Function of a Function (Chain Rute) If | ‘ys a differentiable function of f and r is a differentiable ‘function of vie.,y=f() and 1=g (x), then edt de dt ae Similarly, if y=/ Qo, where a= g (v) and v= h (, then, el te ae du dy de Ify=w", where wis a function of, then BBM yet MD apy l ae ds ae @ Ld Derivative of Parametric Functions Sometimes, x and y are separately given as functions of a single variable 1 (called a parameter) ie., x = f (2) and y= a (0. In this case, & dla £0 ae alt g(t)” fy .4 (2) ma, ra lee a ie: a at Differentiation of a Function with Respect to Another Function | Ify=f(a)andz=g (x), thenin order to find the derivative of SGe)wst.g (3), weuse the formula & _dlde | fs) | de delde g(x) Logarithmic Differentiation | If differentiation of an expression or an equation is done after taking log on both sides, then itis called logarithmic lifferentiation. This method is usefil for the function hav- | ing following forms @ y= OR = A). he 0 aie. anGe).. ERO * @ » (where = 1,2,3, f(a) andg, (x) both ae differentiable, @Casek: y~[/G)]H, where f(s) and g(x) are functions of +x Tofind te derivative of this type of functions we proceed as follows: Let y = [,/(x)]*?. Taking logarithm of both the sides, we have logy ~ g(x).log f(4) and then we differenti- ate wnt. x. aa You have either reached a page that is unavailable for viewing or reached your viewing limit for this book. Differentiation 4.5 se tan? 2 gop? Successive Differentiation 2 4, cos m= 2 — ih — Lety=/la) bea function ofx, then is again a function of 1+tan?™ cor? 1 ar 2 2 x and is called the first derivative of y w.r:t. x. If the first ba & +] _ Ltanx derivative is differentiable, its derivative is called second mgt) qoans 2 ae ds tive ofthe origin function nds denoted by 2" or §.am fe -) ~Trtanx vy. Ifthe second derivativeis difereatiable, its derivative is called the thd derivative of the original function and is, 7. tao te + tary tan ( éy . Imxy dencted by Tory, andso on. This proces of iferen- tiating a function more than once is called successive differentiation. Differentiation of a function given in the form of a determinant 8 tone + taney = w+ tan! provided s,y> O and ay> 1 . =r), 9. tan” le —tan’ =2itay>0 if) ex) Ae) neat Hy=| p(x) a(x) r()| then os ea ei aay) wx) WOH) i ot = seo"! + cose y= sin"! (xf y? ya? ), 4 £1) x) HC)| 1 Pod g(x) Me sin tea si provided x, 20 andx?+y?<1 u(x) v(x) w(x) 12, sin-let sin-ly= e- sin! (xJ1-y? + yV1-x?), FO) a(x) AG)| [F) 8) A) ifx.y20and2+y?>1 +lp@® gs) P@|+]pQ) ae) ra) xa) v@) WO) [v@) ve) we) 1B, cos'x + cos"'y= cos"! (yr vi=w? Y=). 2 me * Note that differentiation of a determinant can be done in ifs,y>Oand?+?<1 prensa as 14, cos tet cos ty = 2 cos (xy FVI-x? I=»), ifx,y>Oandx2+2>1 | TRICK(S) FOR PROBLEM SOLVING 16 Jt ( }=2tarts If ais a repeated root of the equation f(x) = 0, repeated 4x? times, then f(x) can be written as . FG) = Gay g(0) ~) ue: where g(a) #0. Clearly, from (1) 1. Ifyand2 arethe functions ofx and ify? +z*= 2?,then 2 (@ none of these ra) (5) is equal to Bay el As 2. IFS, denotes thesum of n terms of G.P. whose com- zd ade a, @ <$ &® ZS son ratio is then (r— 1) Sis equal to aa You have either reached a page that is unavailable for viewing or reached your viewing limit for this book. 21. 2. 4 Bh G A=|4, B Glis A Bs OG @o (b) 24 ow oa = Ds itfe)- SS Ge isequal to pd 3 © -taF © oa © oy (@) none of these 7. IEF) is a polynomial of degree n (> 2) and f(x) = F(k—2), where kis a fixedreal number),then degree of fais @ x a Qn (none sie wf te LB ff, Hore @ 2 ) ie © cea @ none of these The solution set of f"(x) > g(x) where f(x) | Tiidst nd gGjed arise | (a) (1-9) b) 0.1) | © O-) @ [-) | ). If for all x, y the function f is defined by fix) +f) | +f@)-f0) = 1 and fix)> 0, then (@) F%(@) does not exist (b) f(x) ~ 0 for all x ©) SO equals @ 3 ) 3 @2 @ © Letfex)= [] (cos 2k tye + isin (2k —1)2) then fel (Re foxy" + Km f(x))”isequal to @ rf ) = nf) © -7fe) © “fe 23, 25, 26. 27. | 28. 29. 30. Diferentstion 47 Tf 2f(sin x) +, =. then 4 f(2) i (sin x)+flcos x) then (2) is @ sinx +cosx (2 © 4 (@) none of these Letf(ay= fem + fe +2410 e151 2s @-! m1 @o (@) does not exist The derivative of the function represented parame- tricallyasx=2t—| 1), y=P +7 | 1/at1=0is @ 0 1 @- @ docs not exist ‘A polynomial /(x) eaves remainder 15 when divided ‘by (x—3) and (21+ 1) when divided by (x~1)?. When ‘fisdividedby (c—3) (x1), the remainder is (@) 22+ 2x43 () 22-2r-3 (e) 22-2843 (@) none of these If for a non-zero x, the function f(x) satisfies the equation 0-0 }=4-8 (a+b), then J"(x) is equal to (@ 45 (s+) © = a(S s+) alt) (@) none of these (©) ’ Ix = cos? and y = sine then 2% = e 10s 10s (oP singo (©) “sin29 105 (0) P cesdo (4) none of these orae(e (Ea (@) OVrER () OV xe @,~) (@) OVxe R- {0} (A) none of these 48 au. 32. 33. M. 35. 37, 38. 39, Mathematics for IIT-JEE (a) odd (b) even (©) periodic (d)_none of these Ups) =(1-xFtetevaeot f0270)+ 24 e LOi (aon (b) 0 @r (d) 2"-1 ad ws en x= ea ye in 2 aec= ea fa) 0 1 A (d) none of these (a) none of these Lat F(x) be a polynomial function of second degree. IfQ)=f-Hand ay, a, a, are in A. P, then f(a,), Flay S(es) are in @) AP © HP (b) GR (G)_ none of these 1 & 45/6) +3ff + |-x+2andy—xf(sythen © axa 1 Hsfey (2) fesyinon Fat is equal to @t (b) 1 1 1 O5 @ Let f(x) be a polynomial function of degree 2 and fi)“ forall.xe Rif ge)=f6)=/(0+/"Co). then lor ‘any x @ ax)>0 (b) e<0 (©) ae) =0 (@) go) s0 Let fix +) =F) */Q) + 2-1 Vg ye RI Js) i differentiable and f"(0) = sin, then (@) fO)>OVxER (b) [O)0,forallxe R (@) none of these TEfG—y),f@)- SW) and fe + y) arein A. P, forallx, y and (0) 0 then @ FAHIC ©) fB)+/C3)=0 © FAKE @ £B)=fC3. A function f: (0, +2) + R satisfies the equation ° then H(@) ~— soo-rre0-s(3) ff is differentiable on R and f(1)=0,f(1)=1, then 1 @ ros] © se=* (©) f(@)=Inx @ fare Iff:R— Rbea function such that (x)= +27"(1) + af’ Q)+f'"G) is differentiable for every € R, then TEED Eg Gant € 0a) bral gor) andf!(3)=4.Then, [/(10)}?-[ (10) =... Let (x)= (+ 2). Iff” (2) is ¢ polynomial of degree 20, where f” (x) denotes the mth derivative of f(x) wert. x, then the value of nis % 12 Differentiation 49 @ s0=5 ) © £°@)=6 (d)_ none of these MY&—+/@+Y)=2/O FO WH,y€ Ry then @) fiseven (b) Fisodd (©) f'iseven (@ f*isodd Infiax)=3? +3g(I) +e" Q)and g(x) =f(1) “3? +f (0) +f"(x),then @ s@=r-3 (©) gx)=3r+2 (©) feyav 3x (d) e(x)=-3r+2 (1+ at" +80"), then =x {b) © a=-(+l) Iy=s@)=min ¢9;-3 Sesxwhere gx)=l|=1 |x + Uj then (a) f()is non-differentiable atx: (b)_/(a)is non-differentiable atx: (©) £7Uo)=0 0 @ Jf@a-s 3 x sinx —=cosx 1ef(x)~| nt sin(om!2) cos{nn/2)] then f(0) for @ a a n=2m-+ Tis @1 fb) -1 ©o (@) independent of Letf(e) =a) + tx? 335-33 forx> O and g be its inverse, then the value of & such that ke’(2) = 1 is equal to @ 36 ©nR &) 31 (@) % IEF(x)= fle) gC) and f(x) 2") o 3. Ifx= secO—cos@ andy =sec"@ cos", then (¢) 2 =k (4} here ia Sa. 410 Mathematics for IT-JEE ar 4. fp? a? cos? + B sin? then p+ 22 2, dept where k= . af in(efx?) Incex*) 8. If'p(x) be apolynomial of degree 4, with p(2)——1, p"(2) S Wena ( nety | [inetaety | P'Q)=2.9"2)=~12and p"2)=24, thenp"(1)= dy . 9. Letafunction g satisfies the equation g(x +») = e'gix) + egy) forall veal x and y. I'g(S)~ 32 and (0) ~ 2, 6 zea yl ey" then G2 ae =, then the value of g'(5)-2e"is . a wg where k= 10, tfy= +. then Ys 08 (log) (2) a ty flex HP E)= f= Omd y= f[ Tb 1. The derivative of (tan x)w.rl.g(seex)atx= *, 4 where (I) 2and g’ (J) =4.is oe I. Lety=x?—8x+7andx=f().1f < =2andx=3 y fo. a dx ati=0,then “7 ati=Ois MIL, Let f(x) = sin x, g(x) = 2¥ and Ae) = cos. 1 6) ~ fg0 (7) (9, thon *(f J sequtte ™ Uy)~ oo s+ 4% sin n(x) ai (3) = 3ythen (g 0 f) tr, is equal to LXaDOG weH L The function y defined by the equation satisfies J” +£yp"=0. The value of kis IL. Ifthe function v(x) represented by. ye oo Fru re ‘equation (1 —32)y”— 29’ = dy, then kis equal to UM, Let x)= (a) g(x) M(x) for all eal x, where f(3), _g(*) and A(x) are differentiable functions. At some point sy. ifF (xq) =21 Flxg).f"Cx)) =4 f (i) 8'€%Q)=— Taig) and M9) = Khe) then & is equal to IV. Let f(x) =x", n being a non-negative integer, The number of values of for which the equality Sat b)=f (ea) +f'(b) is valid for alla, b> 0, is 2) satisfies the ‘The matching grid: Let be a function such tha ( uno (-$.4), Let fstisy the equation IE*LOI= LY 1-9? + ya?) 1. Thefuntion (is (a) even (b) add (©) constant (d)_ none of these 2 Iff(x) isdifferentiable on (— 1, 1) and /“(0) = 1, then J°@) is equal to t 6 Ones () ae @ - Te? tee © 3. The function f(x) is equal to (a) cos! x (¢) tantx (©) sin x PASSAGE Ii Let f: R—> R be a function satisfying the condition (242) £04 £0) k k + where k-# 0, 2. Differentiation 4.14 @ 4 (b) 2 / PASSAGE IIL Be oa® ORG) ‘The function f(x) is differentiable on R and f“(0) =m. F'() is equal to (a) m (©) mt (b) 2m id) 0 The function f(x) is equal is (a) me (b) m+] (c) =2mx (d) none of these A function f: R -> [1, +») satisfies the equation f(y’) SOLO)-F@)-10) +2 6 ‘The function fis differentiable on Rand f (2) ~ 5. FC) is equal to L9-1 pqy py @) fo x (0) LE 0) 6 ome st Fox) isequalto @) 2-1 (by 1a? () P41 (d) none of these 442 Mathomatics for IIT-JEE PASSAGE IV A function prof 5) sins the equation xty my ‘The function fis differentiable on R and /’(0)=2. 8 The function fis (@) aneven function — (b)_ an odd function (c) a constant function (J) none of these LO*I0) 9% FGHis: A to ® os © © @ Ite 10. f(is equal (@) tar! x (©) 2tan! x (©) 4tants (4) none of these ice PASSAGE V | Let 2 =/(¢, ») be a funetion of two variables « and y ‘The partial derivative wr. of the function 2=f(s,3) at (x,y) is defined as Ler 86 = JOY) a8 As provided the limit exists and is finite ial Ta & Cleariy, 5 is the derivative of 2 = f(z, ») wort. x, regarding y as a constant Simi, we can define 2 a a (a vz a (a (=) devoted by SF OF far ale he denoted a a (a 22 SE Se 3(E)} denoted by —— Ho bs ty 2 ali} denoted by = (oF fy ate called second order patil derivatives of = (4, au Seant2 then x M4 a y x ay a ite 1 (4)_none of these 2 $y +2, then or er ir, where k= FP ar @ mt) >) (©) mim 1) (@)_ none of these 13, If @= 7" €°" then the values of » for which If Uand V are two functions of x having derivatives of the ith order, then (UV), = UV "CU, Vy H" CU Vy = CU, VY, oeur 14, wpe Poazmes ahs ava (e+e +), where k 2 2 (a) nx (b) ne (o) — ar (d) -2nr 15. If cos! (3) = we( = J sthens’y,,.9# 20+ yo +hy, = where f @ 9° (b) 20? ©- (a) -20 16. Iff(x)=tany, then LSMODV= "CaP? (0) HCL 10) = (@) sin (6) cos a (©) wan (@) none of these 17, 1rd, = 5-68" tog 9) then f= fy ky where k= n> a 1 fa) nt (©) (n-2)! () (m1)! (a) none of these ifferentiation 4.13 SECTION Vi ASSERTION REASONING TYPE QUESTIONS Instructions: Jn the following questions an Assertion (A) is eet" given followed bya Reason (R). Mark your responses from Reason the following options. T= (a) Assertion(A)is True and Reason(R) is True; Reason(R) 3. Assertion + Iff(x)= (cos.x+ i sin x) (cos 2x + sin isa corect explanation for Assertion(A) 2 (€08 35+ isin 3) (Cosme + sin (©) Assertion(A) is True, Resson(R) is Truc; Reason(R) rns) and f(1) = then f%(1) is equal to is not a correct explanation for Asseriion(A) nla +) (6) Asserton(A) is True, Reason(R) is False -( : (@) Asserton(A) is False, Reason(R) is True den Wiab atthe, canted, 1. Assertion + Let f(x) be polynomial function z z satisfying 4. Assertion + Ifthe parametric equation ofa curve is s001(2) = ro+s{2) 1tf(4)= piven by x = 080 +g tan & and 65 and J, ly J,are in GP, then U,), 7 dy Jaf Uy ate also in GP. y= then he pints for which > Reson 3 feared =0 ore given by O= ne, ne Z 2 Assertion + Hy=U +90 +9042)... #y _ sind Reason: SE QUESTIONS WITH ONE CORRECT ANSWER © 3. o @ 7. (@) 2 0 2) 2 @ f @ BO 4@ %@ 160 1 bb) (b) 19. © (b) 21. (a) 22. (b) 23. 24, (a) 8) ® 2726 ® 2a 0% Be 3 oO i) oO LO 3a 8 wo 4. © 43. tb) 45. 46. (b) ‘SECTION II QUESTIONS WITH MORE THAN ONE CORRECT ANSWER - 1 be) 2b) 3 @b 4 @d 5 bd & &bd 7% @d 8 lad) 9 (be) 10. (aed) 11. Gd) 12 fab.) 13. abo) | SECTION Ill ; _-FILLIN THE BLANKS 1 9) 2. (70), 32 @ 4. 3) 5. (0) 6. (25) 7 (12) B, (26) 9. (32) 10. (0) "SECTION IV _ COLUMN MATCHING 1 16 @ le ©,I > 0), Ve @ 2. 16 (d), Her (b), Ill (a), IV (b) aa You have either reached a page that is unavailable for viewing or reached your viewing limit for this book. aa You have either reached a page that is unavailable for viewing or reached your viewing limit for this book. aa You have either reached a page that is unavailable for viewing or reached your viewing limit for this book. From equations (2)and(3), we have ie, © Solving equations (4, (5) and (6), we get 28 (a). We have 1) arnt) tos a) Substituting 2 in place of x, we have ar(2}+yeras-s 2) Eliminating (3) from equations (1) and (2), we have x wrawse=d ts) 10-5 1 [f-oe+ 56-0] [4 ind = f= f£@= 29 (a). We have, x=cos’@ ard y Differentiating w.rt. @, we get % 7 c0s°@ sind a 4 and, % a cos6 a6 Thus, we have, a S 7 cos*6sind a ax ( )- A (dea ay yay} doh Jay (5 cos*8 sin?0— 70386) cos@ =35 cos*Osin?@—7cos°O =35 c0s30(1 —cos’@)—7 cos'6 =35 cos?@~42cos°@ - 4 {as \ 0 18\ ay?) dy (105 cos" sind +210 cos*8sn 6) 8 105 sin@ cos® (2.cos*6- 1) 1D sin 20¢0520- 3 sis Differentiation 4.17 30 (b). Wehave, However, the above function is defined only for values of, given by xl -Is=s1 xtl x-1, x ie, SFP +1 20nd 150 Bey a ie yar 20 md 55, 20 ie, x <-Lor2 Oandx>-1 ie x20 Hence, we have, @. and, ao 31 (a). Wehave, aol = 2 “Tae (ey nate which is an odd function, since L)=S@) 32 (b). Wehave, f@)y-(=x)" £@)=-n0-n"! L)= Cl n=) =x)? Pe) =n! Af OV=N (N= Dyn aa You have either reached a page that is unavailable for viewing or reached your viewing limit for this book. aa You have either reached a page that is unavailable for viewing or reached your viewing limit for this book. aa You have either reached a page that is unavailable for viewing or reached your viewing limit for this book. (a,c). Since f(xy), fl) -fW) and fe + »)arein AP. = fer VAfe-9=4O)SO) A) Puttingx=D, =O in (1), we get LO+IFO-YO-SO=fO=1_—— t+ AOV#0} Putting x=0, y=x in (1), we get SOIEN=2AO) SOALOSAMEAL) Differeniatiig/(2)=/(-2) wet.x, wehave LO+SER=0 £ LD+*SCD=OLA+SLCH=0 5 (a,b,c). We have, Fox) =2370) (5) “ Puting x= inequation (1), we et 70) -70)- 5 = fo ~a(¥]rn=ae%en @ A ete Sox On integrating, we get S()= inx +c, where ¢ isa constant Since, f(1)=0=> c= 0, Thus, we have f(s) ~Inx 6 (a,b, c). We have, FO) =P HPP O+S") (1) Differentiation 4.21 Differentiating w.rt.x, we have S)=324 2G D+/Q) Q) 3) Differentiating w.rt.x, we get "a =6 (4) Patting x=3 isequstion (4), we get f"G! Puiting x= 1 in equation (2) and in equation (3), we get LW) =342F4£'Q) ie, FO+/"Q)=-3 3) and, SQD=124+2F0) (6) Solving equation (5) and equation (6), we have ‘f()=Band/"@)=2 7 (a, d). Wehave, LO W*FO+) =2FOYO) A) Putting y= 0 in equation (1) we have FO FO=1; = FO) =1 Putting x= 0 in equation (1), we have SOLO =YOHFW)=20) = FON =T0) +2) = _fiseven Differentiating equstion(2) w.rt.y, we get SISHY = fisodd. 8 (a,d). Letg/(1)~aandg'2)-5 1) Then, fo)=etarth, f)=1+a+b Pw)=2tasf"Q)=2 eG) =(I+a+ by + Qe tayx+2 =VQ+atb)+art2 > PO)=2B+atb)ta Hence, ¢(I)=28 +a +5) +a 2) ¥Q)=43+0+b)+a ~Q) From (1). 2)ard (3), wehave, a=23+a+b)+a and b=23+a+d) ie, 34a+b=0 and b+20+6=0 Hence, and a=-3 So, fQx)=a?—3x and g(x)=-ae+2 (b,c). LetS=1txtst teat et which isa geometric progression 1d=x"") Satertstiet yee MER ‘On disferentiating both sides, we get, 041+ 2e4 3x7 44 + ett dayton ba -(I- a) - (=? aa You have either reached a page that is unavailable for viewing or reached your viewing limit for this book. aa You have either reached a page that is unavailable for viewing or reached your viewing limit for this book. aa You have either reached a page that is unavailable for viewing or reached your viewing limit for this book. Differentiation 4.25 4 & 42 = oay'-y'tP=0 ie, Gy-Dy'tyP=0 Hee) Neve een g a8 Differentiating again with respect to x, we get It is given that when ¢= 0,x=3 Oey Grey Dyrew 2 When t= 0,2 =3.3-8=19 = FY" ty) =" + Byy'= 0 4 ola _ Also, a ela AD) (b) i ‘ fe ee Dis wi v from (1), 19 Ea a6 and, i Vi (a+ dye! : 12 (b) Wehave, (£4) (2)=/(2) : h (x) =sin x cos x & 2 = oe ws veo 2 * Leo¢ FiO) G) =2 (CFA) = (FG): he) i oe =a (sinxcosx)=2sinxeosx=sin2k ty (2) = ie. in 2x tt ~ Differentiating w.rt. x, we get = (@) = 2 cos 2x and 6" x) =—4sin 2x E a fOr ee) (= Py aye cas olay “ or(2)=- asd —uy = (1-2) y" say" = 2y [dividing by 2y'] % k=2 IVa) f'(@) =—2c0s x sinx2.c0s (+4) sin(r+2) |W. (a) We have, ’ p FG) =f) g) h@) + sobesin( +2) + sin cos (+3) = log F(a) = log f(0) * log £9) + log A @) Differentiating both the sides with respect to x, we get a bx ~ sin (2x4 22) + sin(x20) Fi) _ £2), 8) HO) Fo) f0)” gi)” We) Fea). FG) , 840) , HC) =~2sin (2=+4) cos = + sin (2x03) FG) Fle) * eo)” Mx) sin (2003) +in(2x02) <0 = “ned THke k= => (2) = consiant for all x. W.(b) f= asm! But, (0) = cos? 0 + cos? 3 + sin 0+ sin 5 = LAN“ OTO 5 Ff nah BYE = nahh nit! Sa)= 3 forall, eee ie ier el ae eeney-strorns(3) : ype epee 21. (d) Differentiating the given equation with respect to | So, f“(x)=0;f'(a+5)=0 x weget sosato=s(a+s'o) wty i=jyso Hence, there are two values of m. LSS bes forsea= f @i=F-2f1-#) =O) SO)+F0) =F eVi=y¥ +» t-x#) Putting x =0 and y= 0 in equation (1), we get Putting y =~. in equation (1), wehave f0)*f0) =f10) aa You have either reached a page that is unavailable for viewing or reached your viewing limit for this book. aa You have either reached a page that is unavailable for viewing or reached your viewing limit for this book. 45 (b). Step 1: We have cos ‘(rn dee—toon) > y= 0s [n(logx- log n)] Al) Differentiating, we get y= ~b sin [n(logx—logn)] = Squsring both sides, we get y,2 = 020? sin? (log.x—log m] = 128? [1 cos? {n(log x—log n)}) Using (1), we obtain ype nti aah? Differentiating again, we get Dey? + 2xpyy=—2wyy, | Cancelling out 2y,,weget?y,+xy,+nly=0 .Q) | Step 2: Differentiating (2)m times by the theorem, we get Deva Crags 2x4" Cy, 21 +B HC yy tay, = 0 OF, 7 y_42 + ANE Ing MAD Ig FAY ps1 + HYq AMY =O OF, 7 Yyag Ont W)XY gg F2M7 Y= Oc R= Del 16 (a). Apply the theorem to f(x) cosx=sin x. 17 (b). Weave, Zi (eem) fre osx ta} mt ae mt mt on fete Sy Tank t@-I)! k=@-0! 4 (a), Since f(x) is a polynomial function satisfying | s0y-5(2)=r00+4(4), 2 fG)= +1 of fa) =-x+1 if SG) =~x" + 1, then f(4) =~ 4" +14 65 foy= 241 SQ=65 441-65 90=3 JQ=P +1 ase) =3e LG) = BRL) = 35S G) = 35 Since fy ly, fy are in GLP, S'U)L'Q)L"() ate also in GP. 2 (a). We have, yr (ta (+x (ta) 1x2") = CEC + x) x) x4) CL t22") mx _ dex? ory +(x?) G92? @. a (ax? 4). a7 > | 34). a)= (cosx + sin) (cos 2+ sins) (cos3x + sin 3x)... (cos m+ isin nx) cos (x+2r+ 3x +... +m) tisin@e+2c+3x+...+m) aint (+1) mos OE), 4) cos xt isin x nine) ans) (n+) sf AED ee] wre (SE om (zee = sw nin), antes wate) 2 ) Sa) n(ntl) oft - (220 --( nine) ve) . QUESTIONS FROM PREVIOUS YEARS’ lIT-JEE 1. There exists # function f(s) satisfying f(0 FO=-1,F(@)> Oforall x and © F(W)- Let Sx and 8y denote the increments in x and y, respect- ven - £62)- Sea) beh Thebsensn ns £2 ‘0 is called the average rate of change w.r.t. x in the interval Un) Awe contin ctoosng te vlues of in auch way that the interval [x,, x3] shrinks to zero, i.e., & > 0 then the dfn of derivative accordi tin 2 This is known a the rate of change of y wit. x forthe samevalue x=; 4 Here, 2 represents the actual rate of increase iny per unit increase in forte particular vale ofx or 2s the rate at which y is changing with respect 10x. Oud Again, oa ode = dt de dt In other words, ifthe rate of change of variables.x andy are taken relative to time f, we have Rate of change of a 2x rate of change of. Sp fale of change of x IMPORTANT POINT(S) TO NOTE The differential coofficient ofy wnt. vie, © isnothing but the Tangents and Normal Geometrical Meaning of Derivative at a Point The derivative of a function f(x) at a point x = a is the slope of the tangent to the curve y = f(x) at the point [a, f(a)} Slope of Tangent Consider a curve y= f(x) and & point P(x, ») on this curve If tangent to the curve at PC, y) makes an angle 8 with the positive direction of.x-ans, 52 Matematics for IIT-JEE then, at the point P(x, y): 2 = tan @ = m = gradient ot slope of tangent to the curve at Plx, ¥). Equation of Tangent The equation of a tangent to a curve y= f (x) at a given point P (c,, »,) is given by a Imn “(Ze [Using point siope form of equation of the straight line] Slope of Normal The normal to a curve at a point Plxy, y4) is a line perpendicular to the tangent at P and passing through P. Slope of the normel a Slope of tangent 3) & Joes.) Equation of Norm: The equation of normal to a curve y = f (x) at a given point P(zy, ») is given by @-x) Ly 1) Fig. 52 [r+ the normal to the curve at the point P(x, ¥,) is @ line perpendicular to the tangent at the point P(x,, y,) and passing through it Therefore, slope of normal at P(x, 94) = =1 +r © 5, tmgest mates ano ange th the r-axis. & + 1f S <0, the tangent makes an obtuse angle vith the x-axis, a |e If S =0, the tangent is parallel to x-axis. «If the tangent is perpendicular to x-axis, then I De aig de ay « If the tangent is equally inclined to the axes, then a & = tan 45° or tan 135° = + 1. > ortan ‘© The slope of. line having equation ax + by + c= 0 is given by a _ _coeificient of x 5 coefficient of y” ‘© The two lines having slopes m, and my are () perependicular if mymy = —1 and parallel ifm, = my ‘Thus, if m be the slope of a line, then the slope of 1 | alline perpendicular to it is =~ and parallel to it Angle of Intersection of Two Curves Let y= f(x) and y = g (x) be two curves intersecting at a point P(x;, y,). Then, the angle of intersection of these ‘twocurves is defined as the angle between the tangents to the two curves at their point of intersection. Fig. 54 If 6 is the required angle of intersection, then, 8 = (8,8), where @, and 8, are the inclinations of tangents io the curves y= f(x) and y = g(x), respectively, at the point P. ices ‘TRICK(S) FOR PROBLEM SOLVING Method to Find the Angle of Intersection © Find f(x) and g'(x). © ef") g"() =— 1, then the two curves are said to cut each other orthogonally, wherever they cut. © If the product is not ~ 1, solve the equation of the two curves to get their point of intersection. If (a, B) be their point of intersection, then find f"(@) and g' (a). Let m, = f(t) and m, = g" (a). ‘© If Gis the angle between the tangents, then tangs MI 24 S(@)-e'(@) Temm, 1+ J"(@)a'(@) Repeat this process for other points of intersection IMPORTANT POINT(S) TO NOTE The two curves are said to touch each other at their point of intersection (a, B), if the slope of their tangents at (cz, B) are equal KEY POINT(S) TO REMEMBER fone angle between the tangents (acute/obtuse) is 8, then the other angle between the tangents (obtuse/ cute) is (180° - 6). Generally, we take the acute angle to be the sngle of intersection of the given curves, Length of Tangent, Length of Normal, Sub-tangent and Sub-normal Let the tangent and normal at the point PQ y) om the curve meet the axis of x at the points 7 and N, respectively. Let M be the foot ofthe or- dinate at P. Then, Fig. 55 () Length of the tangent = PT'= | ycosee 8 i |»vireor"6| = |S de (i) Length of the normal = PN-=|,y sec 6] = jira =| fH (8] 7 ‘Applications of Derivatives 5.3 y (2) & (2) Length of the Intercepts Made by the Tangent on the Axes Equation of tangent at any point (x,, y,) to the curve veseviey-n=(Z) @-a : ry | Gi Sub-tangent = 7M =| y cot 6] = | | Ge) Submormal = Mav = |y ten 8) = | | ci a (oe ‘Then, x-intercept = x, @ 4 Kan) ; - 4 and intercept = 9, ~ »(2) | ( \@ fan Length of Perpendicular from the Origin to the Tangent Lenght of perpendicular from origin (0, 0)to the tangent drawn at point P(x, 9) t0 the curve y = f(x) is nah) Increasing and Decreasing Functions (Monotonicity) Increasing Function A function f is said to be an increasing function on an interval J, if 1 <9 F 0H) SSG) YH % EL Fig. 56 Increasing Function Strictly Increasing Function A function fis said to be a strictly increasing function on an interval J, if

FO) va EE trey t {fey a A & x Fig, £8 Siricthy Decreasing Funetion Monotonic Function A function / is said to be monotonic on an intervel / if it is either increasing or decreasing on 1. Test for Monotonicity of Functions @ f(s) is increasing in [a,b] iff "(a) 20, 7x€ [o, B) iy f (2) is strictly increasing in [a,b] iff” (x) > 0, xe [a, 5]. Explanation LADS, , 7 fle +H)> fla)if fis strictly increasing] Fo = tim, Conversely, Letx,, By LMY theorem, there exists a point ¢ © (81.4) ‘such that FU)“ M8)= FEN) -%))>0 [= £'9)> Oandxy~x,>0) 2 € [a Bhx exp = f)>fe) => f(2)isstricily increasing in (a,b). (ii) f(x) is decreasing in [a, Jif") $0, ve [a, 5} (v) F€2) is strictly decreasing in [a, 6] iff" (x) <0, ‘wre {a,b} a t TRICK(S) FOR PROBLEM SOLVING © Ifa function f is strictly increasing (strictly decreasing) on an interval J, then f~" exists and is also strictly increasing (strictly decreasing). * Iffis monotonic on an interval /, then fas at the most one zero in the interval 2. # If the functions f and g, both are increasing or decreasing on an interval /, then the composite funetion gof (2) is an inereasing function on # If the function fis increasing (decreasing) and g decreasing (increasing) on an interval £, then the composite function gof is decreasing on the interval [. * A function may be increasing in some interval /, and decreasing in some other interval, Comparison of Functions using Calculus To compare two functions (x) and g (x), we define a function (x) = f(a) ~ a(x) oF F(x) = g(x) —f(@). Then, ‘we check whether F(x) is increasing oF decreasing in the given domain of /(4) and g(x). Maxima and Minima of Functions Local Maximum 4 fune~ tion y = (4) is said to have a local maximum value ata point x= a, iff(x) €/(a), wre (a-ha +h), for small A> 0, i, fa) isthe ‘greatest ofall the values of Fle) inthe interval (a—ha gh hi). The point © of the function f(x}. Local maximum rea) fla—h) ath Fig. 89 is called a point of local maximum aha Local Minimum A function y= f(x) is said to have & local minimum value at a point = a, if f(x) 2 fla), ¥ x € (ah, a* h), for small h > 0, i. f(a) isthe smallest ofall the values of (2) in the imerval (a —h, a * A). Fig. 5.10 ‘The point s = «is called a point of local mit the function f(x). KEY POINT(S) TO REMEMBER ‘© The points at which a function attains either the Jocal maximum value or local minimum value are called the extreme points and both local maximum and local minimum values are called the extreme values of the function /, ‘© The local maximum and local minimum values are also known as relative matimum and relative ‘minimum values, respectively. Method to Determine the Points of Local Maxima and Local Minima Method | (First Derivative Test) 1. For the function y = f(x), find f" (x). 2, Put "(x)= 0 and solve this equation for-x. Let its roots be a, b, e, ett. These points are called sta- tionary points o critical points, 3, Atx =, determine the sign off (x) for values of x slightly less than @ and that for values of x slightly greater than a. () ff" (®) changes sign from positive to negative as x increases through a, then x ~ a is a point of maximum. yp local maximum a fish \— ah Fig. S11 (ii) ff" @) changes sign from negative to positive as x increases through a, then x = a is a point of minimum, y (Gil) 7) does not change sign as x increases through a, then x = a is neither point of maximum nor a point of minimum, Such a point is called a point of inflexion, We repeat this process for other values of x and examine them for maxima or minima. Applications of Derivatives 55 SE canon BR ‘+ A function may have maxims or minima at a point without being derivable at that point # If f(@) does not exist, then there is no question of extremum atx =a. Method Il (Second Derivative Test) 1. For the funetion y = f(s), find f*(x) and f"(x). 2. Put f’(x) = 0 and solve this equation for x. Let its roots be a, 5, , ete 3 Atx=a @ iff" (@) <0, then x ~a isa point of local maxima; (i. iff" (@)> 0, then x =a isa point of local minima; ii) iff" (@ = 0, we cannot say any thing. Greatest and Least Values of a Function in a Closed Interval (Absolute Maximum and Absolute Minimum) If f(2) is continuous in an interval [4, b], then greatest or ebsolute maximum value of f(x) = max. { f(a), f(b), Values of f(x) at all critical points in (a, 5)}. Also, least or absolute minimum value of f(x) = min. {F(@), #(6), values of f(x) at all critical points in (a, 6)} IMPORTANT POINT(S) TO NOTE Ifa function is de (orl ned and continuous on an interval seal alue other than local maximum or local Concavity and Convexity of a Function Concavity of Function 1f/%(x) > 0 in the interval (a, 6), ‘then shape of (x) in interval (, &) is concave when observed from upwards (.., concave upwards) or convex down, Geometrically, a curve is concave upward in the interval [a, d] iF all points on the curve lie above the tangent to the curve at any point in the interval [a, 6]. If the curve is a concave upward, then the slope of the tangent increases as x increases, i.c.,/"(x) is strictly increasing in [a, 6) = f"0)>0vxe [a,b]. Convexity of Function (a, 6), then shape of f(x) i observed from upwards (i. down Geometrically, a curve is concave downward in the interval {a 5) if all points on the curve lie below the tangent to the curve at any point in the interval (a, B]. If the curve is concave downward, then the slope of the tangent decreases as x increases ice. ‘£'(2) is stricly decreasing in (a, 6] = £'@) <0vxe [a, bl. It 7%) <0 in the interval ferval (a,b) is convex when . convex upwards) or concave 56 Mathematics for IIT-JEE y Concave up Convex Down FeasnsBe Fe 513 Point of Inflexion Ifatx= a, the shape of the curve changes from concave to convex or from convex to concave, then x =a is known as, the point of inflexion. ”(x) changes sign as xpasses through the point a, Method to Evaluate Point of Inflexion Points of inflexion can be obtained by equating /"(x)~ 0. It is not necessary that all values of x which are obtained by equating ”(x) = 0 are points ofinflexion. Only those values of x for which F(x) changes sign are points of inflexion, Higher Order Derivative Test to Determine Local Maxima, Local Minima and Point of Inflexion Let f(x) be a differentiable function in an interval J and let x = a be a point lying in the interior of 7 such that @r@-fa-ro Sf” Ya) = 0 and (&) "2) exists and is non-zero, then: # If mis even and f%(a) <0, then x =a is a point of local maximum. ‘If mis even and f"a) > 0, then x = a is a point of local minimum. # If n is odd and f"(a) > 0, then x = a is a point of inflexion where shape of curve changes from convex upwards to concave upwards. # If n is odd and f"(a) <0, then x = a is a point of inflexion, where shape of curve changes from concave upwards to convex upwards. RR cuTON * This testis used only when second derivative fails to decide between local maximum and local minimum ive, when at x = a, /"(a) = O and /(a) ~ 0. © This test is not applicable to those critical points where f(x) remains undefined. ‘¢ Maxima and minima occur alternately, that is between ‘two maxima there is one minimum and vice-versa. # If/(@) > a8 x a or b and f'(«) =0 only for one vahie of x (eay 0) between a and B, then f(c) is necessarily the minimum and the least value. # If/(x) > — asx > aor b, then f(c) is necessarily the maximum and the greatest value, ‘+ If a function is strictly increasing on (a, 6), then F(@) is local minimum and f(b) is local maxirmum, « If a function is strictly decreasing on [a, 5], then £(@) is local maximum and f(6) is local minimum, Rolle’s and Lagrange’s Mean Value Theorem Rolle’s Theorem If a function f, defined on the closed interval (a, 5], is (@ continuous on [2, 5], (i) derivable on (a, 6) and (ii) £ (a) ~ F @ then there exists at least one real number c between a and b (a {b) (abscissa) (©) (ordinate) > (€)_ none of these Letf: 2, 7] > [0, »»)be acontinucus and differentiable function. Then, pa agteyp 10)-f0) LOO LOO) @ SOFC © SOLO wheree © (2,7) isequalto 3 () So) (@) none of these 1 Itp(e) = f ef" (SxS), the maximum value a of fis @ Je-1 (b) %e-1) (@ e-1 @ 2We-0 Giveng(x) 22 and the line 3x + 10 =D, then. the line is (@ tangent to gx) (© chordof gi) (b)_normalto gtx) (d)_ none of these ‘The sub-tangent, ordinate and subnormal to the parabola y? = dav ata point (different from the origin) arein @ oP (AP. © HR, (@) none of these If: R— Ris continuous and differentiable function stich that [todas @ [a= fie ds [ie 3 +P anpiede ‘Then, value of “(A)is (@) 8-8"()+/"2)@) 48-8F(0)-F°2) (©) 48-8()+/72)@) none of these The fiaction exceeding its pth power by the greatest number possible, where p 22, is rhe 1 wf oft (@) none of these Let the function f(x) be defined as tan“! g-3x? ,0e and g(x)=x°+ br+cbe two given functions such that f(s) and g(x) attain their maximum and minimum values respectively for same value of x then the value of & is :-3.-5, @) 1 (b) 2 3 @ Ifthe equation a,x" +4, 0"! +. ,x =O has a positive root x ~ «, then the equation aye + (nV ay yet 2+ positive root, which is, (a), smaller than & (b) greater than @ (©) equal to @ +0, = 0 has a (@) greater than or equal to Ia, 6, ¢ be non-zero real numbers such that Jorscos! tar? eon ela = J(0+ cos! x)(ax? + br +e)dr =0, then the equation ax? + dx + ¢=0 will have (2) one root between 0 and | and other root between Vand2 (b) both the mots between 0 and 1 (6) both the roots between | and 2 (@) none of these Let f be a function which is continuous and differentiable for all real x. 1ff(2) =~ 4and f(a) 2 6 forall xe (2.4). then fa) fA)<8 & faz 12 (b) fee (4) none of these i (b) ab (@) none of these 23. 4. 25. 26. 27. 28. 29, 30. 3. Lei/bea continuous, diferentiableandbijective func- tion, Ithe tangent toy = fix) atx =a isalso the normal toy = f(x) at x= 6, then there exists at least one ce (a, b) such that @ so=0 © fe<0 The values of & for which the funetion ((0)= ke? 9x2 + 9x-+3 may be increasing on Rare (@) k3 () kS3 © #3 (@) one of these ‘The least possible value of k for which the function (x)= + kx + | may be increasing on[1, ZJis @ 2 ) -2 © 0 (@) one of these Let f(x) be a quadratic expression which is positive forall rsa x gs) =fls) -f"(x) +f), then for any realx (@) ga)>0 (©) g@)20 ) fE)>0 (a) none of these (b) g(x)s0 (@) gx)<0 <2 and g(x) bea monotonically inereas- ing function of s. 4 g(a)de+f g(a) dr a (@) increases with increase in (b~) (©) decreases with increase in (ba) (©) increases with decrease in (b~ a) (@) none of these The equation x ~ e*=0 has @) only one real root ()_ only two real roots (© norealroot ——_(d)_none of these The value of @in order that F()= sinx~cosx—ax+b decreases for all eal values is gi @ a2Vi © a21 187" (@) b>0, ‘when (@ a=b ® (© a=36 @ A function fis such that (@) £2" (a)=Oand f has a eeal ax itf@is (a). apn"? (©) b= (ay? (b) 6-1-@+1-ayrt () (apr? p24 nd g=x-4.re (0.x) thea minimum value of PIO? @ si ) is-2vF (©) does not exist (@) none of these Ife) and g(x) are differentiable functions for 0 1 such that (0)=2,¢(0)=0./(1)=6,g(1)=2 then in the ineerval (0,1), (a) "(x)= 0 forall (0) f'(2)= 2g'(2) for atleast onex (0) f'@)=2g'(a) for at most one x (@) none of these "Neharacters of information are held on magnetic tape, in batches of x characters each: the batch processing time is @+ Br? seconds; a, B are constants. The ‘optimum value of «for fast processing is bw) 2 A Oo © & @ £ ‘A finetion fis such that f’ (4) =" (4) = 0 and f'has ‘minimum value 10atx=4, Then (x) (a) 44+(e-4)" () 10+@—4* (x4) (@) none of these @ = 510 Mathematics for IT-JEE 40. at. a a 47. The range of values of k for which the function F(x) = E-Tk+ 12) cos x+2 (k-4)x + log 2 ‘does not possess critical points, is @ (5) (©) (1,5) © 1.4 (@ none oF these If a differentiable function f(x) has a relative minimum at x = 0, then the function y = f(x) + ax +b hes a relative minimum atx=0 for @ alla>o (©) aloo (©) allaandd (@) allsife-0 Let /(@) and g(x) be defined and differentiable for 42%, and fig) = a(%)o 18) > ga) forx> zo, then @ fO)% —(b)_fl)=B(2),x> 5% fe) £(2)> 8@),x>%9 (d)_none of these If and B (a< B) be two different real roots of the equation ar! + bx ¢~ 0, then b 6 @ a>-> ©) B<-3y @ a Zsa If pls) = ay + aye + ax? +... + 0,0" and [p(n € je*!-1) forall x20, then |a, +2a,+3a,+..+na, | @ sI b) 21 © 20 @ so ‘The maximum value ofradius vector where ©) a<-2

Dis 4 @ @ro)? wo +, (a+ by e © a @) Hats) =a? x Letyco= { +108: o OK 0Vxe Rand gx) =f(2-2) +/+). ‘Then, g(x) is increasing in @) C~,-1 (b) (-2°,0) © Che) (@) none of these The curves x? - 4y? + ¢ = 0 and »? = 4x will cut orthogonally for (@) ce (0,16) © ceG.ay ‘Which of the following is not true? @) ce-3,4) (@) none of these “The function f()= x? + 4 hase minimum atx=2if = 16 maximum atx=2ifA= 16 ‘maximum for noreal value of 2 point of inflexion atx=1ifA=—1 If the parabola y = f(x), having axis parallel to the ‘y-axis, touches he line y= zat (1,1), then. @ FO+HO=1 & 2%O+/O=1 © 2f@-FM=1 @ 2f'O-fO)=1 ‘5.12 Mathematics for IIT-JEE B. 74. ‘The angle between the tangents at any point P and the line joining P tothe origin O, where P isa point on the curve fa (x?-+ 3) = o tan”! cis a constant (b) varies as tan“y (4) none of these (@)_ varies as tan'x (© isa constant Ifthe equation ar? + Bx +e=0 hastwo distinct positive ros, then the equation a? + (b+ 6a}x+(¢+30) =O has (@) two positive roots (6) exactly one positive root (©) atleast one positive roct (© no positive root 78. 16. If /(2) is continuous in (a, 6] and differentiable in (a, b)then there exists at least one c ¢ (a,b) such that LO) KO squats Daa - fie @ 0) © 32 ©) fle) fe) (d) none of these 23 Let) =inxandgto)=?. oe (4, 8).then oo( S| ‘equals (@) clnS-8 (b) 2c n4-8) © Ac n5-8) (@) cina—8 1 We) =3 (E}v6 = 27), Ve (-3,4), where f° (x)> 0, 0 xe (3,4), then h (x) is. : oan 2.4) a2 0 sonar ( 3.0) (©) decreasing in (- (@) decreasing in (03) (@)_ maximumoccurs at (t) minimum occurs at (©) maximum occurs at (6) minimum occurs atx= nz, neven Ite a)" (x — BP" |, where m and mare positive integers and a > 6, isthe derivative ofsfunetion then (8) x=agivesneitheramaximum nora minimum (b) x=agivesamaximim ivesa minimam (@) x= bgivesneither amaximum nora minimum Uf (a)=|x]-+ [x1] +)x—2 then (0). fix) has minima at..=1 () fix) has maxima at=0 (©) flx)has neithermaximanor minima at (@) fix) has neithermaximanor minima at 5 The function Se)= fle = DO 1U=2) = 3)Sdr bas alocal minimum atx= @o 1 @2 @3 (+P st xP, where @ p> (b) OSps1 (©) x20 (@ x<0 Teg ()= fl) +f ~x) and f"(x)<0 for 0S.x5 1, then, (@). g (2)increases in (-- 4) siozonwseast (0 1 (©) g(x)decreases in ( 1) @) g(x) decreases in ( >) The function f¢ (a) increases in (—2, 0) (1,2) (b) increases in (0, 1) (2,29) {c) decreases in (0, 1) 42,02) (d)_decreasesin (~ 2,2) U(1,2) Leth (x) =f(0)-[/@) + L/O0) forevery realnumber x. Then, {a)_ iris increasing whenever fis increasing, (b) iis increasing whenever fis decreasing (©) iris decreasing whenever fis decreasing (d) nothing can be said in general aa You have either reached a page that is unavailable for viewing or reached your viewing limit for this book. aa You have either reached a page that is unavailable for viewing or reached your viewing limit for this book. 1, Theangle between the tangents to the curve = 1 atthe points (a, 0) and (0, bis © x oF oF © 5 (@ none of these 2. The angle of intersection ofthe curves y= 2 sin?x and y=cos xatx= 2 is © x @ | ©F @ § (© none of these 3. The parabolas y? = 4ax and x? - 4ty interseet orthogonally atthe point P(x,,y,), where, y,0, then @ @® bea © Bad (@ none of these 4, The acute angle between the curvesy=|x*— 1 | and 3223] al sein tus ofioksiacanbiwheds Ow 2N2 a2 8) Sg Op ) Re (@)_ none of these i PASSAGE II Iff(x) be a function of x, where f(x) is continuous in the closed interval (a 6] and differentiable inthe open interval (@.b). Also, f(@) = f(b), ie. the values at the end points a and 8 are equal. Then, there exists at least one point c between and b (i.e., a) exactly one root (©) at most one root (4). no root PASSAGE Ill Iftwo functions fand g defined on (a, 8] are ( continuous on the closed interval (a, 6] (i), derivable on the open interval (a, 6) (ii) g(x) # 0 for anyxe (a, 5) then hereexistsat eastone eal number ce (@, 8) such that £@) fio) 2(b)- gla) gc) We may write itas £0)- $0) oF a@)=aa 2 FO Hence, there is an ordinate x= c between x= a andx=b such that the tangents at the points, where x = e cuts the L£0)- f(a) ss othe! 6) a apis of the Ranctions £0) aa so) — (a) mutually parallel 9, The value ofc for the functions f(x) = [x and g(x) g(x), are in the interval [a,b] is © fo (@) none of these 10. =F(@),where0 x for 0 cos x> ssin (cos x) sin x0 aa You have either reached a page that is unavailable for viewing or reached your viewing limit for this book. 5.48 Mathematics for IT—JEE 14 (a), Let y = xx”, where x is the fraction & “4 a grlewe ao For maxirm imum, = ‘or maximum or mF we» = ep! =0-9x=(2) . P ; ayo 1 gemedinowaen’ LO, . 15 (d). We have, (6x, 0 1 709 CTS FW) =-61-K<0 and, f"(1+h)=-6<0. Since f(x) does not change sign as x passes through 1, therefore, f(x) does not have a maximum or minimum at x = 1, whatever be the value of a. 16 (a). We have, [nx,-25x<0 s@e4. x20 x, O FY) = + e084 (+ by+ 6) ll) 1 Now, f(1) = [(1+00s" x)(ar? +bx-40)dr = 0 a 4 and, f(2)= [(1+c0s! x)(a? thr te)d = 0 ‘ Also, (0) = 0 £O)-S0)-F2). Now, by Rolle's theorem for f(x) in [0, 1], F1(G) = 0, for atieast one a, 0< a<1 and by Rolle’s theorem for f(x) in (1, 2], £'(B) = 0, for atleast one 8, 1 < 8 <2 From (I), f"(a)=0=(1 + costa) (aa?+ba+c)=0 But 1+ costa #0, ao? + ba+c=0, ie, @ is a root of the equation ax? + bx +c =0. Similarly, f"(B) -0 = af? + of +e~0, Le, Bis 2 roct of the equation ax? + be +0= 0, But the equation ax? + br +,c = 0, being a quadratic equation, cannot heve more than two roots. s The equation ax? + br +c = 0 has one root @ between 0 and 1 and other root between 1 and 2. 24 (b). By Mean Value theorem, there exists a real number | © © (2, 4) such that f(a) 2 Since f’(3) 26vx € [2,4] f+4 2 2 fO26— Peas ese ie =sy28 F'Q)=0 Sa- b But ax +> 2; « f(x) 20 for all. x>0 . sfey"]=»=0)" sols)" sexo sib 203 abe 23 (a). Since the same line is tangent at one point X= and normal at other point x = > = Tangentat x= b will be perpendicular to tangent atx=a = Slope of tangent changes from positive to nega- tive or negative to pasitive. Therefore, it takes the value zero somewhere. Thus, there exists a point ce (a, 6) where /'(c) = 0 24 (a). We have, fix) = ke’ - Sx? + 9x43 = f1Q)= 3h2= 18x +9 Since f(x) is increasing on R, therefore, f'Q)>OwrER 3k? — 184+ 9>0vxER ke 6x+3>0vxER A> 0 and 36 — 12k <0 [wv attbete>OwreR = a>OondD<0] = k>3 Hence, f(x) is increasing on 2, if > 3. 25 (b). We have, f(x) =x? + ke + 3 fae +k, Also, f" (x) =2 Now") =2,7€ [1,2] > f()>Ovxe [1,2 = f'(a)isan increasing function inthe interval {1,2 = (0) is the least value of f"(x) on [1, 2] But /(3)>Dwre (1,2) [+> £0) is increasing on (1, 21] F'0)2 0 wre [21 ke -2 Thus, the least value of & is ~ 2 ug ‘Applications of Derivatives 5.19 27 (a). We have, a+b=4 = 6=4-aandb-a=4 =2a=t (sy) 4 4 4 +o Now.s(a)= JaCokte+ fatsdde = facades f eteide ‘ > a a Hla) de 7 884-0) As @<2 and g() is increasing 4-a> a ga) e(4—a <0 vom, Ha). Wa) dt, da) _, HO, 5 "da “ dt dt “da dt Thus, la) is an increasing funciton of 1. Hence, the given expression increases with increase in (b — a’. 28 (a). Let f(a) — a te 0, Since f(- co) = — oo and f(+ F(x) = 0 has a real root, Let the real root be a Then f(a) = 0. Now f'@)=1+e>OvxeR J (x) is an inereesing function v x € R. for any other real number B, S(P)> S(@ 08 f(B) 6 c isa (ss a sin? 8 + b cos? 9 = (a ~b) sin? @ + ) Given: a= 2Yab. a= 4b aa You have either reached a page that is unavailable for viewing or reached your viewing limit for this book. aa You have either reached a page that is unavailable for viewing or reached your viewing limit for this book. aa You have either reached a page that is unavailable for viewing or reached your viewing limit for this book. 56 (c). Wehave, x(e+2) >0VxE(-09,-2) <0Wxe(-2-1), fo ERD sow ne 1.0) = f0)=dmasiy= S43 re4d Since, Rolle’s theorem is applicable = 10) =f Wada F+heced => lat3b+6e=0 Hence, required condition is 2a++ 33 + 6e=0 58 (a). Leu fay= 0-300 = Nox, ‘The roots would be real and distinct if, SOFEV <0 = (@-2)@42)<0 of > ~2? |sin£]=0 2 ma = » = a=land a2 = a= landja—2|22 = alanda-2220ra-2<-2 > anda24oras0 => a= landae (-=,0)U4,-9) Therefore, ae (-=,0]U(1} Ul4.e9) 63 (b). We have, F00)= J [ow, (log; (logy (cost + a)))| dr a Differentiating w.rt.x, we get FC) =|log, (log, (log, (cosx+a)))|, ‘which is clearly increasing forall R But it nustbe defined, ie, log,log, (cosx +a) 20,0 re R = log, (cosx+a)21,VxeR > cosx+a24,VxeR = a24—cosr = ars Thus, /(x) is increasing for all real values of *x” when azs 64 (a). We have, Sbs)~ (e? AY" (2 =x + Ly assures local miinimaat.x~ 2 = IQ)0, Since f(2)=0 = FO-h>0 and, FQN 0,9 H>0 = EMG - hy -3h+ 1170 and, WA hy UP + Sh+ 1) >0 ie, Chy >0. [: (4-H) 0,2? 3h+ 1>0, 4+h> 0,8? +5h+1 > 0V A> 0) <= mis an even number 65 (c). Sincethe function f(x) increases for all x, therefore, | faa fo) ( Sa pessoavsen bal and, orf b+1 ‘The above inequality holds, when (1) b+ 1 <0 and (it) 21-4b—-P>0 z b<-Land b+ 4b-21<0 = b<-Land(6+7)(6-3)<0 = b<-Land-7<6<3 be-7,-1) ‘Again, when b+ | >0, (x) will be increasing for all, if, 21-45—# > Oand 1> bet or, BP 4b-21 <0 and, (6+ 1 > Q1 46-6") [as b+ 1> 0] or (*7)(6~3) 0 aa You have either reached a page that is unavailable for viewing or reached your viewing limit for this book. aa You have either reached a page that is unavailable for viewing or reached your viewing limit for this book. 7 (b, ¢). We have, f"(x) <0, for 0Sx1 = —f’(x)is a decreasing function in (0, 1). Now g'()=f"(a)-f"(1 -¥). (2) is increasing if g(x) > 0 = f@rs-9 = x Berd eel Clearly, f(x) > 0 forx <0 or 2 Thus, f(x) is inereasing for ( @, 0) U (1, 2) and decreasing for (0, 1) U (2, =) 9 (a, c). We have, WG) =f'@) 0-270) +3 (FO) =3/'@[ seo? -270)+4] “ sres{(10-4) | - Note that h'(x) <0 whenever f"(x) <0 and h(x) >0 whenever f’(x) > 0. Thus, f (x) increases (decreases) whenever f(x) Increases (decreases) 10 (a, b). Leth) =f) - 2) Since, f'(x)> g'(x) ¥ x € R, therefore, HQ =f'@)-g'@)> Ove R. => Gis an increasing function v xe R But h (0) = f(0) - g (0) =0, So, for x > 0, we must have i (x) > h(0) = 0 and for x <0, we have A(x) 8@rvxe @~) and, SW) Oand g(x) g(x I) and fog (2)> fog + 1) (0) gof+1) SO >Se— land gafa) x= 0 isa solution and then y= @y Also, SE =2 + e% + e*>0, hence is minimum, a 2 The shortest distance OP = OPI? =1, 3. (6) Let a, A, 7 be the roots of the given equation, Ther, a+ B+ 7= a, af + ay+ By= 4, oBy=8 Since AM. 2 G.M.=9 ser > YoBy = $2Waaz6 The mi um value of a = 6, 6. (1) We have, f(2) = cosx + cos (/2x) = [f0)|=|cosx+cos/2x| $|cosx| +|cosv2x| oP EP eR Maximum value of f(x) = 2. This requires cos x = 1 and cosy2x = 1 = x= 2nzand y2x=2mz, nme 1. 2m i ana = 2 y= BL, OD we This is possible only when n = m = 0. -+ Theres ony one valu (x = 0) st which 2) sans its maximum valve. 7, (12) Since f(x) is decreasing in the interval (~ 2, -1), therefore, £7 Q) $0 9 6x? + 18r FASO, The value of A should he such that the equstion 6x7 + 18x + A= 0 has roots - 2 and - 1. Therefore. -2)(-N=4 saan 6 8. (5) Let the sadius of the sector of the flower bed be rand the angle subtended be @ S = Surface Area of Sector 9 20-3 = S* 358 __ Length of Area Als Oe Radius > a Y 20-2» oP = S=(10-rr Now. s =(0-")+r-1) For surface area to be maximum as Bnomr=s 10. (3) The function f(«) is continuous and differentiable on R. Suppose, that (x) vanishes at four points (more than three points), so that the equation Six) =0 has four different roots, say a, B, yand 6. Then, ‘We have, So) =f(B)=fP=/(6)=0 ‘Thus, according fo Relle’s theorem L@)=0 forsome x in [of B },[B, 1, and [% 5] ‘which implies that fx) will vanish fora least three values of x, According to the given condition /"(x) vanishes for two values of x only. Hence, by contradiction, f(x) cannot vanish at more than. tee points, 11. (6) We have, = SPL OO ZU LO} aa You have either reached a page that is unavailable for viewing or reached your viewing limit for this book. aa You have either reached a page that is unavailable for viewing or reached your viewing limit for this book. 531 A (a). fx) =e = fal = £0) £9) ~ g 10 (b). Let fq) = sin x and g(x) = cos x ®-s@) Then, (9) = cos « and @'(s) ~~ sin x 2 f pus Clearly. f and g are continuous on (a, f. fand g are derivable on (4 B)g’(x)=—sin.x# 0 for any xe («, B) ‘By Cauchy's Mean Value theorem on [@, f}, we have, £B)-f@) _ £@) e@=aa@)~ 8@ @ PF = sinB-sina _ _cos6 cos=cosa ~ = sind a= ne Lac o cos x. x é Also, since 0 cos.x > sin (cos.x) if 0 < x 5 (a). Let f(a) = rtans > F's)~ tone 1 +8 sec > 0, forse (04). But tanx > x and cos x > 0, for 0 f(a) > Bun p> aun ais i, x40 3 (c). Let f(x) = { eco 6 (2). Since ¢ (is decrensing, sro 205) < g(x) when x, > x, Then, f'() = ASSES Since /(x) is inreasing, = Sose(=tamx) 5 (6. 2) “fle @)l2fle,)) x mee oor => h(x) 2 A(x) when x, 2 x). (stane>rand emx>Owhend M8 desestg fiction of and 4 (0) =0, 2 aa You have either reached a page that is unavailable for viewing or reached your viewing limit for this book. aa You have either reached a page that is unavailable for viewing or reached your viewing limit for this book. aa You have either reached a page that is unavailable for viewing or reached your viewing limit for this book. 29. 3. (b) (2) hasa local maxima (©) FG) is stricily decreasing function (@ £() is bounded 2. ‘The function f(x) = sin* x + cos* x increases, if @ 0 @) 420 3 @ 6d) 7% bo 8 © 2d 1% Mod 20 B2@ %WO +66 16 © 72 1% @ 1% @ MG 2.@ %@@@ 2 (ab 2 () 25. @) 26.) a7. @) 28, 2, 0). aa You have either reached a page that is unavailable for viewing or reached your viewing limit for this book. aa You have either reached a page that is unavailable for viewing or reached your viewing limit for this book. 6.2 Mathematics for IIT-JEE 4 TEA GL OL Ol xy then JUiG)#2 014 60)-Jae =] Aj) dt] flx)det] f(x) dee. 5. tf f(a) de = Fos), then Jy (axed) de umber) are functions of| Flat b) Standard formulae of integration ‘The following resultsare a direct consequence of the defini tion of an integral. not 1, fate =e Gna. 1 2 fae =log|x|+C ea 4 fat de= + 5, fsinede =~ cos x+C ‘cosxdy = sins + C sec? x dy =tanx+C cot C 6 1 & fcosec?x de 9, J seoxtans de =seex+C 10, feosee x cot xd =—cosee x + C i funde log | cos.x| + C=log| sec x |+C 12, feot xde =log| sine |+C 13, sce xd =log| seex*tans| 4.0 14, Jeosee x ae = log] coser xcot x | +C 15. sint e+ C3|x]<1 16. rix+C (+x?) a 7. msec! | x|+C3/x[>1 wae 1 19. 20. IMPORTANT POINT(S) TO NOTE semmulae, if is ve same Formulae is applicable x vide by coelMicient ofr nr derisative of u genestlat f flrnds ~ aa) * Cathen Methods of integration Method of Transformation When the integrand is a trigonometric function, we transform the given function into standard integrals or their algebraic sum by using trigono- metric formulae ‘SOME USEFUL TRIGONOMETRIC IDENTITIES] 0s 2m 2 gg _ Lt e0s2me + eosin - SS . 22 in SM gg BE sin me=2 sin cinfme = 25iR-mt = sin 3 moe sin? mx i 5, Seosmx +c0s3 mt © conn = SSCS ooo » tan? see?me— 1 © cof?mx = cosec’mx ~ 1 + 2¢08 4 cos B=c0s (4 +B) +005 (A -B) # 2sin Acos B=sin (4 +B) +sin (4~B) + 2sin Asin B = cos (A B)—c0s (4 +B) Method of Substitution By suitable substitution, the variable xin J /(x) dis changed into another varieble 1s0 that the integrand f(x) is changed into F ()) which is some standard integral or algebraic sum of standard integrals, ‘There isno general rule for finding a proper substitution and the best uide inthis matte is experience. However, the following suggestions will prove useful. 1s Ifthe integrand is of the form f(x + b), then we put ac b= rand dc = oh a Thus, f f"(ac+8) dr -frot -1froa 10, Max+b) 2. When the integrand is ofthe formx"='f"(x"), we put rand nx"! de= at, Thus, (v") ae=f 7 we Ley aelr (t)de =tpo=1 pe, ie 3. When the integrand is of the form [/(x)] "./"(). ‘we put A(@)=tandf"(x) de= dt, tt nel G@y*! nti Thus, fU/G" sede = fe" a ~ 4. When the integandis ofthe form £2), we put F(t) fls)=tand f(x) ded. £2) F(a) Thus, a= ee =log t= log f(x). Panos « Jtan x dv =log |sec x|=-log| cos x|+C # Jootx de= log |sinx|+C # Jsecxdr=log|secx + tan.x|+C= log] tan | = + |cosee dx=-log| cosec x + cotx|+C= : Explanation 3 fran xtc J $2 a log| cosx|=log| seex|+C Sotade= f 25% de = f £82 vigginsisc sesbeertimg) (seox + tanx) © Sseoxde= = [ieee ‘Gecr#tany ~08|8e0x+tanx|+C Indefinte Integration 6.3 Now, 2 tan I+ 2. 5 wente z fang Vesely eee seex+tan x= —- 7 osx 2e x mtn? 1 tan 2 2 1+ tan? = see xde=log|secx+tanx|+C nx tan( += G4] + Foosex dra J SOseexlensers toot) 4, cose x teat = log +C d (cosec x +cots) easeex+ cotx == log lcosee x+ cot x|+C Now, 2cos? = covee a +eats= LEE 2 = op 2sn 58> Jeosee x de =o onsets + C= gan +c. Method of Integration by Parts. The process of integration of the product of two functions is known as integration by parts. Forexample, if u and v are two functions of, then du Jeep edemn Joa (Hote) as In words, integral of the product oftwo functions = frst. function * integral of the second — integral of (differential of first integral of the second function). ‘© Choose the first and second function in such a way that the derivative ofthe firs function andthe integra of the second function can be easily found. © Incase of integrals ofthe form f f(x).x" de take 2” ‘as the first function and f(x) as the second function, aa You have either reached a page that is unavailable for viewing or reached your viewing limit for this book. Some Special Integrals * a logk +{x?-a]+c toghe + VF Fe] +c a A. Let/= lai Put x=atanO=3 = tan@ = @=tan12 a 3 de=aseods asec?@ « te “laren d0= +a? tan? 2? (1+ wan) @rae—a) 1 p(xta)~(e-a) J G+ax=a) Nef dares zel(za- sti) Indefinite Integration dk (a+x)(a-x) = paraiso, De (at aa-a) 65 sind = @= sin“! a = de= a00s8d0 acosd 1 - [Ss a6 iF fa? -a? sin? =f 20058 4g arcs) ‘ac0s0 =faemercesinlZac Purx=atang=~ =tan@ = 6= tan! a a = de=asec’Ode asec?@ 1=[—S* _ae Setanta “f ases?@ ya? (an?@+1) = [sec 640 =log | sec 6+ tan 6|+C" tog no + Jit [+c z x fe z+ es aye xtye tx? a, = logix + ya? +x? |-loga+c’ = logix + fa? +x? |+C = log +c = log| +c aa You have either reached a page that is unavailable for viewing or reached your viewing limit for this book. ‘Substitution x=asin Borx=acos @ x=atan Bors acot@ x= asec Borx=acosee @ th ar 08 28 Integrals of the Form {sinx cos"x di © Ifthe power of sin x is an odd positive integer, put 0s x: (@ Ifthe power of cos x is an odd positive integer, put sin.x=2, i) Ifthe power ofsin x and cos x are both odd positive integers, put sinx=2 orcos r= 2 (jv) Ifthe power ofsin xand.cos x are both even positive integers, use De" Moivre’s theorem as follows: Let cos x+ sin Then cos.x—/sinx Adding these, we get ‘Now, expand each of the factors on the R.H.S. using Binomial theorem. Then group the terms equidistant from the beginning and the end, Thus, express all such pairs as the sines or cosines of multiple angles, Further integrate term by term, “er Alternative Method: Simplify the integral asthe sum of sines and cosines of multiple angles, using Boos x+ cas 3x a cos’x Indefinite Integration 6.7 (W) If the sum of powers of sin x and cos x is an even negative integer, put tanx = 2 and de= Simplify the integrand using sin x = Integrals of the Form [tan"x sec”x dx and ‘Joot™x cosec’x oh. Working Rule () Ifthe power of see x is an even positive integer, put tanx=1, (W) Ifthe power of see x is an odd positive integer, then ook for the power of tan x,i.e.,m * [fm isan odd positive integer, put secx =r + Ifmis0, use integration by parts © Ifm isan even positive integer, write sec*x — 1 in place of tan*r; the integrand reduces to a polynomial in secx; use integration by parts. Similarly, J cot"xcosec"x dx can betackled, Integrals of the Form the givenintgrand becomes faneton of tn = Z- (i) Integrate the resulting rational algebraic function of 2 Loox (@ Putten > =2=9 5 sec? 5 de = de (v)_ In the answer, putz = wn = ) pi 3 Integrals of the Form de & a » [—— © Nacty? — Saeeats © § & z cos? x +bsinx cosx-+esin® x 68 Mathematics for IIT-JEE Working Rule (@ Divide the numerator and denaminator by cos?x. (ii) Inthe denominator, replacesec*s, iffany, by 1+ (ii) Putian x=2=5 see*x a= de, (iv) Integrate the resulting rational algebraic function of = (iv) In the answer, put = tan x. Integrals of the Form poses balns a ceosx + dsinx Working Rule (Put numerator = A (denominator) +4 (derivative of denominator) acosx + bsinx= A(ccosx+dsin.x) + esin+deosn). (i Equate coefficients of sin x and cos x on both sides and find the values of & and 1 Split thegiven integral into two integrals and evalu- ate each integral separately, acosx+ bsinx ecosx + dsinx xesinx + dees: waftdeen J Toserbsine = Ax=tlog| acosx+b sin | (jv) Substitute the values of A and yt found in step 2 Integrals of the Form fees. e+ feosx + gsinx Working Rule () Putnumerator= (denominator) “+m (Cerivative of denominator) + a+ beosx+esinx=l(e+ feosx+ gsinx) tm (fein e+e cosx)+n (i) Equate coefficients of sn x, cos x and constant term ‘on both sides and find the values off, ,n. (Gi) Split the given integral into three integrals and evaluate each integral separately, jatboosasesing 4, e+ foosx+ gsinx ilies as al yma mle mlog| a4 feou s+ guinsl +nf——* __ oa femsx tga (iv) Substitute the values of /, m, n found in Step (ii). Integrals of the Form i cots -tsins C08 A -? Also, 2sin xeosx —(sinx—cos x? Pat sinx+cosx =1 => (cosx~sinx) dr= dt Also, 2sinxcosx =(sinx+cosx)’—1=P=1 1 BS free Bc iB layal = 1 jog! sin + 0083 V3 Yi lsinztcosre | ° Integrals of the Form de Slee 8 tae © Jar +bx+ede ‘Working Rule (@) Make the coefficient of x7 unity by taking the coeffi- cient of x outside the quadratic. }) Complete the square in the terms involving xj write ar? + bx + cin the form a [(x+ a}? + PI. (i) The integrand is converted to one ofthe nine special gral. (iv) Integrate the function. Indefnite Integration 6.9 Integrals of the Form © fot a, wy fetta ©) f (prt) ax bev ae free TS Jims) Pulpx+ 9 = Qax +, P=2ahandg = 2+ Bp ars P 5 =F log) ats 2a 8! pexg ee ee fx? + tx Slex+a) lar tine de Integrals of the Form ml) ae ar? +bxte degree n. ire Fx) is a polynomial in x of Working Rule Write Pa) qo teste ars Vere 1estareae +» yy Ate constants to be determined by sifferentiating the above relation and equating the i Powers of x on both sides Comparing the coefficient of x and constant term on Doth sides, w Then, the integral becomes a thre Table 6.1 Working Rule DVM oF px+ 9=2 (derivative of quadrui) +p, | e get | bp 2a p nant f 2a ( (-- (« a tox te 5 = ds +} and we bp of yy cal de 9 ee | bet e}+ 5 ar torte In this case, the integral besomes | dash lax? + bx + ¢ \ = ar? thxtc + Ld = Vr rere + (q—) 2D jar rlere | The integral in this case is converted to i J(ex+0) fa? Fete ux 2 fears) far pierce ae | +(e 2) fle ee v4 Integrals of the Form Fae i s— dror Saree Sata Stant positive, negative or zero, Working Rule @ Dividethe numerator and detominator by x2, @ Putx—t zone yt Whichever substitution, on differentiation gives, the hhumerator of the resulting integrand, (Gi) Evaluate the resulting integralin > (iy) Express the result in terms of x, ds, where kisa con 6.40 Mathematics for IT—JEE Integrals of the Form 2 x zde, [AA a, where k isa cone Fekeee stant positive, negative of ero Working Rule ‘These integrals can be obtained by dividing numerator and denominator by s?, then putting x - and x x+Z = prospectively. 78 Integrals of the Form dx ct whereP,Qarelinearorquairatcfrtionsofe Ire 2 s | _TRICK(S) FOR PROBLEM SOLVING Integral ‘Substitution pat ego lene athe a tare J + bx +0) fort a a 1 Saale +brte. z i> (es) are Integrals of the Form J f(s (ae+5)""") ae, where rand n are integers Working Rule Purax+ b=2" Integrals of the Form Jr (s(ae+ 5)" (ax+0)°") where a, B, m, nm are integers Working Rule Putax + b= 24, where k=l.c.m, (n,m) Integrals of the Form x” ad ® Lge * ®) leur fo fomformry’ a TRICK(S) FOR PROBLEM SOLVING Integral Substitution Puta+br=z Seer nisa+veinteger i" Puta+ br=2x (ato? suhere either (m and p are positive integers) or (mand are fractions, but m+ p = integers> 1) Je arb’)? ax, where m,n, pare rationals. (@ pisa+ veinteger » Apply Bincmialtheoremto (at bey. (i) pisa—veinteger Put x ~ :* where & ~ ‘common denominator ofm and Put a+ bx"=24, where k= mei (ii) 7 is an integer denominator ofp. Puta+by'=x" + where k= denominator of fraction p. +1 - + pis sivimeger fy) Integrals of the Form J (xs hey Kk) Working Rule Purtyerk = = de k 7 A) Rationalising L.H.S. of equation (1). we get, Feet 40) From (1) and (2), we have 27 “Thus, the integral reduces to +k a 2 fro: Integrals of the Form [ (x Yor? +ox+e) x ‘Working Rule ‘We use one of the three Euler substitutions: iat sorve = rx t¢r00s are imaginary and a>0 us far torre - tye ifroots are imaginary andc>0 ML : Yar? +be+e =~ a) if ois a real root of ae tbrte, | E cution ‘The Euler's substitution often lead to rather cumbersome calculations, therefore, they should be avoided and applied only when itis difficult toevaluatethe integral by any other method. Integrals of the Form [e* [/(x)+ #"(x)] dx “Working Rule © Split te integral imo two integrals. (@ Integrate only the first integral by pans, i., Jetle@+r@lae = fet Slade + fe s(x - [709 af ser a] sfers(e)er =e Si +C. Integrals of the Form where the initial integrand reappears after integrating by arts. ‘Working Rule © Apply the method of integration by parts twice. (On integrating by parts second time, we will obtain the given integrand again, Put it equal to 1 (Gi) Transpose and collect terms involving I on one side aad evaluate I. Integrals of the Form: fe ‘e** sinbx dx and Je cosbxax Jets = ite Integration 6.11 Fp Mos bet sind) +6 = eof bx tan!?} 0 fa be a 2. fe costae = Explanation Lett fe onda Iegntingby rans, wes sinbe sinbx pe SOE fgg SOO ae ep eg | ger aff pat por sind ~& foe sind Integrating again by paris, we get 1 fot ine oe =O 08 ay] = he sinter + Se eosin fo cosbu de b o e a a = Sy (sind + acosbr) - 1 2 = (+3) = (+a?) T=e* (acos bx + b sin bx) = (a cosbe + Dsinds) +, ew * Sr cosby + sind) 3, Jorsiors eas = pe lasin Ort e)~b 0s (be +e)] +k “eh fa cos(bx + c)—b sin(bx + e)] + f Vat +8? a! (@ ] +t Reduction Formula ‘Any formula which reduces the given integral depending.on the index n> 0, called the order ofthe integral, to an integral ofthe same type with smaller index, is called a reduction formula forthe first integral, = ele 4, Jot conte ab ox +0- Some Useful Reduction Formulae (@ fsin® 6.12 Mathematics ‘or lIT-JEE sin xeos"™! x 2 fet sa ” a sin?" costly (i) Joost xax = (iit) sin” x cos! vate = tan” (i) fant xa ~ Joon? xt nol (i) foot” xete= cor” x~ foot" x ae OS all, (9) scot xa = ca [seo? xtan x+(1=2) seo"? xd] 1 (n-1) [-cosee™xcot x+(n-2) Joosee"™*x ar] (0, foosetaae Every antiderivative may not be expressible in tems of elementary functions such as polynomial functions, trigonometric functions, logarithmic functions, exponential functions, etc. We say that such «ntiderivatives or integrals cannot be found. For example, J ef dss} sine a, JR a, fina? ax, Jxtanx dx ete cos? pt =I Prd (0) fin? oust xe = fein? ? scoot xde PROBLEMS FOR PRACTICE 1. Foranatural number nthe value of the integral 3. Let/(x) bea polynomial satisfying /(0)=2,/'(0)=3 "(x)= fla). Then, F(4) is equal Pr $22" 42) 2x" 430" +6)!" avis mdf 2 FG) Thenf( neces * 5(—-1) Sieh +1) 1 7 @ FF oF © DAe +3" 6x EC le re" on 7 et @ 24 o LGM 43 4607" 4c 5+) 5 " 4. Lett #nn-l,me N.The value of mae ee etyllnel a - © aspen se tense fo RDI @)_ none of these 2sin(x* +1)+ sin 2(x* +1) cos’ x el 2 [ae (a) log|sec +c ) tog|dsee OF an]ec ifmateat ye! © log se (x? + 1)|+¢ © +f mur J © (d) none of these "3 1 co mest} 46 5. J {lonee-log;, e-logs, e}ax= 2 tax 1 wre (@) 5 {log.x} —log {log.1} + log {logx} +C aa You have either reached a page that is unavailable for viewing or reached your viewing limit for this book. aa You have either reached a page that is unavailable for viewing or reached your viewing limit for this book. aa You have either reached a page that is unavailable for viewing or reached your viewing limit for this book. 6.18 Mathomatios for 1IT-JEE Sead 40. I tax te where k= erty Pert @ 1 2 © 4 @s ‘cos 7x — cos8x a. seer (a) ©) sin2x_sin3x oO 4 (d) none of these. 42. Ift,= [x" ya? - 7 de, then (nt 2, —(a— Dad, (@) x"ya? =x? (bo) =x"! Ya? - (©) (A), IV (b) 8 16 0), 116 Ge lll & (a), Ver (0) A (OL = f(t 43") 0x8" 4 32" +6) ae | = foot ay Ge eae ee ¥ | = ferent) | | | | (2x +322" + 6x")! "de a = Yoo s2 eon 8 SOF? 6 666d | gegen Hetty AEB in 8 FOSS) sg ]ee" at where = 2x0" + 32" + 6x" ~o-t._1 _. al" 2S" 5a- =f on « ffenaes Fe-$ inje-iste =+1 | “yy " bey 12a). f= fri) Pe 1 Saar +30" 40x" )""4C Let 1x = 2 =~ 3d = 21 at (n+1) aa You have either reached a page that is unavailable for viewing or reached your viewing limit for this book. Putting Therefore, the given integral reduces to re fled }E =f -(:}-(eJ+e le ‘22 (bh). We have, 1 | (+s 4 x) (ne e30 +5)!" de a] (reel) (zee rae +6)! ae af (Co ete at) (aren 32s ext) de Putting 213° + 33° + 6x"=n = (6a! + 6ax"! = bax!) de= du 1 a Lue we du pa fuee +¢ forge de 1 a! 437? 46x)" 40 ~ 6+) vee . 23 (b). Putx-y=r (1) ‘According 10 the given condition ey? ie, yt @) Solving equations (1) and (2), we get andy P18 =P RD @-1 2 2-3, 2 Pe Indefinite Integration 6.28 -fit- 2 14 Putting? -1= 2 24 2 [and 2c = de -i|+e (cosx-+sin.x)+(cos.x—sin T-sinx cosx cos +sinx cas ~sinx. oomecesing 4. pp mermame: T-sinxcosx "4 T-sinxcosx ~ Sle] x4c08x. 4 Putting sinx—cosx <1 ~ (oos.x + sinx)de =dr and, 2sin x cos.x = 1 - (sin x cos x) =1-? 2a a 2H 2am +e Sie 2tan (sin x-cosx)+C c0sx~sin J ae tah, OSE Putting sins + cosa 0 =» (608.x=sina) de~ de Gin x+¢08x)= 1-2-1 a 2a sho | aay Sae {'] and, 2 sinxcos. Al, nl 8 B cosf xe ds 2+ sin 2x sinx +eose sinx +eosx +43 cosx~sin.x V2 (2+ 2sin x cosx) disinx +0083) 4, de 2500). | a (sinx + c0s.x) 626 Nathemates for IIT=JEE Putting sin x +-cosx= 4, we have 1 ale z™ tan"! (sinx + cosx)+C eC 1coa* x -j—*—, (I= c08" x) (I+ cos? x) ae -l ee ey (SSeS Ade (1-608 2x) G+ cos 22) Potting tenx=¢ = de= —4. and cos 2r= +0 4 de 1-P) +P aris alert] lee “J 40+?) d _ 7 d+? yd wares 9 P42) Dunx 23" 21 (). Given integral = | — i bate) Vian = (1+ 6sin x cosx) = f cians) see at Jian s (se? x+ 6un) Putting tan x =P => secx de = 2t dt af Ged yf le iereas 7) le (- [Potings + >| cae (SE fe. . == tan 2 cos5x + cos4r 1 2eos3x 28 (a). | = f SmBeCconse emt | sin3x—sin6x 9K cog in 3x-2.cos = cos si cos i = = J 200s 3 cos ax 22 == J (c0s2x+ cos) de qq Venetian +e j | | __ sin2x i 29(a). [ 2e0sa I c aa You have either reached a page that is unavailable for viewing or reached your viewing limit for this book. 628 Mathematics for IIT-JEE Also, sinx= VP) de lt ~aelnerze?)s +c [Pongo 2 2 =F sin"! os? Hee de leh Le t= oF cos Tx —costx AV Fs 2e0s5x 2 sin 14+ 2 cosSx Multiplying and dividing by sin =* ae Es sin +2sin = -cossx 2 2 ap tante sar rag eg SE 2 2 ak Put tan” Ln in + sin sin [ sin +sin ls =logl u ite r fed =log| tar" /=tog tan"t[ x41) +0 aa You have either reached a page that is unavailable for viewing or reached your viewing limit for this book. aa You have either reached a page that is unavailable for viewing or reached your viewing limit for this book. aa You have either reached a page that is unavailable for viewing or reached your viewing limit for this book. 6.32 Mathematics for T-JEE Again, dividing (2)by (1), we get 5 1 O cs 7 3ooi3x-cotx _ tan3x tan tan (b—Q) (re) anc Stand tanx—Stan3e 1 Ly des 3x4 2 a] = tan") F1an'09) +E ' Re, = =2. og | 3 #4) —2 tan" 4 dee 1 wa 3-1 a = tant (<- 5) Fam (+0 x) 3 and k= 8 (a,b). Let #2 oo, Put 2x+3 2y en Pas =2f— 7s _ @?-nGr-D ‘ of2 1 Les salt =tj% ar aa You have either reached a page that is unavailable for viewing or reached your viewing limit for this book. aa You have either reached a page that is unavailable for viewing or reached your viewing limit for this book. aa You have either reached a page that is unavailable for viewing or reached your viewing limit for this book. 6.36 Mathematics for IT-JEE | se (Putting tan x=2 => sec*x de = dz) in (tan x) de. 1+tan? x sesh I plttbere 2tanx 2 a [Putting tn x= 24 = sec*y de = 22 de] - By). J sand!2x al(nd 2 ams + 2 tan de sin} x =| —————_—*—_— « cos (seo x + cost) cosx fsee! x +1 +008! x sin} x de | Eieased [aaa [ratios ssonems— St | = seem ec! (see x + cos x). ee = | —2anoeoe eae (1+sin 9) sin? @= sin? 9 (Putting x = sin’@ = dr = 2 sin @ cos 0d] 72s" rox? 2 FMS ae=2 can 0 - seed) 7) py? (6). Let led)" ae Uvex) = {esos \'? d(cos? 29) 100820 cos? 26 Putting x = cos*2@ and dr = 2 cos 26 (~2 sin 26) d6, wwe have, jessy” 108 20 sin 20.6 cos? 20 a jee Eanes sp ame cos8 c0s28 20828 ~3{( S35} $4] -sex20) a0 = 49-2in|sec 26+ tan 20 Putting = cos 26 and dr =~2 sin 266, we have, T= 00s Treoag © 2si0" 8)d0 I= [0528 J 0s26 2 (-48in@ cos@)d0 =4 fcos29 om? a = 2 fc0s2014+ cos20)¢0 cos4e -2f(so20+ \o = sin20 -0 ~ aa You have either reached a page that is unavailable for viewing or reached your viewing limit for this book. aa You have either reached a page that is unavailable for viewing or reached your viewing limit for this book. aa You have either reached a page that is unavailable for viewing or reached your viewing limit for this book. 6.40 Mathematics for IIT-JEE QUESTIONS FROM PREVIOUS YEARS’ IIT-JEE | a (043228 440M) +6 where k= 1. The value of the integral [> sin 5 “te (@) sinx—6 tan" (sin x)+c | @s5 (b) 6 (b) sinx—2(sinx)'+e os @ 6 (©) sinx—2 (sin x)! 6 tan“\sinx) +0 ‘ (@) sinx—2 (sin x)! +5 tan"\sinx) +e *! ff pate J be ies = #( cos"! Ve — og) 1+ =x |- 2. Thevalue of [Sa is (co fi-Wog|1+ Ji=x|-2 all}te, ® ; 3 7 (d) none of these i feat () 5\?2-—r tate none of these feos: 1 ©) BP RATT nm oft ESE ater GTI = dx+ Blog (9% —4) + C,thon 3. Ax + Blog (De —4) + Cy th amt, a “FF Wi-fi | ) B= (@ none of these where k= @1 @-1 4. Forany natural number m, ©2 @ + Jeens2ns2 02843" 6)aex50 - 2@ 2 @ba 4 & 5. @) & @ aa You have either reached a page that is unavailable for viewing or reached your viewing limit for this book. aa You have either reached a page that is unavailable for viewing or reached your viewing limit for this book. aa You have either reached a page that is unavailable for viewing or reached your viewing limit for this book. TA Mathematics for IT-JEE Explanation x Also, when x= b+na,y= band when x=¢+na,y=c [Punting +=—y and dt =— ah) J fede = [sr naydy = fronds =Jrione ba 5 ; : [fis odd function, =. /-9) =f) a = fried Ae) 16. Forany two functions (x)andg (x), integrable on the interval (a, 0}, the Schwarz-Bunyakovsky inequality | holds F(x) is an even function, if f(0) is odd. 20, 1£/() isan even function, then F(x) = [ f(@)at is an 5 < [JP @de-J ede odd function, 17. Ifa function f(x) is contiruous on the interval [a, 6], | Explanation then there exists a point c¢ (¢, 5) such that a [J £0) =) i Fens | fae Jf) 4-1 -@), whew ase 0, et f(x) = peta Then, the value of i sors(t)s @1 2 ©} (@) nore ofthese. ‘The value of the integral fee*Pcos(sindya0 is 0 @ 0 On Ox (@) cannot be determined Ler/bex real valued function satistying /(6) +/e+6) mp =fert3)+fe+9).Then, J f(a is (@ alinear function (®) anexponential function () a constant function (@) none of these sees = 279) f()dy, then f(x) attains ° arminimnum at 8 8 @ a5 x=-5 8 9 Og @-3 im (ADS ht PP +24 ht) iy (+ 2434.4 08) is equal to 7 2 OD or aa You have either reached a page that is unavailable for viewing or reached your viewing limit for this book. aa You have either reached a page that is unavailable for viewing or reached your viewing limit for this book. aa You have either reached a page that is unavailable for viewing or reached your viewing limit for this book. 7.20 Mathematics for IIT-JEE (7) , f= __ ae. ™ ye? 14x +49) + [27] (C) denotes the greatest integer function) The matching grid: L@OO@® 1@008 2 1 fx y2x-2? ae a aa Wf [sinx}de= sa (where (-] denotes the greatest integer function) 1 HL ff sin 2ex| de = w polsak ‘The matching grid: L@G®O@® 1006 3. x L fdx-2i+bar= wf Ss a= [x] stands for greatest imeger less than or equal tox) I, If [(6~4x)dr26~5b, b> 1.then b= i * x WW If = J fteos? x) de and Jy = J f (cos? x) dx a a then fy = kf where A= ‘The matehing Grid! L@G©O® 1@GO@ «d) ML. @ (b) () @) nt b) © (a m. @20eO@ x 5, z 3 2 z @0O@ 2 z 7 a @®O® vV.@C@@ aa You have either reached a page that is unavailable for viewing or reached your viewing limit for this book. aa You have either reached a page that is unavailable for viewing or reached your viewing limit for this book. aa You have either reached a page that is unavailable for viewing or reached your viewing limit for this book. 724 Mathematics for IIT-JEE 1) 2G) 4. id) 5. @) 6. (b) 7 & 9%) 10. 2b BO % @ @ 1% 17) 18. 20.) 2. a, @ 2%. 25. b) 26. @ 28, @) = 29. 30. by wo 32, @) 33.0 3. 36. =| 3. 38.) 40.) a1.) a) a. © 45. 8.) (Ba) 49. (50. (a) 52 53.) 54.) (B56, Oh) 57.) 58. 60. (a) 62. (a) @ 6) 65. 66. 68. ©) «69. 6) 70.) @ 2 6 73. = 7% @ 7%) 7 © 78. @) @ —-& b) a1. @) 2.) a4. 8S.) 88. Ob) (BB. a. = 90. @ 92. a) 93.) id) 96. tb) 97.) 98.) 100. (6) 1. @b) 2 @od 3. © 4 @bd 5 @b) 6 @b 7% @d 8 bad 9 (bc) 10 (be) 11. (12, (a,b) 1. (10) 2) 3. 4a (4) x 2 9) 10, 5 125 13: 8 ia a 6 70 1% @ 1 190), 1 ©, ho @), V > @) 2 19 (b), 1 > (©), I > @, V> @) 3. 1 (<)> @), I> (@),V > () 1 ©) 2 © 2 @ @ 5. () 6 (©) 8 2 @ 10% % 11 @ eae 18 a 2 @ 2 @ a so) me me aa You have either reached a page that is unavailable for viewing or reached your viewing limit for this book. aa You have either reached a page that is unavailable for viewing or reached your viewing limit for this book. aa You have either reached a page that is unavailable for viewing or reached your viewing limit for this book. 728 Mathematics » I | folde=a {given) a 45 (a). Weare given [lro-a)a = 3 [2a)*? -3a+3-2 V3]. = | cosxdr< Feossinsy de 2) from (I and (2) 4 >4,> a, nM ! 52 (0). 6,404.97 float” + cot"*?3) ds,n22 Differentiating w.nt. a, we get fay~ 33 Vaz 2-3] = flay=2 V2a,a21 f()=2 VIX, x21. 48 (a), The required area - [urine 2-41 sothatdr=— 24] sint auf hoe Vieni 54 (a). We have, sinx0 => sin (60s x) a ands, cre [0,5 x>sinx > cos x 3a 0424072) sin’ lass f @-Ep © +f opps fc ed > sint=+ sc0s=b ait i 3 4 ms +f ee-tepha f= 3 wis 2 JF =a sin —a)eoss | m4 =] -plaes | @-aPder | (r-3) de i =f @=()He= i 2 3 H6 = fin sin sina 3 vin =D] ” | DEES ear a) sin % sin DE) 4 sin Hsin #2) (x + 2felhde (fe Integer] fa) + Inepe tet)“ 1) | Li] + 2E]de = i Bixee -s{foa oprah 317} =30- =f Ute + J taal =3, aa You have either reached a page that is unavailable for viewing or reached your viewing limit for this book. aa You have either reached a page that is unavailable for viewing or reached your viewing limit for this book. aa You have either reached a page that is unavailable for viewing or reached your viewing limit for this book. 7.38 Mathematics for IT-JEE to Jracf ra Sine ar-120 for all x, so the only possiblity is a 1 a . [ear 2 ax> J and sinx=~| . = x= 2nr 5 Therefore, ix . £) £0) 1 . since, tim |1+ £22] exists, so tim £2 ave 93(b). Si fin + is, fim 2 “ Af f)tagttagl+ tag", a,40,n24 since, (0 wena etic)" at 96(0), Pu g(x 21) =F soo}* tim || +2) se pa,= = awe inl] cesertorer’ Te The graph of y=x‘ and.x? +)? =2 are shown in the figure = Fal ae teded ye@l 2e Foe (ete > a= | When x 0,190 and when x0, 1-9 20 aor j ie Ja 94(d). For O—xsothae > eg Therefore, fe cos! xdr> J e“*cos? xd a Also, cosy <1, therefore Dg |—— 2a [aame™ Game j cos? xde tan? < tn? x i aie -. g (a) is periodic with period 2 | ke pa . gQn)=0 mal <2 40S pad 41 (0,0), Ife, and e, be the eccentricity of hyperbola and Legged pours [% aan Asa aa You have either reached a page that is unavailable for viewing or reached your viewing limit for this book. aa You have either reached a page that is unavailable for viewing or reached your viewing limit for this book. aa You have either reached a page that is unavailable for viewing or reached your viewing limit for this book. 744 Mathematics for IT-JEE in(e? + 8¢ + 13)dr Inthe second integral = f sin(x? + 12x+33)de 2 Put x=(-142)1-2=1-2 3 sin(1? -4r+ 44121 = 24433)de P | = fing? +81-+13)2r ° & Hee | 18 (4). We have, #2 2(cos3x 3 ) , a?{ 22835 3 coy + asins 20 c0sx te i ( oe ai = [2 (Bef aios}-asour-20sine 2 , jp een 202 7F +0 20 2 tie ta - 205 Given: 7 £,@°+a-20S0=-S 3a mmber ce [E. 3 such that fO=h wo sor ededpea ia) = legys lim = BPS pen omll-Blea}--3] 1 = jin 4 $ tog(1+4)= florea ae 0 \ = [rls +90) - [5 0 =e2-|{ log? —[x—log (+3918 log? —[(1og2) 0] =2 log 2~log e=iog + 3(a). Wenave,2/00+3/(4) = 2 () +3fQ)=x-2 ig the above two equations, we get 4 a). Let aa You have either reached a page that is unavailable for viewing or reached your viewing limit for this book. aa You have either reached a page that is unavailable for viewing or reached your viewing limit for this book. aa You have either reached a page that is unavailable for viewing or reached your viewing limit for this book. 7.82 Mathematics for IIT-JEE 12 13. 14, 15. 16. 7 Letf(e)=x- (2 forevery real numbers, where[x] 6 | the integral part of x. | : Then, | fee) dis s @1 @2 | 1 ©0o @- | For any integer a, the integral | fe costant Dede | a | has the value @ © ot ©o @) none of these If JG) is differentiable and fx f(x) de = 4 1) equals 2 @ 5 1 @ 3 Let f(x) = [ {2—2 de. Then, the real roots of the equation *- /’(x) =O are L @ 41 & 475 © 4 @ Oand1 \ 1 Iti (m,n)= J M+ 0)"dt, then the expression for U(r n)interms of 1(m+1,n= I) is r n © Spe the () Seri mt had 4 msn) © m+loom+l mein n @ Bylot 1 The value of the integral f [—* de is | o Vite | @ Sn &) F-1 | © -1 @i | 19. 20. 21. 2. 23, fu, 1pce)= fe” at, then f(a) increases in (@) (2,2) (b) no value of x () Os) @) -=,0) The area enclosed between the curves y= ax* and x= (a> 0) is 1 square unit, Then, the value of "a" is @ 5 ©) © bie wie @ The area bounced by the curves y = /(x), the x-axis and the ordinates x= I and.x= bis (b~ 1) sin(36+ 4) Then, f(a) is {a} (x 1cos(3x+4) (b) sin@x+4) (6) sin Gx+4)+3(e—1) 08 (3x44) @ Let T= 0 be a fixed real number. Suppose, fis a continuous funetion such that for all x € Rf (r+) aa none of these =f(e).lfl= Jere, ten hevaieot J fond , a 3 is cc) 3 7 (©) 37 (d) 67 Ifg (x)= J cos' radi, then g (x +2) equals. (@) e+e) () ea) lx) © swam © , C5 €4y 05 are arbitrary constants, is whose general @s 4 © 3 @ 2 Itxdy= y (de + ydy), (1) = 1 and y(x) > 0. Then (-3) = @3 (b) 2 o1 @o 1 @ 2 @ 2 ao 4 @ 5 @ 6 @) aa You have either reached a page that is unavailable for viewing or reached your viewing limit for this book. aa You have either reached a page that is unavailable for viewing or reached your viewing limit for this book. aa You have either reached a page that is unavailable for viewing or reached your viewing limit for this book. aa You have either reached a page that is unavailable for viewing or reached your viewing limit for this book. aa You have either reached a page that is unavailable for viewing or reached your viewing limit for this book. aa You have either reached a page that is unavailable for viewing or reached your viewing limit for this book. aa You have either reached a page that is unavailable for viewing or reached your viewing limit for this book. aa You have either reached a page that is unavailable for viewing or reached your viewing limit for this book. aa You have either reached a page that is unavailable for viewing or reached your viewing limit for this book. aa You have either reached a page that is unavailable for viewing or reached your viewing limit for this book. aa You have either reached a page that is unavailable for viewing or reached your viewing limit for this book. aa You have either reached a page that is unavailable for viewing or reached your viewing limit for this book. aa You have either reached a page that is unavailable for viewing or reached your viewing limit for this book. aa You have either reached a page that is unavailable for viewing or reached your viewing limit for this book. aa You have either reached a page that is unavailable for viewing or reached your viewing limit for this book. aa You have either reached a page that is unavailable for viewing or reached your viewing limit for this book. aa You have either reached a page that is unavailable for viewing or reached your viewing limit for this book. aa You have either reached a page that is unavailable for viewing or reached your viewing limit for this book. aa You have either reached a page that is unavailable for viewing or reached your viewing limit for this book. aa You have either reached a page that is unavailable for viewing or reached your viewing limit for this book. aa You have either reached a page that is unavailable for viewing or reached your viewing limit for this book. aa You have either reached a page that is unavailable for viewing or reached your viewing limit for this book. aa You have either reached a page that is unavailable for viewing or reached your viewing limit for this book. aa You have either reached a page that is unavailable for viewing or reached your viewing limit for this book. aa You have either reached a page that is unavailable for viewing or reached your viewing limit for this book. aa You have either reached a page that is unavailable for viewing or reached your viewing limit for this book. aa You have either reached a page that is unavailable for viewing or reached your viewing limit for this book. =n Now, 2 cos a+isin a are where oristhe angle between (2, ~z,)and (2, +2,) => a= —2tank is the angle between (z, —z,) and (+2). 29 (b). x? Vix +1=0 =0-48=-48 30(b). |e, +iV55) + BR, +iN32,)° = (v2; +W35)(125-N5e9) + (V3q +229) (V3. =5 (12; PH 2, F>5-2ylaP lal Since 4» M> G--Mfor| 2,|#12)1 34 (b). Ifzis aroot of (2—1)5 =2ar(z + 1)5,then Gf att 0 Ee As2'™* #1, we get zlies on acircle. 32 (a). =, +2, ) ay 2) 4 8) From equations (1) and (3), we have ~ ~b 204+)=> 2 FI-9 Complex Numbers 9.27 & aa 9" G2! From equations (2) and (3), we have = a 4 ” and angle betweenz, —2, and 2,28 5. triangle is equilateral 34 (d). 1f 21 =z, the given equation becomes 4 Poztl=0 ie, z=-qand- a? , and, =|z)|| 1+ |=12 || -@? |=]291 ‘Thus, 2,2 and origin form an equilateral tangle. lz, 35 (a). We have, |-—22) = => = lies on the perpendicular bisector of the seg- rnentjoning (1+ O/and (1+ 09, 4 sb saiforsomeae R a a aa You have either reached a page that is unavailable for viewing or reached your viewing limit for this book. aa You have either reached a page that is unavailable for viewing or reached your viewing limit for this book. aa You have either reached a page that is unavailable for viewing or reached your viewing limit for this book. Complex Numbers 9.31 56 (b). Given 22 4 = (J3 — 1) + 1 (V3 +1) ” =L then x, + iy, = kG +) Ia If k= 1, 2, = zy which is not tive and if k #1, 121412, It 1222 js purely imaginary. 54 (b). Since | CA} = | CB | and ZACB = 90° : i@-2) = = 22,25 ie ape 2) = 2 [zy - 231 [5 - 2; = a)" = 2 fe, ~ 23) 25 - 22] 2 1 [ove (200 +5 sain (mS 55 (a). Let A=2,, B= 2, and C=2,, 12, a2 where 4, B, C are vertices of equilateral triangle. Using (cos @+ i sin 6)" = cos n8 + i sin n8, we have Given that third point C is crigia, so 25 1 Sn (2mm +22 Jo isin < | ne += Ue some enpeaiks ans fase aa or, #7 2) = 2-2 n=01,2,3 Bat Bt yaanath 6 on a+ B+7 wl) ST (6). & (sn 2H ins tt) Since the tang is equlateal tangle, ne cana = fin +s $+. 112m} on, ~O|=|O-z,[= 12-2] LE n or, Bi=I71 2 cos oF 1 Joos + cos © 4...4 10 terms %, (BP =|rP { 1 | i sin (2% 49% Jain OF cog 2 onan Te. si <1 Ni (1) 2am = Y6p+2)] ¥ [sin 7 -sc0s s x Pp fE a “ Zopeni =3 Epirea Er rl rl on, oda Q) Now 4 =i +22 +39 +. +32 om AiR PED + BU + 32 AM pHs eRe 32 or, on py 3-3 Ee 3204) gg ‘i on pt =) 6) and, B=i+?2 4. +72 =0 ® or, Hence, $= 3x 16 (1-1) = 48.(1~ 0. aa You have either reached a page that is unavailable for viewing or reached your viewing limit for this book. aa You have either reached a page that is unavailable for viewing or reached your viewing limit for this book. aa You have either reached a page that is unavailable for viewing or reached your viewing limit for this book. aa You have either reached a page that is unavailable for viewing or reached your viewing limit for this book. aa You have either reached a page that is unavailable for viewing or reached your viewing limit for this book. aa You have either reached a page that is unavailable for viewing or reached your viewing limit for this book. aa You have either reached a page that is unavailable for viewing or reached your viewing limit for this book. aa You have either reached a page that is unavailable for viewing or reached your viewing limit for this book. aa You have either reached a page that is unavailable for viewing or reached your viewing limit for this book. aa You have either reached a page that is unavailable for viewing or reached your viewing limit for this book. aa You have either reached a page that is unavailable for viewing or reached your viewing limit for this book. Compex Numbers 93 Subtracting equation (2) from equation (1), we have, 10(48). Wehave, 4a) VP-P EID td =V3Gi-mIOity) —_ [using] ase Lig _ oo 2 tid 2 4 Wty IG tH)= Taking z= 10s + isin = 9 (10). We have, = Me cos Et isin n= 1,2,.0,24 where wis acube roct of unity ‘Now, 25 =P qi" = (2 « (axa!) =CI(0) (f= 1anda=1) and, $1 fl a; 2 +P =i tay (ia? [l+o+o?=0) =P? x22) (WE x oP) = Po? “According to the given condition, we have 2 = (24 64 = G@y' =P? = xa a1 Then, n—2~4a and 2n-2=3bwherea, be 1 Eliminating n, we have 2det2y-2-36 ie, M2 Smallest ategeral values fora, bare a=2andb=6 and hence, smallest integral value of n is n=4x1+2=10 21d). Since zcos(Z)+ran(2), = IGA )=0az-H0r2=1 FELD Now, 2=-i=|z/=|i)=1 we avez 292 an, | 2=-1=9]2|=|-/13|2P=1 12191 -( 2 =| —_ +) ‘Thus, in both eases | 2|=1 =(cos-+isin2)| cos-4-+ sin isin 3 3. Fs ¥ 3 3, WI (c). Letz=x+ iy . oebs2 ADE ee [G=2)+o][(+2)-i] i (42)? ty? 3 2G? ty? ay +1(4y) wee (E+- SF Jes ae a & fl ¥ int Gray moos FE +isin = 04) 1-1 2 ee Since 22 is purely imaginary, «2? +y?—4=0 => iF(@-)-E-)=0 = x4yPigalz = |z=2 aa You have either reached a page that is unavailable for viewing or reached your viewing limit for this book. aa You have either reached a page that is unavailable for viewing or reached your viewing limit for this book. aa You have either reached a page that is unavailable for viewing or reached your viewing limit for this book. [cos (a~ B)+ cos (6-7) + cos (y- a] +i [sin (a— B) + sin (B— y)+ sin (y- @)] 140i. ‘Comparing real and imaginary parts, we get cos (a B) + cos (f— 7} + cos (y- a) [Putting 2 =2+ iy] : ep Os 8 = sty?_2.y-120,which isacirete, B 3 (a). Area of the triangle on the argand plane formed by the complex mmbers~2,i,2~icis® [2 3 J lzP=600=3)2|=20. ‘4{a). Given, | z—1|+|z+3|s8 + Zlies inside or on the ellipse whose foci are(1, 0) and (© 3,0) and vertices are (— 5,0) and (3, 0). 10) Now, |2—4] is distance ofz from 4,0). Minimum distance | is | and maximum is9. | Complex Numbers 9.47 5 (a). We have, = files z > [zP-2|2|-4s00r(|2|-1)?-5s0 => (12|-1PsSor|z}-1s-¥5 =|z/sV5+1 Hence, the greatest value of |z | is V3 +1. 8 (a). |2—1|=|z+5/ | represents equation of perpendicular bisector of points (0, 1)and(0,-5),ie.,y=—2,now| z|=2is etyped = Ptd=4 = x=0 _ represents a single point (0,-2). 7 (a). We have, (129) FyG)= C1 ~2g2) (12) Fg) on 292) U2 PV 2 VL ta (I Ha) = (1-22) ta?) (+29) =(-27") aan) 8 (a). Sinceamp. (2, (2,-2,)]=amp .[z,(2,-2))] nla-2)) 4 wo( 2223) (2-2) (2 ~4) Hence, 0,,,25,23 are concyelic (: for four concyetic points Gian ea is purely real. is purely val) aa You have either reached a page that is unavailable for viewing or reached your viewing limit for this book. aa You have either reached a page that is unavailable for viewing or reached your viewing limit for this book. aa You have either reached a page that is unavailable for viewing or reached your viewing limit for this book. aa You have either reached a page that is unavailable for viewing or reached your viewing limit for this book. aa You have either reached a page that is unavailable for viewing or reached your viewing limit for this book. aa You have either reached a page that is unavailable for viewing or reached your viewing limit for this book. aa You have either reached a page that is unavailable for viewing or reached your viewing limit for this book. 40.6 Mathematics for lIT-JEE (b). The equation having a, b as its roots is w= x(a b) + ab =0orr? 24x + G?=0 [4 and = ab] () The equation having its roots as the given num bers is? -24x + & 24 (4a? 4G? Harmonic Progression (H.P.) Assequence of ron-zer0 numbers ay, ay a harmonic progression if the sequence tad isan AP. la For example, the sequence 1. 3. 5. 39+ isan HP, since the sequence obtained by taking reciprocals of its, corresponding terms, ic, 1,4,7, 10,...i8 an AP. toot A general HP. is, 5, 3g nth Term of an H.P. L ith term of the corresponding A.P. nth termot H. IMPORTANT POINT(S) TO NOTE 1 Harmonic Mean (H.M.) Single HarmonicMean A number His said tobe the single harmonic mean between two given numbers a and bif.a, H, bare in H.P. Li 1 1 Forexample since 3,5, 5 arein HP, therefore, isthe E seal HM, between 5 and m-Harmonic Means The numbers Hy, Hyy 5 H,,are said to be the harmonie means between two given numbers 4 and Dif, Hy, Hyyoom Ap Darein HP ie, InsertingSingle H.M. between Two given Numbers Let and b be two given numbers and HI be the HLM. between them. Thea, a, H, bare in HP. ie, 2ab a+b Inserting n-Harmonic Means between Two given Numbers Let /y, Hs, H, be the m harmonic means between two given numbers and b, Then, a, fy Hy wn Hyy bare in HP. Thus tad 1 Sd dF arein ae. OH Ha, L Now, 5 = (+ 2h term ofA P. 442-1, a where dis common difference of A.P. Thus, we get a mel abn 1) ant bra a akinsi) ~ abin+D) aint) % hme 1, Ma-b) _2e+(n—b 2S sxe) abntD) abn & ca eee aba+t) Ingeneral, we have H, = Relation between A.M. G.M. and H.M. Let, G, and H be arithmetic, geometric and harmonic means. between two numbers « and b. Then, $b ab 0 4 S86 Sai ants 28 (i) G=AH tw a>aon Explanation Let aand 6 be two real positive and unequal quantities and 4, Gand ffbe the singleA.M,, GM, and HM, respectively between them, a+b a = vab = 22h Then, 4 2 aed aa You have either reached a page that is unavailable for viewing or reached your viewing limit for this book. aa You have either reached a page that is unavailable for viewing or reached your viewing limit for this book. aa You have either reached a page that is unavailable for viewing or reached your viewing limit for this book. aa You have either reached a page that is unavailable for viewing or reached your viewing limit for this book. aa You have either reached a page that is unavailable for viewing or reached your viewing limit for this book. aa You have either reached a page that is unavailable for viewing or reached your viewing limit for this book. aa You have either reached a page that is unavailable for viewing or reached your viewing limit for this book. aa You have either reached a page that is unavailable for viewing or reached your viewing limit for this book. aa You have either reached a page that is unavailable for viewing or reached your viewing limit for this book. aa You have either reached a page that is unavailable for viewing or reached your viewing limit for this book. aa You have either reached a page that is unavailable for viewing or reached your viewing limit for this book. Ln 10.18 Mathematics for IIT-JEE 10. na The sequence {S,,..) is (@) increasing (©) non-monotonic (b) decreasing @ unbounded Say 1~ Spy must be equal to @ Gy 0) ay” (S-8:) o Gy (&-S)) @ Zero T'S, >S,.then lim S,must be equal to @ 5-5 8,425, @ SS (©) nome ofthese Instructions: Jn the following questions an Assertion (A)is | 3. Assertion : Ifa,b,c,de R+anda,b,c,dareinHP, given followed by a Reason (R). Mark your responses from | thenb+e>a+d the following options: Reason =: H.M.>A.M. for unequal numbers (a) Assertion(A) is Trueand Reason(R) is True;Reason(R) | 4. Assertion : The sum of the series isa correct explanation for Assertion(A) | 1 2 3 (b) Assertion(A) is True, Reason(R) is True; Reason(R) | TP +if 142? 424 143? +34 isnota correct explanation for Assertion(A) | tomtermsis 20+ (©) Assertion(A) is True, Reason(R) is False | Un +n4l) (@ Assertion(A) is False, Reason(R) is True | Reason: The nth term of the above series is 1. -Anertion. : Betweea two numbers wiiose: sum is: | af) As i | 21S (=n Tenn, 2G an even number of arithmetic | 5, Assertion : Thevalueofx+y+zis15ifa,xy,25 scans ae inserted Ifthe sum ofthese sea Ratienevaneot ts byt means exceeds their number by unity, x ye then the number of means are 12 | is § ifa,x,y,2, barein HP. The values Reason: ‘Ifa and bare two given numbers and | Aj Ay «4, are n arithmetic means aa sil bag 0, respeétively- Be ee a Reason =: The sum of m A.M.s between two een them, then Hee i = di ‘quantities is equal to » times theit single Ath vr dyan( 22) sean. 6, Assertion : For every natural number 2. Assertion = If a, 5, c are distinct positive real aly numbers and + 0?+2= 1, then ab + oscar cn (BE be + ca is less than 1 2 Reason =: A.M > G.M. for n distinct positive Reason: A.M.>G.M. for unequal numbers. aes 0 ANSWERS 2 © 3 1. @ Th 18. (3) 19. 26.) 27. 3a 35. a. Q 4 50. (b) SI. 58.) 58. 6. 6% 6 2 a @ m2) 15. @ 16 @ 2) 26 2m & 30.) 31. 8) 38. 3%. @) 40.) 48.) 47.) a, 5.) 55. 56. ab 6.0 64 @ 7. (3 aa You have either reached a page that is unavailable for viewing or reached your viewing limit for this book. aa You have either reached a page that is unavailable for viewing or reached your viewing limit for this book. aa You have either reached a page that is unavailable for viewing or reached your viewing limit for this book. 4 a Gy bay bahay’ ay Pay hah arein LP. a) 74, +a, 43 (¢). Let the three digits be a, ar, ar”. ‘Then, according to the hypothesis, 100a+ 10ar + ar? +792 = 100ar? +10ar+ a = a(?-=8 and, a.ar+2,a7? rein AP, then, 2(ar+2)=atar = a(P-2rt 4 (2) Dividing (1) by @), av?) 8 then, SS =F ah 4 GeDe=) oe 2 (r=? tl 232 from(I),a=1 Thus, digits are 1, 3, 9 and so the required number is931, j 44 (c). ‘The general term of the given sequence is 2 i ee | ** 500+3n° i dT, _ n(1000—3n3) | then, = Se = So | dn” “(s00+3n°) | Formaximum orminimur of, | a, _ = | 1 n= (top \ 3) Now, 6< (222) <7 Hence, 7, is largest term, So largest term in the given is 9 sequence is AB {c). Consider the equation =D &-2 &-3) = 100)=0 Itsrootare 1, 2,3, .. 100. (1)isapolynomial equation in x of degree 100. Coefti- cient of x! =1, Now, sum of the roots of equation (1); taken one at a time ma) 1p! Seettofx”® cocif, of x coeff. of x”? ie, 1243+... +100 Wwox101 s+ coeff. of x =—(1+2+3+...+ 100) 2 = 5050 1100 wkentwo Sum ofthe product ofthe routs 1,2, 3, statime aye coe of 7% coeff, of x1? coeff, of °8= sum of the product of 1,2, 3, taken twoat atime = coef. of , 100 f(i+243+ 100/72? 1 2 1002 101%201 z [soso 6 ] 1 cos? -sssso jets 4G (b). Let the sides of the triangle be a, ar, ar’. If |, then the three terms of G.P. will be a, a, aand tence an equilateral triangle will be formed. ‘Thus whenr= |, tringle will be formed a) Ifr> 1, then greatest side will he ar” and inthis cae ‘triangle will be formed if atarratar-r-1<0 IB tS 2 in m2 = [ere t).2) Ifr< 1, then greatest side will bea and triangle will be formed if artaP>a =P+r-1>0 ret opp es 2 z Fah erect. 0¢r 1. 4 aa You have either reached a page that is unavailable for viewing or reached your viewing limit for this book. aa You have either reached a page that is unavailable for viewing or reached your viewing limit for this book. aa You have either reached a page that is unavailable for viewing or reached your viewing limit for this book. 10.28 Mathematics for IIT-JEE 65 (b). We have, S,=PABD4PHSH ES +, Let n—2m. Then, Sy =(P +39 +5? ..tom terms) +3(2 +45 +67 +... terms) H (BEDE P+ B+. + Qm-1P+ Ome} {244+ (mp yD +4 26+ + my} = MORN eae 12 02 40m nein sty =n m= 116 MO _ Po? +3n+1) 2 66 (c). Since p,q, rare in A.P. we have P-q=g-r=k(say) (0) a-x_a-y @ | aj) (a4 . He) Pg ar (by componendo-dividendo) = (Using equation (1)) | 2 Aoci = = 2.441 yx fe uyse ee nL, 67 (a). We have, SH1++ab+( +a 2) 4 sate +a) B 4.00 1+ B+ BP +.c0)$a(B+ B+ B+.) +a (Pb... wt 1b a 1 1 (6, d). Let the numbers be a and b, If 1 is the harmonic ‘mean then H=4,244+G?=27 (1) (Given) 69 (a). Given: q+ Sd=2 Let y= ar eter =ay (a 3d) (a ~Ad) =(2-Sd\(2-2d)(2—dd (Putting a, = 2 ~Sd) =2(4—16d+17a?—Se) The value of dat which y attains maximais given by ans mr (by calculus) = -16+Md-18P=0 34414 a de 2 4-30 d> Oford= = 3 8 0.7>1 | weacawena rar a er~lx 66 (8) Since.p.g,rarcin AP yar Kart Pa P28 ee Met, ot) ane Since rootsof px? + gx += Carell real, therefore, ny BS art) EE rey ; ov ; From (1) and (2), #-sr203 ( | aereO he) “ra 18 gg ten & edtuseeo SetxPcuapee on ie, (@-2)(a~68)=0 = (2) -u(2\ei20 => or 64, (Pa Pp Then, ~ (2 nya =f kaa Agr tyr" =32 Ve ei 0, 202) 17 (8). Givena, = V2a, . Aloe {+ Length ofa side of S, = length of «diagonal ofS, 204-327) ayes 1 20232) _ 196 which gives r=2 = wt Beincin’d an 2 Bone ce ay dy. reinG.P. with istterm= 10 andcommon ie. n=6. Hence, there are 6 terms inthe progression ratio 48 (5). Letthe9 AMsbe dy Aeon i Then, 2, AjAyy wn Agy3 are IMAP. (a) If isthe common diference, then We 1 ‘Also areaS,=02,<1, woof 5) et 3=tlthterm=2+ 10d ord= 7h 100<2"'=92"- "> 100 12 29 aede2e tes) gn asgg- 22 * Thesmallest valueofn=® (+: 26=64,27=128] Bededs Ta Ze adgnaeed= 2 t aa You have either reached a page that is unavailable for viewing or reached your viewing limit for this book. aa You have either reached a page that is unavailable for viewing or reached your viewing limit for this book. aa You have either reached a page that is unavailable for viewing or reached your viewing limit for this book. 10.36 Mathematics for IIT-JEE Subtracting (2) from (1), weget (-A)8 =342rt2r? +27 + 100 =342r(I 47+ P+ a) SHU P=2701 9+ 18 = A(L-2r+P)=27-27r+ Br = 477974170 Tt (1)? 4x44 x17 xi _ DENT _ 79257 7 Now 77> Liea|r|> 1 1 iri (Given), axa) 3, did ay? 4 Ga? d-2. 8(c). Wehave, 3! 929. 2727. 8148"... to00 we) Let tom 1 as 3 On subtraction, Pedtyty ty 3°30 9°27" 81" 1-v3 2° 9( ).and10( ). Sine s,= S182 5,<5,<5, Sy + Sy Sy + Sy Again, Sy 3S) <54 5558, Further 5 Continuing in this manner, we will get S,< 5455, = Sy Sone Sp 55 <55 5p =9Sequence /S;,] is inereasingand {S,,,,,, decreasing, Thus, correct choices of are (a) and (b) respectively. These sequences cannot be ‘unbounded since all terms of the sequence lie between S, and S, 5,5 <5, <5, 44 (B). 53.01 “Say = B(Sag + Sipe) “Son (Son ~San-1) Efe Continuing in this manner, we get wel Suse (PB) (@) This recursion relation shows that this choice is not true, (©) is ruled out because 5,, , ; ~5yy is alway positive (d) is ruled out since 5, ,, is stric:ly greater than Sy and so on. aa You have either reached a page that is unavailable for viewing or reached your viewing limit for this book. aa You have either reached a page that is unavailable for viewing or reached your viewing limit for this book. aa You have either reached a page that is unavailable for viewing or reached your viewing limit for this book. Quadratic Meprxions and Expressiouy REVIEW OF CONCEPTS Solving a quadratic equation is one of the most important and useful topics of algebra. It finds its applications in almost, all topics in mathematics especially in Algebra, Geometry, Trigonometry and Calculus, Quadratic Equation ‘An algebraic expression of the form: a? + bx + ¢, where a (40), 6, c€ Riscalled a real quadratic expression ‘An equation of the form: ax* + br + c= 0, where 4 (20),b, c€ Riscalled a real quadratic equation. The numbers a, b,c are called the coefficients of the quadratic equation and the expression b? — ac is called its discriminant. Diseriminant of a quadratic equstionis usually denoted by Dor A. Roots of the Quadratic Equation A root of the quadratic equation at tbxte: isa number @(real or complex) such that ac? + bor The roots of the quadratic equation (1) are given by i? = 4ac Nature of Roots of the Quadratic Equation 1, IfD<0, then roots a, Bare imaginary 2. IfD> 0, then roots o, Bare real and distine! 3. IfD=0, then roots a, Bare real and equal TRICK(S) FOR PROBLEM SOLVING For the quadratic equation ax’ + br + c= 0 © Onerootwillbe reciprocalof the other ifa=c © Onerootis zero ife=0. ‘© Roois are equal in magnitude but opposite in sign if Pa * Both roots are zero if -0. ‘© Roots are positive if D> 0, a and ¢ are of same sign and bis of opposite sign. ‘© Roots are of opposite sign if a and c are of opposite sign. ‘Roots are negative if > 0 and a, 6, care ofthe same sign ‘© Roots are rational <> D isa perfect square. * Roots are irrational <= D is positive but not a perfect square. © Ifatb+c=0, then 1, © are the roots of theequation act br+e 2 '=0, then the roots are ~ 1 and ~£ © Ifar? + bx +c=(is satisfied by more than two values, itisan identity and a= b= ¢=0 and vice-versa © Ifax? + bx +.c=0, where a, 6, c © R, has one root +g, then the other root will be p — ig. Hence, the imaginary roots occur in conjugate pair. © Ifac’ +bx+e=), where a, b,c are rational, has one root p+ Jd; then the other root will be p— Va Hence, irrational roots occur in conjugate pair if the coefficients are rational ‘© The quatratic equation whose roots are reciprocals of the roots of ax? + bx +c=0 is cx*+ bx+a=0 (i.e. the coefficients are written in reverse order). ‘* Ifa=1,b,ce Zand the roots are rational numbers, then these roots must be integers. © The condition that the roots of the equation a+ bet c= 0 may beimihe mon: ais mn 0 =ac (m+n). 4 Ifsum of roots ofa + br +0=0 is equal tothe sum of ‘their reciprocals, then ab?, bc”, ca* are in A. P. or b SegyS arein HP andifa—b+ aa You have either reached a page that is unavailable for viewing or reached your viewing limit for this book. aa You have either reached a page that is unavailable for viewing or reached your viewing limit for this book. aa You have either reached a page that is unavailable for viewing or reached your viewing limit for this book. Cubic! ‘Equation If a, B, yare roots of the cubic equation ax} + bx’ + cx +d=0, then, ’ o,=a+B+y a cor B + esapy=—-£ Biquadratic Equation If a, f, 7, 6 are roots of the biquadratic equation ax* + bx? + ex’ + dx + e=0, then, % Ba y+ 8 = oBiy+ of + a+ By 3 api5=*. Formation of a Polynomial Equation from Given Roots Tay dy Oy. rarethe roots ofa polynomial equation of degree n, then the equation is Pao"! +0," ? oy 3+. IYG, <0 where 6, =F 0404. Particular Cases Quadratic Equation Ifa, Bare the roots of a quadratic equation, then the equation is.x°— (a+ Pix + a= 0. CubicEquation Ifa, f, yarethe roots ofa cubic equation, then the equation is #-9,7 + ox-0,=0 or, (art B+ yx" + (B+ ary+ By) oBy=0. Biquadratic Equation If @ B, 7, are the roots of a biquadratic equation, then the equation is xto,x34 op? a,x + j= 0 . or, x4—(at B+ y+ Bx? + (Aft cyt a5 + Pry + B5+ yy? — (OB y+ OB5+ 078 + BB+ oS Sign of a Polynomial Expression Step 1: Factorize the given polynomial expression es FQ)~ (x= a)" (x 0)" (x05) (xa)! =a, )" where yy hyo ky € N and i, Oy, Oy, Ob, € R(Qy << Uh. < 0, Plot the points c, c, 64... @, on the rea line. Mark plus sign in the interval of the right of the largest of these numbers ie., on the right of o,, eee ae Quadratic Equations and Expressions 115 Step 4: ‘If, iseven, put ‘+’ sign ofthe left of a, and if, is odd, put “sign on the leftofo,,. ‘Consider the next interval and put signin it using the above rule. Thus, consider all the intorvas. Step 5: Step 6: The solution of f(x) > 0 is the union of all the intervals in which there is a “+ sign and the solution of F(@) <0 is the union of all the intervals in which there isa‘ sign For example, consider the polynomial expression 8 Fesy~ee+ 29 49" (x— 3) 6-4” Thus, f(x) > Dif re (~2», ~au(- owl) Rational Algebraic Expression and, /@)ker=x<-korx>k aa You have either reached a page that is unavailable for viewing or reached your viewing limit for this book. aa You have either reached a page that is unavailable for viewing or reached your viewing limit for this book. aa You have either reached a page that is unavailable for viewing or reached your viewing limit for this book. 45. 46. 47. 48, 4. si. 52. Let 5 denote the set of all values of S for which the equation 22-22a+ Ix ta(ati)=0 hhas one root less than a and other root greater than a, then S equals @ ©.) L a> ° ( 3) Solution of 2°+2*!22 V2 is @ Comlog, 2+ (b) (0,8) © (Fone -») ©) C10) (@) none of these @ Clog, of-nu[}~) 22+ 2bx + 2c? and g(x)=~ 37 ~2ex + O such in.f()> max. (x), then the relation between b and cis (@ jel<|b| v2 0) O\o\v2 If the roots of the equation Ptaxra +a-3=0 are real less than 3, then: (@) a<2 () 2sas3 (© 3-8r (@ 4pq~8r The solution set of| +1 +ix+1| (@) tx]220) ©) flaroU © EL @ (&lx2lors~1) If @, Bare the roots of the equation ax? + bx + (a#0)and ot 6, B+ Sare the roots of Ar? + Bx +C 0, (A#0) forsome constant 6 then @ {d) none of these Leta, bebe real, ifax’ + bx + e=Ohas two real roots and B, where o<—1 and B> 1 then 1+ 33. 56. ST. 58. 59. 60. Quadratic Equations and Expres ne @ <0 © <0 The roots of the equation (3 — x)* +(2—x)* = (5-2x)* are (@) tworealandtwo imaginary (b) all imaginary (©) allzeal (@) none of the above Ifa, d, caren G. P, then the equations ax? 4 hy + e= Oand de? +2er+f=0 (b) >0 (@)_ none of these. have a common root if @) HP © AP. Ifthe equations s? + aby + ¢=0 and? + acx +5 have a common root, then their other roots satisfy ‘the equation @ Prabtox+@be=0 (b) P-alb+ ox athe (©) P-a(b+dx-abe=0 (@) nore of these If (ae + bx + oy + a’? + bx > c'= 0, then the condition that x may be a rational function of y is (a) (ac!= ac)? = (ab!—a" ) (be! - bre) (b) (@o'=a" (c) (be'—b" (@) none of these ©) GR (@) none of these If n and r are positive integers such that 0 8+ 1)r+1=0 Since x is real, s0(k+ 1-420 => +2k-3205(k+3)(k-1)20 = kS-3o0rk21. Therefore, number of solutions is infinite. 412 (a). Since the given equation has imaginary roots = D (a-btoP<4ac = -Wacb = (Vatve)>o or Va+Vve>vo, Similarly, Jb + Ve > Ja and Ja +b > Ve. Therefore, Va, Vb, Ye cam be the sides of triangle. We have, Bt at b= (xx) (e— ty) 0 (4) xitartd *% 3 Ga) e-m)-G-4)= > &-)G)-5) pat oeth, 1-%,)= lim 22 mK sail te 20 (a). Since the roots of the given equation are real, 5 B-44C20= 407-4 (a+ a~3)20 > -at3200ras3, A) Since the root is less than 3, s0,f(3)> 0 => 82a) +a +a-3>0 => a-Sa+6>0or(a—2)(a~3)>0 > a<2ora>3 2) From (1)and (2), wehavea<2, arte 24 (b). Letz= : tata = rtatetelorte = wi2totrtz—1= = PG-N+xetl)+e-D=0 Quadratic Equations end Expressions 14.17 Forrealx, B’-44C20 > GrIF-4@-Ne-Hz0 ee 4 2e+1—427 + 82-430 = +374102-320-32 + 9242-3 20 => 2 (2-3)+1G-3)20 = @-3 C34 1)20-9 45783, misimum valueofs=1, x2 -26. 224). Given, L.< % = 28+4 Se 3 forallxe R 30x? 42x44 7 <3 forallxe R 0) Then, ye R forallxe R. 932 +63 44 BA 429 EG Tos ee PREG ay tayrd Cy? =2y44 From(1y <2 +2044 3° P= 2pe4 P4644 363 +4 23(a). Le f= +(1-2) r+ P-k-2 ‘Thenumber3 liesbetweenthe ootsof the given equation, iff@)<0 Now, f(3)=9+(1-2k)3+R-k-2 =10-7k+ =P -Tk+ 10 Hence, ((3)<0 9 2—7k-+10<0 $9 (K-2)(k-5) <0320, we geta<0~ Also, B=—b—a<—a=| a Thus, a<0 HDG) =051= 2 nie ole ole = log, s= xa at, 8 7 (b,c). Whenx?+4x+320ie.x2-1ores-3 Then,|.7+4e43 [+2c45=0 = eae 434204520 => x+6xrt+8=0 = Thus.x=-4asxe (1x2-1}U fr: xS-3) Whenx?+4x+3 -Gt+ar+3)+2r+5=0 thet 3 2x5 = Ome? + 2e-2=0 =14 JaeR > Se > ‘Vii because -14J/3 does not lie between~3 and 1 1-V3. Hence, we have either x=—d orx = 8 (a, b,c). The roots of the equation 10x? ex? - $4x—27 = Oare in H.P. Putting x= Up, weget Dy! + S4y? + ey 10 = 0 has roots in A. P. Let the roots of the equation in » be a fia, a+f.Then, sumof roots= or-B+or+a+ B= 2-28 =2 ss c= satisfies the equation Bey 4 2e 278454426 on0 ons, a7 Ag yom emt. Product of the roots = (af) (ai) (a+ B)= a or P=1 = B=21 Quadratic Equations and Expressions - 11.25 3 Roots of y-equation ae ~3, 1 33 or roots of x-ecuation ae -2, -2,3, 4 eau Boy 9 (a,c). Given equation is x7 + 977 — 4r + AD or, 4+ 7 +3=0, Since visreal, -. (-4¥-4(974+3)20 3 16-497 +3)20=4-9)7-320 => 97-150 (3y-1)Gy+1)s0 1 = F8955. Equation (1) can also be written as Oy? + Opt xd 3=0 Since yisreal -. 49 (x'-4r+3}20 = Madx+350 > @-1)6-3)s0> 18x83. 10(a, ¢). We have, a+ p= and, ots B= The given equation be -Aacle + 271+ Pm =O hasdiscriminant D= 16? AP 4a? (281+ a m) BaP 1? Jali 2{22_m om aha gt wier(-m bo [aeons] Hence, the roots are real, Also, we have, 27t+a’m at product of the rocts = =-(@- <0 The roots are of opposite signs. 12 (a,b, c,d). Given, a> 5>e 0) The given equation is (at b- 20 + (6 + e-Za}x+ (C+ a2) =O n(2) Since 2)has a root in the interval (1,0). FASO) <0 > Qa-be(c+a-2b)<0 8) From(1),a>b=2a~b> Vand a>e=>a-e>0 2a-b-e>0 oA) aa You have either reached a page that is unavailable for viewing or reached your viewing limit for this book. aa You have either reached a page that is unavailable for viewing or reached your viewing limit for this book. aa You have either reached a page that is unavailable for viewing or reached your viewing limit for this book. 8(a). Letf(x)= (at I?—3ar+ daand ts of the equation/(x)=0. The equation wills:.< ser ‘than 1iff (Dis 20 Gia+A>2 — tiiya+ DFY>0 Now, Disc. 2 0 =» 9a 16a (a+ 1)20=9—7a?- 16a20 16 = (ari <03- Me geo @) 7 3a ba a+ f>2= > 0» 2-250 jo Sy So = a<-lora>? «i and, (a+ 1)f(1)>0. => (at I) (as l-3a+4a)> 05 (at I) Qa+1)>0 = a-12 ti) From (9, (ii)and ii), we get: oP sa<-tie,ae -167,-1) 6 (b). Letf(x)=x7 +2 (a—3)x+9. If6 lies between the roots of f(x) = 0, then wemust have the following ()Disc>0,and (ii) J(6)<0(* coetf.of: Now, Dise>0=4(a-3)°-36>0=(a-3°-9>0 = @-6a>0 = a(a-6)>03a6 “ and, f(6)<0 = 36+ 12(a-3)+9<0 is positive). = Rat9<0=a<- 2 il) From (i)and (ii), we get; a<~3/4 ie.,€ (-©2,~3/4) 7 (b). Let/(x)= (I-a?)x? +2ar—1. Then, f(x) = between Oand 1 if (Dise>0and (II —a2)/(0)> Oand (1-7) f(1)>0 Now, Disc 20 = 4a’ +4(1-a")> 0, which is always true, (1-@)f()>0>-(1-a)>0 = @-1>0=>a<-lora>] wi) has roots 1 (a). Weknow that if a Bare roots of the equation AS + Bet then, a-B- ‘Equating the value of a from both the given equations, we get Ib? - 40 = fc? — 46 38 -4e= 2-45 Quadratic Equations and Expressions 11.29 and, (1-a?) (1) >0 => 1-a) (2a-a*)>0 = a(a-1)(a+1)(a-2)>0 =} as-lora>2or02. 8 (d). Let f(a) =x? —4ax+2a?-3a+ 5. The conditions for both the roots to exceed 2 ere (Dise.20 (ii)f(2)>Oand (iii) sum of the roois> 4 Solving these three conitions, we get a> 2. Hence, 9 (a). When.x> 0, P, (x) >Oand so P, (x) =0 can have no positive real roots Now, P, (x)= 1+ 2432+... (+1) x" SPQ) ant DEH at nen It! = 2p, Q)s tte tte (rt et Leet aie + nt (xy For negative values of x, P, (x) will vanish whenever Fl) =1-(142)x"" 14 (ne x2 =0 Now fea 1142) DED = P,Q) Ifm iseven, there isno change of sign in this expression and so there is no negative real root of f(x). 10 (c) and 11(a). As discussed in the above problem, ifn is oda, there is one change of sign therefore f(x) can have at most one negative real root. In this case f(-I)=-2n-2<0,f0)= 120 So, the negative real root lies between —1 and 0, 12 (a). Let/ix) = (e—a,) (05) (a9) +3Gra,) (xa) (ra) Asx =f) = Hay) = 3, -4)(ay— 49) (2 -g)< 9 Similarly, Flay)? 0,flas)> 0,flay)<0,f(a5)<0,f(ag)> 0. Thus, /(2)changes sign in each of two intervals (a, a3), (a, 44) and (ay a). Since f(3)=0is acubie root in, <. twill have I root in each of the above sub-intervals. 4(b-c)=9 (6-0) (b+ +4) =0 (+ bee) 2(d). Let/(x)=x +2 (k+1) x+9k-5, Let ce Bethe roots off (8) = 0. Tne equation f(x) = 0 will have both negative roots if and only if (i Dise.20 (i) a+ <0 and (ii) f(0)>0 ‘Now, discriminant 20 aa You have either reached a page that is unavailable for viewing or reached your viewing limit for this book. aa You have either reached a page that is unavailable for viewing or reached your viewing limit for this book. aa You have either reached a page that is unavailable for viewing or reached your viewing limit for this book. Permuta Combinatio REVIEW OF CONCEPTS ‘We often come across questions such as the following: 1. Inhowmany ways can four items be arranged in a row? 2. Inhow many ways can five boys be seated at a round, table? 3. Inhowmany ways can a group of five ladies be selected out of a gathering of ten ladies? 4. Inhow many ways can 5 pencils be selected out of 8 and displayed in a row? ‘Answers to these questions and many other important and more difficult ones can often be given without actually writing down all the different possibilities. In the present spter we shall study some basic principles of the art of counting without counting which will eneble us to answer such questions in an elegant manner. Factorial Notation Inorder to express the product of consecutive integers, itis convenient to use the following notation. ‘The product I - 2 3.... is expressed a6 lor lm {pronounced as m factorial or factorial n) It is easy to see that 4=24 and soon. IMPORTANT POINT(S) TO NOTE «We know tha Thus, if m id mr nth be expressed Explanation Qn)! = Qn) Qn—1) Qn—2) 43-21 ((2n) (2-2) (2n~4) 4-2) [Gn Qn—3)...3-1] = [2 nln=1)(n=2) 2 I] [1-3-5..@n-D] Qnty] GRRE caution GR ‘© The factorial is defined only for whole numbers We do not define the factorial of proper fractions or negative integers. © Qn)! #2(m! © (tml em! +n! + cmtemint =n [1-3-5 Fundamental Principles of Counting Multiplication Principle If an operation can be ‘performed in 'm' different ways: following which a second ‘operation can be performed in ‘n’ different ways, then the to operations in succession can be performed in m * n different ways. Illustration: ‘Anu wishes to buy a birthday card for her brother Manu and send it by post. Five different types of cards are available at aa You have either reached a page that is unavailable for viewing or reached your viewing limit for this book. aa You have either reached a page that is unavailable for viewing or reached your viewing limit for this book. aa You have either reached a page that is unavailable for viewing or reached your viewing limit for this book. 18, (a) (O Moutof(p+g+r+ things, pare alike of one kind, qare alike of second kind, are alike of third kind and 1 re different, then the total number of slections is FD G++) Z-1. (@) The number of ways of selecting some or all out of p+ 47r items where p are alike of one kind, q are alike of second kind and rest are alike of third kind islo+D@+He+DI-1. Division into Groups 17. @ Number of ways of dividing m + » different things in two groups containing m and n things respectively (ms mis _ (m+n)! (m+n)! ‘mporant and nl groups is important Namber of ways of dividing m + » +p different thingsin three groups containing m, mand things respectively (m#n#p)is (m+n p)! nc, ifthe order ofthe groups is not x 21, ifthe order of the © ifthe order of the groups is not mala pt (m+n+ p)! mint pl ‘groups is important (© Namber of ways of diviéing 2m different things in two groups, each containing m things and the order (2a)! 24m)? ‘Number of ways of dividing 2m different things in ‘wo groups. each containing m things and the order important and x31 iftheorder ofthe of the groups is not important, is © ‘ofthe groups is important, is (27)! cay? (© Number of ways of dividing 3m different things in three groups, each containing m things and the ‘order of the groups is not importent, is an, 31m!) (Q) Number of ways of dividing 3m diferent things in three groups, each containing m things and the s (Gnu order of the groups is important, is (my? Number of ways of dividing » identical things into ‘r groups, if blank groups are allowed is mle (©) Number of ways of dividing midentical things into - ‘groups if blank groups are not allowedis”“'C,_. (©) Neunber of ways of dividing » identical things into ‘r groups such that no group contains less than we 20, 2, 2. 2. 24. 2. Permutations and Combinations 12.5 things and more than k(m <#) things is coefficient fx" in the expansion of (tame tat ye ‘The number of ways of selecting r things out om things ‘of which p are of one type, q of second type, s of third type and s0 on, = coefficient of in [tetas te? )(dtettt tes (txt 2+ t2).] ‘The mumber of ways of selecting r things out ofm things of which pare alike and are of one kind, arealike and are of second, are alike and are of third kind and $0 0n, is = coefficient of xin [Ut atatt taf) tx batt tx) (4ete tte). ‘The number of ways of selecting r things out ofm things of which pare ike and are of one kind, q arealike andare of second kind and rest (n—p—q) things are all different is = coefficient of xin [Qtxtx2 +. tx?) (txtatt tat) x (taye-g ‘The number of ways of selecting r things out of things of which pare alikeandare of one kind, g are alike and are ofsecond kind, are alike and are of third kind when each thing is taken at least once = coefficient of x7~3 in [d4xtP+ +P) (itrte+. tet Ix (4xt2+ te )] ‘The number of ways in which r identical things can be distributed among » persons when each person can get zero of more things coefficient ofx” in(I +x422+..+x")" coefficient of in(1 =ay"="*""'C, The number of non-negative integral solutions of the equation x; +x, +..+x,=nis"'"'C,, The number of terms in the expansion of (@, +a; +054. +4,F'is"*7"'C,. Derangement Rearrangement of objects such that no one goes to its original place is called derangement. f'n’ thingsare arranged ina row, the number of ways in which they can be deranged so that none of them occupies its original place is, nit et ae aa You have either reached a page that is unavailable for viewing or reached your viewing limit for this book. aa You have either reached a page that is unavailable for viewing or reached your viewing limit for this book. aa You have either reached a page that is unavailable for viewing or reached your viewing limit for this book. 34. 35. 36. 31. 38. 39, 40. 41. a Ina cortain test there are 1 questions. In this tost 2 students gave wrong answers to at least (n ~ &) questions, where: .n. If the total numberof ‘wrong answers is 4095, then value of 7 is @ i 6) 12 © 3 @ 15 ‘The number of permutations of the letters a, b, ¢ d such that} does not follow a, ¢ Goes not follow 4, und d does not follow c, is @ 2 © © B ul its= Se, .then oy 54°C, & SHC, (@) none of these Ifthe numberof ways in which a different things ean be distributed among i persons so that at least one person does not get any thing is 232. Then, 1m is equal to @ 3 4 (©) $ (@) none of these The sum of the factors of 8! which are odd and are of the form 3m-+2, where m isa natural number is @ 0 & 45 © 0 @ none of these itm umber of distinct rational numbers : €@1) such that p, g€ {1 2,3, 4, 5} and m= number of mappings from {1,2,3} onto {1,2},thenm —mris @ 1 ® -1 oo (@) none of these Let ybe an element of the set = (7,2, 3,5,6,10, 15, 30} and x, xp, x, be integers such that xXx, 9, then the nuriber of positive integral solutions of x,%—¥5=¥ is @ @ © 8 The number of subsets of the set A = {4,5 .» d,} (e) 2 (@) none of these which contain even number of elements is, fa) 2"-1 2! fe) 2 2? Given 5 different groen dyes, 4 different blue dyes and 3 different red dyes, the number of combinations of dyes that can be chosen by taking a! least one green and one blue dye is (a) 248 (©) 3720 & 120 @ 4 4. 44, 46. 41. 48. a. 51. 52, Permutations and Combinations 12.9 Number of points having position vector ai +hj-+ck, where a b, ce {1,2,3,4,5} such that 2443? +56 is divisible by 4is (e) 40 ) © 1 (@) none of these 2"C (0. Sr'S2n) is greatest when r is equal to n nat @ > o> (ran @ none of these The number of even numbers greater than 100 that can be formed by the digits 0, 1, 2,3 (no digit being, repeated) is (a) 20 () 30 © 40 (4) none of these ‘The number of positive numbers less than 1000 and divisible by 5 (no digit being repeated) is (a) 150 (b) 14 © 166 (@) none of these Inacertain city all telephone numbers have six dig its; the first two digits always being 41 or 42 0r 46 or 62 or 64. The number of telephone numbers having all six digits distinct is (@) 8400 (© 9200 (b) 7200 (@) none of these The total number of ways of selecting five letters from the letters of the word INDEPENDENT, is (@) 4200 (6) 3320 (©) 3840 (@) none of these ‘Thesum of five digit numbers which can be formed with the digits 3,4, 5,6, 7 using each digit omly once in each arrangement, is (a) 5666600 (©) 7666600 The sum of all the numbers that can be formed by writing ail the digits 3,2,3, 4 only onceis, fa) 3996 (6) 49996 (©) 57716 (@) none of these Thesumof all numbers greater than 10000 formed by using the digits 1,3, 5,7, 9,no digit being repeated in any number, is (@) 4666600 (©) 6666600 The sum of all numbers greater than 1000 formed by using the digits 0, 1, 2,3, no digit being repeated in any number, is (@) 38664 (©) $8664 (0) 6666600 (@)_ none of these (>) 5666600 (@) none of these (0) 48664 (4) none of these aa You have either reached a page that is unavailable for viewing or reached your viewing limit for this book. aa You have either reached a page that is unavailable for viewing or reached your viewing limit for this book. aa You have either reached a page that is unavailable for viewing or reached your viewing limit for this book. Mathematics for IT—JEE 1. 1 (b), He (0, ho @), Vo @) 3. 1 (de (b), Ie (@), Veo ©) Total number of wrong answers =1+(@,-a))+2(@)—a)) +O D(Qy_ a) hay maytag tayt nt ay 2(4). Thenumberof ways inwhich 13 guests may be divided into groups of 8 and $= °C, = rT ‘Now, corresponding to ore such group, the 8 guests may be seated at one round tablein (81)! ie, 7! ways and the five guests at the other tablein ($1)! i.e, 4! ways. But each way of arranging the fist group of 8 persons can be associated with each way of arranging the second sroup of 5, therefore, the two processes can be performed together in 71> 4! ways, Hence, required number of arrangements 1B 131 xTIx ri Ag co 6 (a). Each stall can be filled in 3 ways as there ere three types of animals (animals of each category being not less than 10). Shiplos 3x3 x...upto 10times 2. fillingup of 10 stalls, can be madein 159049, 12 (a). Let the number of papers be n. Total number of ways to fail or pass MCytPC\#"C+ #9C, 22" But there is only one way 10 pass i. innone. Total number of ways to fail=2"—1. . when he fails From question, 2" —1 = 63; +. 2"=64=26 =6. 14 (c). Since the elements in 4 are in A.P,, therefore, the 109: number of elements in d= 41=274, Since, sum of equidistantterms in A.P. is equal to thesum of first and last term = 1+ 1093 = 1094. => maximumnumber of elements in A, that add up to 1094 os 2 7. 18(a). £ = NC, IG, édisi210 HNC yx Cyt MC, (Cyt 1C,)+ "°C, CCy+7C, +7C,) +P CG HE HEE). $39 (189 +196, +t CQ) 210¢. 20.196, 21 4M¢, 224. 41% 210 =(42=3i, 49 (6). Let 4, B be the corresponding speakers. Without any restriction the eight persons can be arranged among themselves in 8! ways, but the number of ways in which speaks before B and the numberof ways in which B speaks before A makeup 8! Also, number of ways in which 4 speaks before is exactly same as the number of ways in which B speaks before A. -. the required numberof ways = 3 (= 20160. aa You have either reached a page that is unavailable for viewing or reached your viewing limit for this book. aa You have either reached a page that is unavailable for viewing or reached your viewing limit for this book. aa You have either reached a page that is unavailable for viewing or reached your viewing limit for this book. 20(¢). ‘Total number of permutations = 7! Let be the property that “beg” occurs. Be the property that cad oceurs. ‘Number of permutations with 4=5! = that of with B and 1 (4 0B) (AU B)= 51+ S!~31= 234 Required number = 7! -234= 4806 24 (c). ‘Thetotal mumber of numbers that can be formed with thedigits2, 3,4, 5 taken allatatime=4 24. Consider the digit in the unit's place in all these numbers. Each of the digits 2,3, 4,5 occurs in 3! = 6 times in the unit's place 2+ total forthe digits in the unit's place =Q+3+4+5)6=84 Since each ofthe digits2, 3,4, S occurs 6 times in any one of the remaining places, therefore, the required total = 84 (1+ 10+ 107 + 109) = 84 (1111) =93324. 22(c). Number of triangles = "C,-—"C,-"C, _ 2 Qn-)Qn=2)_2n(r— (m2) 6 6 =4n@- Gn = 0, 23(d). EHMOR T alphabetical order) Number of words beginning x Seocogm Sawer HE R=1! Rank of word MOTHER = 308 2A (a). Factorizing the given number, we have 38808 =23 37-11 Therefore, the total number of divisors B41) Q+I)N+1)- But this includes the division by the murnber itself. Hence, the required number of divisors =71-1=70, 26 (b). Other than 2, remaining five places are to be filled. byl and3 ‘5. umber of ways for five places 2x2«242¥2=28 For 2, selecting 2 places out of 7 = 7C, Required numberof ways =7C; - os 42.47 Permutations and Combinations 27 (¢). The required number of circles =(C,-4¢) +1 = 117 28 (d). Number of subsets antic, = BIC, +... + 2"1C, = N (say) 29 (a). Starting with the leter 4 and arranging the other four letters, there are 4! = 24 words. These are the first 24 words. Then, starting with G and arranging 4, 4, /and Nin 4! different ways, there are =; = 12 words, the 37th word starts with f and there are 12 words stating with 1, Total 202s up to.48 words. Now, 49th word isNAAGL 30 (a). Ifwe choose & (0S k's) identical objects, then we rust choose (7 — k) distinct objects. This can be done in 2m'Ic , ways. Thus, the required number of weys i =2, 31a). The number of subsets of 4 containing exactly three elements is "C, whereas the number of three subsets of 4 that contain a, is""!C,. We are given, 20 mie, 2 om Co” 9g CD (n=) Also, for 67 ] 1648444241831. Hence, the exponent of 2 in 331 i831, Largest value of m is 31. 159 (b). Number of non-negative integral solutions of the given equation += coefficient of x7? in G-ytas td tty = coefficient of x” in (1x)? (1 =x4) = coefficient of x in (1+ 3C,x + 4C,x7+5C, x79 +°C, rn, ~ + C8 + MCyy ale NCygelO s+ BCyy x24.) x(lexted ex? 44204.) 1480+ PC EMC, + BC+ 2Cyo = INC, + NCH MC, HEEB, ~(a} GaP Ar Ge) = 1415445491 +153 +231 =536, 60 (a). Let r consecutive positive integers be mm | m2, 0 (+r L,where me N Product = (m+ 1}(m+2)..(m+r-1) (m= Dt (m+ 1) (n+2)...dmet r= 1) (m= _(mtr-Dt | q (ma elm mrnic which is divisible by ( n° 16. ig natural number) 641 (c). The number of triplets of positive integers which are solutions of x +y-+2= 100 = Coefficient of x! in (x+x? +38 + Coetticient of x! in.x?(1-s)” = Coefficient of x! 3 Permutations and Combinations 12.21 fe Eee or, ] _ OT#D (9742, 2 62 (a). There are eleven letters in the word ‘Pataliputra’ and there are two p's, two 7°s, three a’s and four other different letter. Number of consonants = 6, number of vowels = 5. Since relative order of the vowels and consonants re- mains unchanged, therefore, vowels will oxcupy only vowels place and consonants will occupy only consonants's place. Now, 6 consonants can be arranged among themselves in 6 Fray *A¥8 since there are two p'sand wo rs] 49 *99= 4851, 31 and five vowels can be arranged among themselves in = ways, since a occurs thrice. ast 2213! + Required number 63 (6). Let be the number of students. ‘Now, number of ways in which two students can be selected out of m students is "C,. #- mumber of pairs of students ="C, But for each peir of students, number of cards sent is 2 (since if there aretwostudents 4 and B, 4 will send a card to Band B will send acard to 4). ©. For"C,pairs, number of cards sent = 2." According to the question, 2"C; = 600 % 2) 2 = 600 or,n?-n—600=0 a, (n= 25)(n +24)=0 +. m=25,-24 om CG) eC) (2+) Ca} )-() eae) nw) DoS (2)-(a} free ‘ ae aa You have either reached a page that is unavailable for viewing or reached your viewing limit for this book. aa You have either reached a page that is unavailable for viewing or reached your viewing limit for this book. aa You have either reached a page that is unavailable for viewing or reached your viewing limit for this book. 6 (a,b). Given numbers are 1,2,3, Let the three selected numbers in A.P. be a, 6, ¢, then ate orate=2h oA) From (1) tis clear that a +c should be an even integer. This is possible only when both a and c are odd or both areeven. Case. When niseven. Let n= 2m ‘The number of odd numbers and number of even numbers =m 4s mamber of selections of a and ¢ from m od numbers = "C, umber of selections of @ and e from m even numbers ="c, umber of ways in this case =2-"C)= mi (m—1) =4(4-1)- n(a-2) “22 4 Case ML. When mis odd. Letn=2m+ | Then, number of odd numbers = m +1 and number of even numbe :. Requited number in this ease="*"C,+"C leapt =qt yy. 7 (a, 6, d). Two straight lines can intersect in at mest one point, Therefore, maximum mumber of points of intersection of mines ="C, *1="C; ‘Again, two circles can intersect in at most two points. :- Maximum number of points of intersection of circles HNC, X2= PC, A line can intersect a circle in at most 2 points. . Maximum number of points of intersection of m lines and circles= (mn)2=2mn NAMC42"C, + 2mn "C,4"P,+ 2m ® "p, +2mn nn—1)+2mn= n(n—1+ 2m) = mi(N-"G)- ()N"P,="Cy+2mn =D dn fe] if mis even, then (m—1) isnot divisible by 2. in general m does not divide (V—nP,). 12.25 @N-"C,~'P, (a), (¢), (@) hold. ntN= (n+ 1) (0+2)..2n) Now, pi(t* 1) (+ 2)..(2n)on prin! N= Pint 1)..(p-l)p(pt V)...(2n) > pln Neu(P-1) (P+ Vow(2m) Thisisimpossible. -. p? does not divide N, + (@)s(@) hold, 9 (a,b). Leta, B,Cbe the objectsselectedand x, x),%3.%, bethe number of objects on either side of theselected objexts as shown below OBO We have, aytytaytyee3 where x). 2 0andx,,x,2 1 Hence, the required number of ways ind teats, tots" S in (lent 4, = coefficientof.x"”$ in (I-x)* ErSHIC emig amt = Cus Cage? Cy 10(b, a). Wekave,r-120,rsn+1. Lor Isrsntl ——s—— 2 n+l nel rit Gay 8s1 > 8<—esseso mi => 859 = ~38k<-2V2 o2 V2 ke [d-2v2)orke @ V2.3] 14 (a, b). Let xs be such that xy +23 +3 +24 +5 =m, ‘We now seek the non-negative integral solution of x; +3 + 2x3 tay +5=n. The number of required solutions ="*4C, arte aa You have either reached a page that is unavailable for viewing or reached your viewing limit for this book. aa You have either reached a page that is unavailable for viewing or reached your viewing limit for this book. aa You have either reached a page that is unavailable for viewing or reached your viewing limit for this book. 6 (a). Required number of ways = 47 >> ‘Gy a5 7 (a). Since pots are identical, there will be 4 cases (4,0, 0), (3, 1,0),(2, 2, 0) and (2,1, 1) but since all coins are different hence selection of coins matters Therefore, forthe first case number of selections = For the second case number of selections =tG IC =4 For the third case number of selections Wt x2e — For the fourth case number of selections, Se x?qx'q _, 2 Hence, the total number of distributions $4434 6= 14 8 (b). Since no box is empty end all pots are identical, so the possible case is (1, 1, 2). But since all the coins are different, the 2 balls can be selected in *C, ways and rest can, 2Gx'o, be put in >" ways. Hence, the required numberof distribution 40, x20 x! exqx'G _, 2 9 (b). Since no pot is empty and all coins are identical the possible caseis (I, 1,2). Butsince all three pots are different hhence, a pot (which contains 2 coins together) can be selected in?C, ways. Hence, the required number of distribution ="¢,x123 1 10 (c). The possible arrangements are as follows: (4,0,0) + Can bedone in 2 ways, i. 4 balls cam be put ineither one of the two identical pots or can bbe putin different pot. G,1,0) + Can be done in 3 ways, either distinet pot can be filled up with 3 balls, of I ball or remained empty. (2,2,0) + Can be done in 2 ways, either distinet pot ccan be filled by 2 balls or remained empty Q,1,1) + Canbedone in2 ways either distinet pot can befilled up with 1 ball or2 bal Hence, the required number of ways =2+3+2+2=9 11 (¢). The possible arrangements are as follows (4,0,0) + Can bedone in 3 ways, letthe 4 coins be 4,4, Band C then all these 4 coins as a single Permutations and Combinations 12.29 packetof coinscan be arranged in3 different pots in 3 ways (3.1.0) > Can be done in 18 ways, out of 4, Band Cwe ccan select one coin in 3 waysi.e..either 4 or Bor. Now, we can arrange this selected coin in any 3 pots in3 ways and the remaining 3 coins as.a single packet of coins can be arranged in remaining 2 pots in 2 ways. Hence, the required number of ways=3 «3 *2=18 (2,2,0) > Can be done in 12 ways ‘There are two possible cases: 0 AMBO @) (4,840 Ineach oftwocases we assume that there are two packets ofcoins which can be arranged in 3 x 2= 6 ways. Since there are two cases, hence the total number of required way 12 21,1) > Can be done in 21 ways There are 4 possible cases 0 UAB i) UB.A.O iii) (4, ©), (4), (B) (iv) (B,O),(4), (A) For the first 3 cases in each case all the 3 packets of coins can be erranged in 3! ways Hence, the numberof arrangements 3 3! =18 Now, in the fourth (iv) case two coins areidentical so the third packet of coins can be arranged in 3 pots in 3 ways. Hence, the total number of arrangements~ 184.3~21 Thus, the required number of ways =3+18+12+25-S4ways 12 (c). In the first round, total number of matches In the second round, total number of matches =2Q)=4 {In each group say I-11. 11-1V] ‘Number of semi-tinals = 2 Then, final = 1 Total number of matches = 56 +4+2+1=63 13(¢). Eaci team will play 7 matches and so any team can ‘winany number. Ifmetches between 0 t0 7 ie. (0, 1,2,3,4,5,6,7) Fourteams will beselested(7, 6,5,4) Thus, team which wins only 3 matches will be out of the firs: round, 14 (a). Clearly, minimum number of matches that a team ‘must win in order to qualify for second round is 4. 15(d). In second round, it has to win one match. inal andone Final Then, one in se *. total number of marches = 3 aa You have either reached a page that is unavailable for viewing or reached your viewing limit for this book. aa You have either reached a page that is unavailable for viewing or reached your viewing limit for this book. aa You have either reached a page that is unavailable for viewing or reached your viewing limit for this book. Probability of an Event The probability of an event is defined in the following two way A, Mathematical (or a prior’) definition 2. Statistical (or empirieal) definition, Mathematical Definition of Probability Probatilityof an event A, denoted as P(A), is defined as ‘Number of cases favourable to 4 yo —————— ‘Number of possible outcomes ‘Thus, if an event 4 can happen in m ways and fails (does rot happen) in m ways and each of m ~n ways is ‘equally likely to occur then the probability of happening of the event 4 (also called success of 4) is given by m men and the probability of non-occurrence of the event 4 (also called its failure) is given by Piay= n Ifthe probability of the happening of certain event is denoted by p and that of not happening by g, then a =I, P (not A)or P(A} +qe—+ Pan man Here, p, q are non-negative and cannot exceed unity, ie, 0SpSland0 and =rayash, probability of not happening of event 4 = P(A) = + Ifodds against happening of an event A ate a:b, then probability of happening of event 4 = P(A) = 4 and probability of not happening of event 4 Key Results on Probability 1. Ina random experiment, if Sis the sample space and Eis an event, then @ PE)20 (WAG)=0 CAS) Remarks: It follows from above results that @ probability of occurrence of an event is alweys non-negative; i) probability of cccurrence ofan impossible event is 0; (ii) probability of occurrence ofa sure event is 1. 2, It and F are rautually exclusive events, then @ PENF)=Oand @ PELA=PLE)+ PE). Explanation Let Sbe the sample space and let E and J” be two muta- ally exclusive events, Then, EM F= 4. n(9) 0 @) PEAR =He)= 20 om wes _MEUP)_n(El+n(F) @® PEA mS) nsy == (EU F)=n(E) + PY] nF = PE) PA. n(S)* n(3) Thus,P (EUR) =P(E)+P (P). aa You have either reached a page that is unavailable for viewing or reached your viewing limit for this book. aa You have either reached a page that is unavailable for viewing or reached your viewing limit for this book. aa You have either reached a page that is unavailable for viewing or reached your viewing limit for this book. aa You have either reached a page that is unavailable for viewing or reached your viewing limit for this book. aa You have either reached a page that is unavailable for viewing or reached your viewing limit for this book. aa You have either reached a page that is unavailable for viewing or reached your viewing limit for this book. aa You have either reached a page that is unavailable for viewing or reached your viewing limit for this book. 10. nM. 2 1B. 14, If E and F are the complementary events of events E and F respectively and if 0 PEVEEE) = Gy and 5) POE SE ExEyP,) = 32 sthen PLE) = © 91-92.93-94 91-92 -93-94-95 97-98 -99-100 96-97 -98-99-100 9495-96 © Soa (@ none of these A is one of 6 horses entered for a race and is to be ridden by one of two jockeys B and C, It's 2to | that 15, 16. 18, vb. 20. Probatilty 13.14 B rides 4 in whichcaseall the horsesare equally likely to win. If C rides 4, his chence of winning is rebled. ‘What are the odds against winning of 4? (@) 13:5 ©) $:13 () 18:5 @ 58 2° + ax? + br + 6, where o, b,c are chosen ly by throwing a die three times, then the probability that /(x) is an increasing function is @ (O) © @ Ina muhiple choice question there are four alternative answers, of which one or more are correct. A candidate ‘will get marks in the question only if he ticks all the correct answers. The candidate decides totickanswers at random. If he is allowed up to three chances to answer the question, the probability thet he will get ‘marks in the question is oF @ : © (@) none of these Four tickets marked 00, 01, 10 and 11, respectively, are placed ina bag. A ticket is drawn at random five times, being replaced each time, The probability that the sum of the numbers on the ticket is 15 is 3 5 © F028 © Toa 1 Om (@ none of these IfXand ¥ are the independent random variables for 1 1 sepa a5) anta(2 nrc 4095 309 ©) 4096 ©) co96 4032 © i006 (@ none of these A certainplayer, say X,is knowa to win with probability 0.3 ifthe track is fast and 0.4 ifthe track is slow. For Monday, there is a 0.7 probability of a fast track and 0.3 probability of a slow track. The probability that player X will win on Monday, is (@) 02 ©) an © 033 (@) one of these The sum of two positive quantities is equal to 2n. The protabilty that their product isnot ess than > times their greatest product is aa You have either reached a page that is unavailable for viewing or reached your viewing limit for this book. aa You have either reached a page that is unavailable for viewing or reached your viewing limit for this book. aa You have either reached a page that is unavailable for viewing or reached your viewing limit for this book. 57. 59. 61. 62. 55 44 ® 1024 ©) 1024 ( a (d) f the © Toa (2) none of these Plent I of XYZ manufacturing organization employs $ production and 3 maintenance foremen, another plant 11 of same organization employs 4 production and $ maintenance foremen, From any one of these plants, a single selection of two foremen is made. The probabil- ity that one of them would be production and the other maintenance foreman is 278 263 fa) S04 (b) 504 Ft 502 © Soa © 08 In acertain recruitment test there are multiple choice questions. There are 4 possible answers to each question and of which one is correct. An intelligent studentinows 90% of theanswer whilea weak student knows only 20%, If an intelligent student gets the correct answer, then the probability that he was auessing is 1 36 oF oF 14 © n (d) none of these ‘The sum of two postive quantities is equal to 2n. The 3 probability that iheirproductis no lessthan + times their greatest prodiict is oF © i 1 Oo) > (d) none of these X folows a binomial distibuion with parameters and p, and ¥ follows a binomial with parameters m and p. Then, if and Y are independent, Pit=r|X+Y=rts)= @) CMCC NC OY IMC, PMC © 2CEMIC WC... (A) none of these ‘Three numbers are selected at random without replace- ‘ment from the set of numbers {1,2,..4). The condi- tional probability that the third number lies between the first two, if the first number is known to be smaller than the second is @s w) 4 @ 5 © 5 1 © z Four positive integers are taken at random and are ‘multiplied together. Then, the probability thai the prod- ‘uct ends in an odd digit other than 5 is (@) none of these 63. 64, 6s. 66. 67. 68, Protabilty 13.15 3 6 @% © oe 16 2 © os OF Iffourwhole numbers taken at random are multiplied, together, the chance that the last digit in the product is1,3,7, 019s, 18 (@) © GE © Be (@) none of these ‘A pack of playing cards was found to contain only $1 cards. Ifthe firs 13 eards, which are examined, are all red, the probability thatthe missing card is black is 2 1 @ > 5 © ; (@) none of these A ten-digit number is-formed using the digits from zer0 tonine, every digit being used exactly once. The probability thatthe number is divisible by 4 is @ * 20 a OF 2 © (d) none of these al Esch coefficient of the equation ax? + bx + c= Ois determined by throwing an ordinary die. The prob- ability that the equation has non-real complex roots is 173 43 © ae © se SA @ (@) none of these 216 ‘Aset A contains » elements, A subset P of is chosen atrandom and the set is reconstructed by replacing, the elements of P. Another subset Q of 4 is now chosen at random. The probability that PUQ contains exactly r elements, 17'S nis @ » ot 1 © 3 ( . (d) none of these ‘A person throws two dice, one the common cube and the other a regular tetrahedron, the number on the lowest face being taken in the ease of tetrahedron, ‘The probability thatthe sum of thenumbers appearing onthe dice is 6 is, 1 1 fa) 3 (b) a ot (@) none of these aa You have either reached a page that is unavailable for viewing or reached your viewing limit for this book. aa You have either reached a page that is unavailable for viewing or reached your viewing limit for this book. aa You have either reached a page that is unavailable for viewing or reached your viewing limit for this book. ‘Twelve players 5}, Sp.» Sip play in a chess tournament. They are divided into six pairs at random, From each pai winner is decided, It is assumed that all players are of equal strength. The probability that 1. Both S, and S, are among the six winners is 12 13 @s oO 3 ot (@) none of these 3 2. Exactly one of S, and, is among the six winners is 6 5 On On 4 Oa (d) none of these 3. Atleast one of S, and S, is among the six winners is 31 32 On OFF 10 OT (@)_ none of these ‘A person draws a card from a pack of 52 cerds, He replaces the card, shuffles the pack and again draws a card, He replaces it and draws again, This he does until he draws a heart 4. The probability that he will have to make at least four draws is 27 175 @ x6 ©) 356 © 2 (d) none of these 64 5. The chance that he will fail in the first two draws is 1 3 og Od 9 1 © a (@) a [PASSAGE mt A shopkeeper inspected some itemsin a box containing a, b and c number of items of type P, Q and R, respectively. He picked up an item randomly and put it back into the box if he found that the item selected wasregularly sod, he also added the equal number of items of the same type as those previously inthe box otherwise he removed all the items of that type. Probability 13.19 6. Ifexactly one ofthe three items is regularly sold then ‘what is the probability that the item selected in the second draw is similar tothatin the first draw? [Given thatthe iter selected in the second drow isa regularly sold iter). @ ct @ By (6 ) a arhe Gabes 2a? 2 at 1 0 (Siete ele] @ » € 1 6 (ire aera mae (@ none of these 7. What is the probability that the item selected in the second draw is not a regularly sold item? @ tb ay t arbec atbec atbte w ote, ate, ate ) Gasbee’ ar2bve atb+2e ot bee (@) None of these 8. What is the probability that the items selected in the second draw is different from that in the first draw, given that the item selected in the first draw is a regularly sold item? (Assume exactly two of the three items ere not regularly sold items). @ (42 Haro ea 1 lasb+c* aeabec a+b+u) arbre abe), Kase) , asd) _1 © [Bae nae cool ae © 4,4 _,_<« _)_t Jasbte’ avdbre 'asbez ) arbre 0 PASSAGE IV ‘Two persons and B are playing with S dice. Player 4 throws 3 dice and player B throws 2 dice. The trials go on simultaneously and successively until 6 shows on at least one of the dice. 9. P(A): The probability that player A and not B first obtains 6 is 52 aa You have either reached a page that is unavailable for viewing or reached your viewing limit for this book. aa You have either reached a page that is unavailable for viewing or reached your viewing limit for this book. aa You have either reached a page that is unavailable for viewing or reached your viewing limit for this book. ‘24(b). A person can alight atany one of n floors. Therefore, ‘the number of ways in which m passengers can alight at floors is ™A No, ‘The number of ways in which all passengers can aight at different floors is"C,, % m Hence, required probability 25(a). We know 74, Le N, has 1, 3,9, 7at the unit's place for k=4p,4p—1, 4p-2,4p~3 respectively, where p= 1,23, Clearly, 7" + 7" will bedivisibleby Sif 7" has3 or in the unit's place and 7" has 7 or3 in the unit's place or 7” has 1 or 9 in the unit’s place and 7” has 9 or | in the unit's place. Forany choice of m, n the digitin the unit's place of 7+ 7Mis2,4,6,00r8.Itis divisible by 5 only when this digit iso. terested tity 26(a). Itisa case of Bernoullian tials with number of trials 1 ‘nand probability = => (a successin one tial) P(r successes) ="C,q"~! p"="C,(1-py""" p! ~s(-4) 6 ola)" 27(c). Let(B) denote the event that minimum (maximum) ‘number on the chosen ticketsis 3 (7). We have, P(A)= P (choosing 3 and two other numbers from 4to 10) 1c, 7x 6x37 We, “10x 9x8 40 ‘P(B)= P(choosing 7 and choosing two other numbers from 1106) Cy _ 6x5x3 1 %¢c, “lox 9x78 PUB) = P(choosing 3 and 7 and one other number fromd to6) 3 3e3K2 0c, 10x9x8 40 P(AUB)=P(A)+ PB)—P(AB)= 10/40. 29(b). The exhaustive casesare 4! ‘The favourable cases are a(t erobabity 13:23 9.3 therequited protabitty = >> = 3 30(c), Lettheorthocentre be (xy) (x28 +3 = (5-2) (5-3 = Pryde 6-0 => Part (y-3P-9=0 = x=22 40 ycamtake the values 1,2,3.4,5.6 Required probabitity= © = 3 equted probity = = = 3 34 (6). The total length of the interval in which a lies = 30-(-5)=35 Ifthe graph of y=12+2(a 4)x~5a-+64 isentirey above the x-axis, the discriminant ofthe sbove quadratic exoression ‘must be negative 4fa+4)+4(5a-64)<0 = a+ Ba-48<0 = (a+ 16)(a-3)<0. = -l6 0) mustlie so that the numerically greatest term in the expansion of (1 — 2)! has the fe 56 catest coefficient is, (3, ©), & * iG 5} Reason: If mis odd, then numerically greatest coefficient inthe expansion of (1 ~x)" "Ct og "Con ig S21 op Sant | 2 2 ‘Assertion Assertion Reason Assertion Reason ‘Assertion If mis even positive intezer, then the condition that the greatest term in the expansion of(1 +1)" may have the great- est coefMicient also is ne n42 For even positive integer. the greatest coefficient in the expansion of (1 +x)" is "Gyn. Sum of the infinite series on "Cyt NCy + The value of 4G. “G Tats n Cy 4, Lyn en i Lie 28!) Fordstsn"G= 2." May, ayy, are in ALP. and S, is the sum of first terms, then DGS, 2" (na, +5,) fo aa You have either reached a page that is unavailable for viewing or reached your viewing limit for this book. aa You have either reached a page that is unavailable for viewing or reached your viewing limit for this book. aa You have either reached a page that is unavailable for viewing or reached your viewing limit for this book. + Coefficient of x"in the expression is br. Femme - fares o 23/5 mig, ye2" ny “C42 | nl += (nt 1)" 28 (c). Letn=2m,meN ar btey'+(atb-c)"=[(at+b)+ cP" +fe+h)-g" = 24+ mr ear Dymres | +c en). | ‘Therefore, the number of distinct terms = Qt 1)+Qm=l)+ went ameten 2 (re aa 29(c). Consider, (cot — i sin6" ="C, cos -"C,cou"™ 10; sind = (mt P= +4 ™C, (isin — (1) (e056 +i sin)" = "Cycos"0+"C; cos" 16 sind +t MC (ising —..2) ‘Adding(1) and (2), we get eos mO=2"Ccos"O—"C,cos™2Asin?@..] (3) ‘Subtracting (1) from (2), we get | 2isin mO= 2i ["C,cos”™'@ sind—"C,cos™> | Osin’@..] (4) | ‘Adding 2) and (4), we get cosmé + sinn@= ["C,cos"8+™C, cos" sin? ="C,cos"-*@ sin*0-"C,cos™ 0 sin?@...) = Misia (moa 2) = es 29 sinto me Joint ="C,c0s"-26 sint@—"C,cos"" 70 sin? 6 ..J "Cyp0u"O+"Cyc0s"10sind | Putting = © , we get Visio (c= ) = part G4" HPC) # OCH "CS -MCG—"C) OC st C2 "Cet "gl Hence, m+ | = 4k, for given quantity to be 0. => m=4k—1, where ke N Binomial Theorem 14.17 30 (a). Weave, S=1+ntnt+,..+ 02 256 _ "ne 5 =D Guat n-l n-l S= (41) (ent et) ‘Thus, the largest value of m for which 1 + m+n? “bao ois divisible by rf" + 1 is 128 34 fb). (La aPCy HMC CE HMC. Ce ‘On integrating w.rt x between limits Oand x, we get aay” nal Multiplying with x and differentiating, we get Len ‘} a 4 frets 26 de 2 net 3 itt ale x(nt (le) -1 n+l FO Gx? .7on = Cox putx= 1,we get laws et n+l Bag dig, Cot SC +S "Cat 2nt3)-1 n+l 32 (c). A = coefficient of xin ["Cy(1 +2)"+C, espns, soefficientof.xin (I+ (1+x))™" = coefficient of xin (2+x?" 2" = coefficient of xin 22” () 4 j 33.(d). General term in(1 + Ay"(1+ "G+ py"is (CAPO MD CAY "CnC ACRE term contains coefficient of 4"y"if ptf-r=f and gtr=n and g=n-r Now, f (nop, Contains coefficient of AU" = Coefficient of A"u"="C,"C, ,"C, s+ Coefficient of Au" ='C,)? aa You have either reached a page that is unavailable for viewing or reached your viewing limit for this book. aa You have either reached a page that is unavailable for viewing or reached your viewing limit for this book. aa You have either reached a page that is unavailable for viewing or reached your viewing limit for this book. Binomial Theorem 1424 The given series is the coefficient of xin the product of 1 1 1 R.HS. of atovetwo. Gan@rD Sum of the series ~ coefficient of x” in (1 +x)" (+3y" t = coefficient of” in (1+xy"*" mC. 64 (b). We have, {88 (6). Letthe expansion bethatof (1 +3)". (43 =O tO xt Ct ht Gx tt) ‘ Leta, brs dBethe (r+ Ne, (r+29h (r+ 3hhand r+ ——_pigereniaing equation (1) W28.x meget coefficients. 126,420 302 a bh nan", a", (1 +2 1=C, +20 + 3Cy? +... tC gl. (2) Differentiating equation (2) wr. 1.3, weget Now (a1) (1 +29"? = (1.20, + (2.3) Cyrene BR ie sect nfa—)C,"2..3) Putting.x= 1 in equation (3), we have (1.2) C+ 2.3)C Fresh (n= Dn C= 2, Wb (reel 42 Simmtarty, oo 62(c). Let =2m,thenk=3in €_(r+2)t1 43 “ am ced nel wat BOT Mae = BI Ces ad a Heap tne, +E PMC 2h =6C- (3)P OG +3) bee’ 1 [ mn 3 6m lil Ds Oo =f 8% (5) "G+ (8 ce. cab vccicke Hsia. (iP "6 +65) ath bic od L os © sg we in. py imaginary partof (1+ V3 1 59(c). 34°= (34)! = (81)' = (1+ 80)'%= 1 +! (80) . mn PM or 8 Lic (60 | = -Limoginary pan of (1 4 V3 ; =1+ 8000 + lasttwo digis in each erm is 00 B 202 Last two digits= 01 | ae | = Jpimaginary part of (cos + isin G0 (a). Consider (1+x)"=Cy+Cyx+CpP + GP tin | ws 4 3 oll) | Integrating equation (1) ws. to, between limits 0 andx, | = pam xono (+ sin2m m= 0) wee B fieoraet at + 22 att! 14299] 1424 (2 OEE tee gy | SU) [SeleS) Integrating equation (2), taking limits from — 1 to 0, we get ae Grattan .y cqegite te. = (142) 01 ra # florcdeaz. a= nat = ~) Tee . [sea ge. (e22 al Hence, the coefficient of >°! in $=5"'C,,,. 2°23 34 1 [orenns2) mei], | 64 (b). Wehave, -[ GG 6)" =6"= 1459" 12 23°34 den fear ae 63 (a). The given series lo | | | GC, S+ CF +CyS+.c+ C5" [where C,="C] aa You have either reached a page that is unavailable for viewing or reached your viewing limit for this book. aa You have either reached a page that is unavailable for viewing or reached your viewing limit for this book. aa You have either reached a page that is unavailable for viewing or reached your viewing limit for this book. man] | eal -o(! I l 1 MCG ARH) "Cs fat 1P ip Gow y $a CG, Leen? + Ss The expansion comains aterm.Sif <= $ of r= 11. 16 (197). Let (J2 +1)°=n +f wherene Nand 0 (mma? (mt n)= 3 mtn=9+12=21 Adding ()) and (i), we get Im=24 orm=12. Ail) 19,912). The generaltem in he expansion of(1 +24 32)4 =—#_.8.an ax? ary !05! BY ge ge gee if ailary!05! where a, dy, 0 are non-negative integers satisfying the conditions ata tad @ For coefficient of x°, a, + 2a, = 5 Q) From (3), possible non-negative integral values of and oy are : > 1,05" 2 +. from), a= 1 73,021 + from@), 0, =0 From (1), required coefficient of x* -— AL yt 4 98. 51-2164 06= a2 gn 2 2-26 96-812, 20 (0). Wehave, aly (at AC, Har 2d\Cy (at 30C, + no upto (n+ 1) terms a (Cy C)+Cy— Cyt EI GY + d(C, +2Cy-30, + FENG) MD) We know that (-ay"=Cy- Cx G2 C2 +. CIC, A) Differentiating both sides w.r tox, we get =n (=x! ==, #2023 + +n, laa) Putting x= 1 in (2) and (3), then we get Cy- yt - Cy +. +E NPC, =O and, ~C,#2G,-30, +. +2 1)"C,=0 Putting these values in (1), we get @Cy (at dy Cy Hat2d) Cy (a+ 3d) C+ =a(0)+d(0)=0+0=0, upto (n+ 1) terms aa You have either reached a page that is unavailable for viewing or reached your viewing limit for this book. aa You have either reached a page that is unavailable for viewing or reached your viewing limit for this book. aa You have either reached a page that is unavailable for viewing or reached your viewing limit for this book. 14.30 Mathematics for IIT-JEE = Bei =e ghmar<2— a 2 > go mde “ eres nt2 n+? n 6 (d). Givenseries is eet 2S tS ty 3 36927 and we know that, times MAD 2 , mn=I(n=2) 5 (ee Heme MED 2 MODE) a Compaitiig these:two, we get nn-l) 21 36 22 ‘Now, divide (2) by square of (1), we get, and, = dn ah n+ In), (nt Iin)(a= 1)(n=2) a ED UD Dk =2)(n=3) ] ee ca ee al Spt de oy oe eel we [er Cg #90, 49, +. 2"-1, BIQH Ct ett, 8 (a). Using"C,= =."'G, ,, fork 1, Noy eas kel oe 2+ the given expression= =>), *Ck-+1 4e2n 9 (a). Let d be the common difference of the A.P, then wehave, a, a Y "GS= YGF la, + (kDa) kao 0 2 -(o-f)S ment $ira =o 2 (« wll yar + dian + n(n=1)2"2) rol dade -1y2*3 n2""+ nla, ~a,)2" [+2 a, a, = (2-1) 2" (4a, +a,-a,]= 2” (2a, +a,+a,) wom MAE easy QUESTIONS FROM PREVIOUS YEARS’ IIT-JEE 1, If'in the expansion of(1 +x)" (1~x)", the coefficients of x and.x* are 3 and ~6 respectively, then mis @ 6 © 9 © 2 @ m4 2. IFC, stands for” C,, then the sum of the series [0-20 +33 -.4 CI" NC, ‘where 7 is an even positive integer, is equal to aa You have either reached a page that is unavailable for viewing or reached your viewing limit for this book. aa You have either reached a page that is unavailable for viewing or reached your viewing limit for this book. aa You have either reached a page that is unavailable for viewing or reached your viewing limit for this book. 45.2 Mathematics for IT-JEE 6 0 6 Forexample, the matrix 4=|0 4 0 J.isadiagonal oo 2 matrix, and is denoted by 4=diag. (6 4-2} Scalar Matrix A square matrix in which everynon-diagonal element iszeroandall diagonal elements are equal, is known, as scalar watrix. 6 6 200 For example, the matrices A=] 5 aa 20 002 are scalar matrices of order 2 and3, respectively. UnitMatrix A square matrix in which every non-diagonal element is zero and every diagonal element is I, iscalled a unit matrix or an identity matrix. Thus, a square matrix A= [ely on i8 4 unit matrix i _ fOwhen ies 40" V1 when i=) A anit matrix of order7 isdenoted by J,0r -Forexample, 1 oO 100 co 1 fand =| 0 1 0) are unit matrices of oot onder2 and 3, respectively UpperTriangularMatrix A squarematrix 4 =[a,]iscalled upper triangular matrix if ay = 0 for all > j, For example, the matrix isan uppertriangular matrix. Lower Triangular Matrix A square matrix 4 = [ay] is calleda lower triangular matrix ifa,=0 forall a a 6 |[ ab A AAM) 92 ab || -a? -ab [t -s Hence, the matrix 4 is nilpotent ofthe index 2. KEY POINT(S) TO. REMEMBER © Minimum number of zeros ina triangular matrxis given hn by. m _ ). © A triangular matrix = [a,],,,. , is called strictly, tiangularif,~0forl I. ‘Then, Ba! B'— ba Bt (@) isanullmarix —_(b) isanidentity matrix (6) asingularmatrix (qd) none of the The number of different matrices which can be formed. using 12 different real numbers is (a) 6(12)! (b) 12112)! © 442 (8) none of these Asskew-symmetric matrix 4 satisfies the relation 4+ 7-0, here Fisa unit matrix. Then, 4 is (a) idempotentmatix (b) Orhogonal matrix (©) Nilpotentmatrix (4) none of these Let A be ann «n matrix such that a” = a4, where cis, areal number different from | and. Then, the matrix, At his (a) singular (b) non-singular, ie, invertible (©) scalar matric (@) none of these adj 8 = AandP, Qare wo unimodular matrices, ie.. [P|=1=| Ql, then (1B Ps equal to (@) PQ (b) PQ (© gap (@ QBP te WA= | then ) Jim La" =0 (@) none of these IA = 0 for some value of kand (J Ay =1+4 +4? | then p is @ -1 © 3 (b) -2 (d) none of these It satisfies the equation x3 Sx24+ dy + K/=0, then A exists if, (@) kel (© kel (b) #0 {d) none of these IfMisa3 » 3 mauix, where 4¢.M= and detM=1, then, dot (M—D= @ 0 @-1 ) 1 (d) none of these aa You have either reached a page that is unavailable for viewing or reached your viewing limit for this book. aa You have either reached a page that is unavailable for viewing or reached your viewing limit for this book. aa You have either reached a page that is unavailable for viewing or reached your viewing limit for this book. aa You have either reached a page that is unavailable for viewing or reached your viewing limit for this book. aa You have either reached a page that is unavailable for viewing or reached your viewing limit for this book. aa You have either reached a page that is unavailable for viewing or reached your viewing limit for this book. aa You have either reached a page that is unavailable for viewing or reached your viewing limit for this book. 15.20 Mathematics for IIT—JEE 6 (a, d). Since | 4 |#0, therefore 4” exists Now, 441 =1= 444 = Ads P(A lays aya’ Aa = uly wy = Ala(A))) => AT issymmeric. Also, since | 4] #0, -. A exists such that Ad M4| 4d? = JANA =1 (eo [4B |=] 4B) 1 = |4! Al 7 (ab,c,d). (a) Forany orthogonal matrix A, we have AA=T Let Bbe amatrix such that 4B = 1. Now we have A'=41 —— [byproperty of unit matrix] =A(AB)~ (AB = 1B -B ‘Therefore, (A(A)=Ad"= AB =F = A isorthogonal. (b) Forany orthogonal matrix 4, we have AA = al=it = La'Iat=1 > |A| #0, i.e.,4 isnon-singular. (c) Let A and B be two orthogonal matrices, therefore, (AB) (AB) = B'A'AB =8(‘AB [by asssociative lax] =P(B)=2B=1 = ABis orthogonal. (@ Let be orthogonal matrin and B be its inverse matrix, ‘Then, we have, AAT fl and, AB =1=BA Q) Now, we have, (ABY (AB) = BA'AB = BU N)B~ 8 IB)B= 88 “) Also, from equation (2), wehave (abi =1= ie, BB=I [using equation (3)] = Bis orthogonal. 8 (b,d), We can seethat 4’ =A. Therefore, 4’4 = = Har a |#=|1 = \#|=1 = [Ajmal ‘Therefore, a? + b +c —3abe=+1 2th +O=24. 9 (a,b,c, d). AMT= LAH) = BUA A) oA | ANT =A,» (a) holds, 1 Ata a=]ase aa a ) (b) holds. (A : (4B + BA) Bed~ 3 (BA AB) = LAB+ Bd) dition s commutative] A*BOBHA, +. (opholds A*B+O = 5 (A(B+C)+ (B+ 0) A) 2 ~Lunesceaiece (4B+BA)*4(AC+CA) *RLAKC + (d) holds 10(a,b,0,). Given 4B ~ BA (@) The result 4"B = BA” is true for 4°B = A(AB)= A(BA) =(AB)A=(BA)A = BA? Let the result be rue for n= k {by property of transpose] | ie, 4*B=Ba* Form + I,we have A B= AcdhB)~ ABA) (AB) AE (BA) AE =BAshl Thus, the result is true for n= &+ 1 Hence, by induction the result is true Vn € NV (b), (c) Since 4 and B commute, both these options hold. (G) (4" = B) (A" + BY) = 4 + A" BY BAB ~ AB, [A and B commute] 11 (b, d). We have, 10 -2 fatj=|-2 1 0) =-20-241)=2 -11 0 @ |Al=—5 lay nm 0 () adjA= ala =| -112 0 “1212 0 (© lagia @ |A'\=|Al= 2 aa You have either reached a page that is unavailable for viewing or reached your viewing limit for this book. aa You have either reached a page that is unavailable for viewing or reached your viewing limit for this book. aa You have either reached a page that is unavailable for viewing or reached your viewing limit for this book. aa You have either reached a page that is unavailable for viewing or reached your viewing limit for this book. aa You have either reached a page that is unavailable for viewing or reached your viewing limit for this book. aa You have either reached a page that is unavailable for viewing or reached your viewing limit for this book. aa You have either reached a page that is unavailable for viewing or reached your viewing limit for this book. 3. Ifany two rows (or columns) ofa determinant are iden- tical, then the value of determinant is zero. Explanation a be Let|4|=|a 8) ¢2] byany determinant inwhich a,b; second and third rows are identical. Expanding by Ist row, we get aja) °|-n [2 Slag|2e & by eg] "Jag ¢g|**|az by = ay (bye, ~ bye) ~ By (ye ~ 040) + ¢, (ashy —asb,) =a (0)-b, +e (=O. 4. Ifthe elements of a row (or column) ofa determinant lied by a scalar, then the value of the new determinant is equal to some scalar times the value of the original determinant Explanation a be Let|4|=|4, by ¢2| beany determinant oforder a, by cy Expanding by Lt row, we get by cal y|az cal, |a2 be \4|= a 1 ey bs es} ‘]a3 es] “lag by = ay (Byes Bye) — By (aye ~ 503) +6; (dyby—ayb.). Let] B |be the determinant obtained from | 4 by muli- plying the elements of third column by &, then by key by key 4 =a, (Kbye,— bye) — by (kage — heye,) “ke, (@bs~ ash) =k fay (byes —bye>)—b, (aes aye) +e; (a:b —ayb,))=K| Al. Notes: I. By using property 4, we have fay th, mey a 4 ka, Ib mey|=Hm|ay bse kay iby mey a, by 6 kay hk nn ma; mb, mes 2. If each element of one column or one-row, is the same multiple of conesponding elements of another column or row, the determinant vanishes. Explanation a kay | fa a 4 Al=|@ kay |=klay ay ey a ka, | Ja a cy a bh G =kx0=0=|ka, hb, key |. a, by cy 5. Ifeach element of any row (or column) ofa determi- nant is the sum of two numbers, then the determinant is expressible as the sum of two determinants of the same order. Explanation ante, +B +n Laj|=| a, by a bogs Expanding] A [by fst row, we get be jaye sal Sf 6-0 ata), (by + By ing eg % tl Ha +1)| dy by & abel Sf a €|" Jag by a, le es)" Jay a be] lo Bon sla, by &|t]a, oe]. a by 63] Jay bs 6. Ifto cach element ofa row (or column) of adeterminant be added equi-muttiplesof the corresponding elements, ‘of one or more rows (or columns), then the value of the determinant is not changed. Explanation a bg Lat} 4|=|ay by cp| beany determinant of order 3, jog Oe Let] B | bethe determinant obtained form| 4 |by odd- ing p times the elements of the second column and q times the elements of the third column to the corresponding ele- ‘ments of the first column, aa You have either reached a page that is unavailable for viewing or reached your viewing limit for this book. aa You have either reached a page that is unavailable for viewing or reached your viewing limit for this book. aa You have either reached a page that is unavailable for viewing or reached your viewing limit for this book. 16. 7 18, 19, 20. aL (2) 19n+1 (b) 19n+2 (© 1% (4) 1943 (o+e)y+2)-ar=b-6, (cha)e+s)—by-c-a, (a+ Bix ty)-ce=a~h, where a+ +040, thenx= exb anc ©) Grose © Grote b-a * —i © Fabee © Tabve flex or 1 Ifx#0,y40,240and |I+y 1+2y 1 |=, tz lz 1432 then x! +y"142°1 is equal to @ b) 2 o3 (@) none of these a (s~ay (say? W2s=atb+cand jis-by BF (s-by GF or =k (sa) 8) (@-@), then kis equal to @ 2 (b) 2s (©) 2? @ 29 If4, Band Care the angles ofa triangle and 1 1 1 ltsinad —tesin 1 +sinc |=0, sin dtsin’ A sinBtsin?B sinc +sin?C) then the AABC must be (@) right angled (b) isosceles (©) equilateral (@) none of these Let a, fibe the roots of theequation ax? + bx +e=0. Let Sy7 OM PF for 1. Then the value ofthe determinant a 28h, 185; les, Its; Less is Its, 145, Ls, fq) (at b+ eN(b? = 0c) at+b+ey 4ac) o Sree at btey (6? 4a fo) (etbter® (Bt tee) @ (4) none ofthese je be e4bl The value ofthe determinantlate 6 eels la-b at6 2. 24, 2s, 26. Determinants @ @+Pte © abe (ar d+e) © WP +e) (etb+e) (@ none of these cower 1 ° If] 1 2eosece = o 1 eosecer then zis equal 10 (a) sin 2 © wna? (b) 0s a/2 (@) none of these ° e e M(art? (bs? (ert? (a-1)? (6-1)? (e-1)? =k (a—b)(b~c) (c—a), then kis equal to 4 -4 @ 2 @ -2 ‘The value of the determinant (a~a,)? ct (@-a,)? (a-ay a (aay! a, (2-4, (aay)! as “1 a,"'Jis @II(a; ~a,) eta, 1 — (b) © calia-qy © Hla, 1(a~a,)? ENE) Ta, MI(a~a,)* 1a, Ma- a, 9 nia, ~4;) @ ~ i where Q is the ctrl O 1 ore ate product of denominators, then P is equal to @ (a-b)(b-oe-a) (b) G@-y)(y-2)-2) (©) (a—b) (bc) (c~a) (x-y) W—2) E-2) (©) none of these Ia, b,c, dare the roots of the equation ca + oo + x2 + Ge +E=0, then the value of the determinant Ita toot ot Toth tT fig 11 tee 4 toot 4 teal bee g-8 @ = © aa You have either reached a page that is unavailable for viewing or reached your viewing limit for this book. aa You have either reached a page that is unavailable for viewing or reached your viewing limit for this book. aa You have either reached a page that is unavailable for viewing or reached your viewing limit for this book. aa You have either reached a page that is unavailable for viewing or reached your viewing limit for this book. aa You have either reached a page that is unavailable for viewing or reached your viewing limit for this book. aa You have either reached a page that is unavailable for viewing or reached your viewing limit for this book. aa You have either reached a page that is unavailable for viewing or reached your viewing limit for this book. aa You have either reached a page that is unavailable for viewing or reached your viewing limit for this book. aa You have either reached a page that is unavailable for viewing or reached your viewing limit for this book. aa You have either reached a page that is unavailable for viewing or reached your viewing limit for this book. aa You have either reached a page that is unavailable for viewing or reached your viewing limit for this book. Rapa spy keds 8, 70 [Applying C, > Cy = Cy] 26 +B +P UB+ N+ @)— af 2B (e+ B+ GF + 07+ BY By (a+ B+ W=2s° (sa) (s—b)(s—e). 3 tes Ts 20(b). LetA=|1+s, 1+s, 155 =| tta+p ieat+ Bp? 14 3 Tes) Ltsy Dts, l+a4p t0?+p ls@+p? t+ah+p itatspt [es S,=@"+ B'forn2 I] ta P| la B biog wher, Aj= jl a BI]. bap 10 0 Nowa,=|1 a-1 pt 1o-1 Bp asa [Apalying C) > =(oB-(a+B)+1)- Yla+ BP — 4p a+p a-1 p-l +1 B+l (oB—(ar+ B)+1) + (B-a) rod ene}, Ges 4, Phare the roats ofthe equation ax? +x + ¢= 0, and ap =< (asbeory ac > _ (ath +o? x(b? ~4ac) i a a and Cy Cy -C)) [Expandingalong R,] | Determinants 16.23 a bec eb 21 (0). Wehave,|ate b c-a a-b ath | 2 re-0 ece+ai| ul 2 ~ Ge laiare) 8 ete=a) a\a-b) (a+)? [Multiplying C,, C) and C, bya, band c, respectively} PHP He? bbe) clerd) ape |etet te? 6? ee-a)) Bee ee? blatd) — e? [Applying C, > C+ C,+G] 2 |] b=) c(e+b) 1B ee~a) @sbiee ‘abe 1 ware) [Taking @ + + 2 common from C)] toa jh bee eth aed 1G t abe e ena loath oc [Taking b and c common fiom C, and C,, respectively] 1 b-c oth @rbse |) e 0 ate a [Applying Ry > Ry~ Ry, Ry—>R,—Ri) OPO Cpe abt act be) [Expanding along C,] =@+P +e) (atb+e, coca 1 o 22{c). Wehave| 1 2voseca 0 1 2cosect eosecat 1 o =] 1 2eoseca-sine | 0 1 2eoseca [Applying &, ~» Ry ~sio aR} ine 1 1 2eosecee ‘ [Expanding along R,] = cosee a(4 cosec? a-2~1) aa You have either reached a page that is unavailable for viewing or reached your viewing limit for this book. aa You have either reached a page that is unavailable for viewing or reached your viewing limit for this book. aa You have either reached a page that is unavailable for viewing or reached your viewing limit for this book. 16.28 Mathematics for IIT-JEE 4 (b). We have, , [@rytore-9 reyes) a 1 1 -t o Gryteerey a a(a+d) (a+d)(a42d) Pe ze L 1 1 avd (@td(at2d) (at2d)(ats) 4 1 L 1 aeid (ar2d)(a+3) (at3d) (a+ 4d) L a(atdy (a+2d)3 (a¥3d)° (at4d) (atd\a+2d) sat2d a x[@+2d(at3d) at3d ard (at3d)(at4d) atdd at2d 1 x yo ty yoz-x 2r-y) ee “ zexny 9-28 afar dy (at 2d) (a+ 3d¥ (a+ 4d) a ” (a+ d(ar2d) a+2d a {Applying C > CC, C4] (a+ 2d)2d qd yeewx 0 =2y (a+3a)2d aq di aye lg peeny -2e [Applying 8; &s~ RyRy > RyRy] DE 2 yh lay (Applying Ry > Ry * Ro) a(a+dy (a+ 2d) (a+ 3d)? (a+ 4d) (atd)(a+2d) a+2d —2d| stetytay Ot 0 Bay (a+ 2d)2d 4 0 |.G3G wee | 2 Met” By (a+3d)2d 4 0 x » ay 2d [Muliplying R, 2; by x and y, espectively) “Was dy (at 2d) (a+ 8dy (ae aya) oy? alas dy (a+ 2d) (a4 3d)? (a 4d) ceyan Or) 9 ° \(at2d)2d a ae | EHD 0 |ar3a2d a ae | op yo dy _ ad? avid | [Applying R, > R,+ Ry Ry By +R) a(a+ dy (at 2d) (a+3d) (a4 4a) [243d 1 (xty42)? xt) oy? _ sat ~ yee x y(e+x)| © alas dy (a+ 2d) (a+ 3d)" (a +4d) {Expanding along C3] 2 2 4 ‘i x42) (beer oc? e we 2G is 6 we 45a). Wehave,| e2 (cra? a? ae a a (a+b)? _2Gtysey [eyes eye abre? ate? a? AinmeR ae ee Bera? a? aweyeep |i I 5 we ate? at ay eee eta Pte [Muttiplying &,.R, and R; by =2 (ab + be + ca), ‘and ¢’, respectively] 46 (d), ‘The three equations in two unknowns will be con- ye sistent if = e+e? (asi (423 -(a43 yo (atl) (a42) ~(at3) (Putting be =x, ea= y and a l 1 = aa You have either reached a page that is unavailable for viewing or reached your viewing limit for this book. aa You have either reached a page that is unavailable for viewing or reached your viewing limit for this book. aa You have either reached a page that is unavailable for viewing or reached your viewing limit for this book. aa You have either reached a page that is unavailable for viewing or reached your viewing limit for this book. aa You have either reached a page that is unavailable for viewing or reached your viewing limit for this book. aa You have either reached a page that is unavailable for viewing or reached your viewing limit for this book. aa You have either reached a page that is unavailable for viewing or reached your viewing limit for this book. 16.38 Mathematics for IIT—JEE 22 (2). Weave, x-a(42)-0 = (It @x-a@rytz)—0 (1) y-Brts\=0 (14B)y-Blery+2)=0 2) z-7(ety)=0 (1492-7 (+y42)=0 8) Since the given system of equation have only non-trivial solutions, «: (+r y+2)#0[ Ife ry +2=0, ten in view of (1). Q)and 3), wehavex=0,y=0.:* 0, a trivial solution} @ os ply Isa xtys21+P xtyee i 2 md = Tey xt y3z Phe, rasa Onading,weeet Foti hg t Tey rot el pelle ita 4p ey Jr oo4 1 23.(3). Wehave,| 244, 78, ng MEA FB, 1 1 ! = nek (n+ 80047) 1 0 -4 on 4 8 2 nOr=1) S412 16n+ 56 {Applying C, + C, -C, and C, + C,-C\] = F[8 16+ 56)—8 (8+ 12)] = 64 = 43 “Maximum value of n= 3. 25 (0). IFristhe common ratio, then @,=aye"" ‘n> 1 = ogg, =log.a, +(n-1) logr=4+ (0-1) R, wher, 4=loga, and R= log r. loga, toga, 42 TOBA, 4 loga, sg loga,,, loga, , 19 WB4y 412 WBGy 414 OBE, +16 AHR AF(HDR AFUE DR H[At@tSR Ate DR Arr HR ARWHIDR AFW4INR Aa (ns 15)R At(@-DR 2R 2K =|A+int5)R 2R 2K At(@MHIR 2R 2 (Applying €,-$C,~Cy,C,> C—G) . [> Chand Care identical. 26(0). Sa, 2 Ero) 0 =| m?=1 om sin? (mm?) sin? (mp moi sin? (m+ 1) mime) 2 (mel) 2" m4 = m1 2 sin? (m?) m4 sin? m sin? (m+1)| 2m ml oF m+ sin? (m+ 1) sin? (m?) sin? m 28 (10). Wehave, 1 3co8 1 sind 1 3e0s0 sing a Applying R; -> R;~ Ry 1 Be0s8 1 =|sin@ 1 cos 0 sin@-3cos# 0 (Gin 6300s 6) 3cos @-sin &) =(BeesO-sin Now, ~ (TFT < 3cos0 ~sinos JFFT + Geos @-sin @)* < 10. 29 (0). Wehave, 4 a B = ASlete™ } 1 4a? 7 aa You have either reached a page that is unavailable for viewing or reached your viewing limit for this book. aa You have either reached a page that is unavailable for viewing or reached your viewing limit for this book. aa You have either reached a page that is unavailable for viewing or reached your viewing limit for this book. aa You have either reached a page that is unavailable for viewing or reached your viewing limit for this book. log, (nt) = nx = n log, m. VL Base changing formula log, m log,m= P82 0 ogy if m, n, @ are positive and n #1, a #1 Explanation Let log, m= x s0 that a = m 0) log, m= y so that a? =n 2) log, m~ = s0 that 1? ~ m 0) From (1) and (3), we have a = n= = a=(a’¥ [using (2)] = daa" = xayz or => Putting back the values of x, y and z, we get log, m og, 7 Note that we can use any base in place of a, ie., for any base ¢ (c > 0, ¢ # 1), log, m 4) log, m fog. log, m= is true, VIL.Reciprocal relation log, a * log, = 1, where a, b are positive and not equal to 1 Explanetion Let logpa=x and log, b= y Then, =a and a?= Eliminating a, we have oy . aye Hence, log, « * log, = 1 L Tog, b orb =b Cor. log, VILL a'*8* = x, where a and x are positive, a #1 Explanation Let log, x= mr then a” =x or altto® = x. Common Logarithms There are two bases of logarithms that are extensively used these days. One is base e (e = 2.71828 approx.) and the other is base 10. The logarithms to base e are called natural logarithms. The logarithms to base 10 are called the common logarithms. We shall discuss only common logarithms in this chepter and write log n, without indicat- ing the base, for logerithm 7 to the base 10. That is, log n will mean logy ". Cleatly, Log 10 = logy 10 = 1, log 100 = logy, 100 =2, log 1000 = logig 1000 ~ 3, ote, Characteristic and Mantissa ‘The logarithm of any number V is partly integral and partly fractional vev- [ieee] + — [Facton T T Caractere Martisse (ve, -ve oF zero) ( Characteristic ifthe number characteristic = (number of digits inintegral part-1) Ifthe number N'< 1, then characteristic* = (number of zeros after decimal point + 1)* characteristic will be negative and a horizontal bar (is put over the value of the characteristic, Gi) Mantissa The mantissa of all numbers consisting of the same digits in the same order are the same, whatever may be position of decimal point. g,, Mantissa of 2916, 29.16, 0.2916, 0002916 willbe same andits value can be obtained from the log table. (always +ve< 1) > 1, then IMPORTANT POINT(S) TO NOTE Logarithmic Inequalities + Ifa>1,p>1=logp>0 # 100 101=logp <0 Ha>1,0a> i= bogp>t # Ma>p> 1 0 (©, Ie (b), Ve [b) 4(@). be “Iog, og, 10, 2.0 Gveiny neon tog, , (2! log, ) logy 89) < Hogg a(—5) = log, 108, 108, -- log, , = Float =5)<0 log,.m = 1. SR SH1SHR6 Axe (GO) aa You have either reached a page that is unavailable for viewing or reached your viewing limit for this book. aa You have either reached a page that is unavailable for viewing or reached your viewing limit for this book. aa You have either reached a page that is unavailable for viewing or reached your viewing limit for this book. Vector Alge REVIEW OF CONCEPTS ‘Wecome across different quantities inthe study of physical phenomena, such as mass or volume of the body, time, tempersture, speed and so on, Alll these quantities are such that they can be expressed completely by their magnitude, i.e, by a single number. For example, mass of a body ean be specified by the number of grams and time by minutes and s0 on. Such quantities are called scalars, There are certain ‘other quantities which cannot be expressed completely by their magnitude alone, like velocity, acceleration, force, «lisplacement, momentumand so on. These ere the quantities, that can be expressed completely by their magnitude and direction. Such quantities are called vectors Scalars and Vectors Scalar Quantity A quantity which has only magnitude and no direction is called a scalar quantity or simply a scalar. Examples of sealarare mass, temperature, volume, work and so on. To specify a scalar two things are needed. 1. aunitin terms of which it is measured 2. areal number (positive, negative or zero) VectorQuantity A quantity which has magnitude as well as direction is calleda vector quantity or simply a vector. Examples of vectors are displacement, velocity, acceleration, force and soon. To specifya vector, three things are needed: 1. aunit in terms of whieh it is measured 2. areal number (positive, negative or zero) 3. paricular direction, B Representation of Vectors ] “The best way to representa vectors withthe help ced line segment, Suppose, A and Bare a two points, then by the vector AB, we mean a quantity whose magnitude is the length AB und whose directions ffom A t B. A and B are called the end points of the vector AB. In particular tis called the intial point and 2 is ealled the terminal point Sometimes, a vector AB is expressed by a single letter a (which is abwaye writen in bold type, to distinguish it from a scalar) Sometimes, however, we write the vectoraas @ or 7. ABean also be wiitenas AB. IMPORTANT POINT(S) TO NOTE Every vector AB has the following three characteristics: (a) Length: The length of AB is denoted by |AB| or simply AB. (b) Support: ‘The line of unlimited length of which AB is segment is called the support of vector AB (c) Sense: BA w The sense of ABs from 4 to Band that of be from 2 tot, The sense of directed line nent is fr ts initial point to the terminal point. Modulus (or Magnitude) of a Vector ‘The positive real number which is the measure ofthe length of the vector, is called the modulus, length, magnitude, absolute value or norm of the vector. The modulus ofa vector a or OA js usially denoted by |.a/ or| OA | orby the corresponding letter ‘a” (notin bold- faced type), ic JOA |= Odand| al Multiplication of a Vector by aSealar ‘The product of a scalar m and a vecior a, is defined as a vector ma or am whose magnitude is the product of the ‘magnitudes of m and a and whose direction is that of a or ‘opposite to.a accordingly as m is positive or negative. aa You have either reached a page that is unavailable for viewing or reached your viewing limit for this book. aa You have either reached a page that is unavailable for viewing or reached your viewing limit for this book. aa You have either reached a page that is unavailable for viewing or reached your viewing limit for this book. aa You have either reached a page that is unavailable for viewing or reached your viewing limit for this book. aa You have either reached a page that is unavailable for viewing or reached your viewing limit for this book. 7. Scalar porduct of two vectors. terms of components: i a=aitaytagkandb=hyi+byi~ bk then a-b=a,b, + ayby * ashy Thus, the scalar product of two vectors is equal to the sum of the products of their corresponding ‘components, Explanation Leta=aita,jtajk and b=hji+h +byk Then, a-b=(ai+aj+2k)-(bi+ bi +bk) . = (ail? taybab- J+ adie k +ayhj-itasbsi? + asbaik + agbhk i+ asbak 5+ ashy (by the distributive law) =ayb((1)+4,b40)* ayb0) - + agh(0)*a.by(1) + ayby(0) +56 (0) +4360) +a5b,(1) = ayby + ayby + aybs. Hence, a b= ab +030) aybs & Angle between two vectors in terms of the components of the given vectors. If is the angle between two vectors 1+ aj +ajk and b ab [ail] ayb + ayby + ayb, i+ bal + Dyke then,cos Explanation Leta~ i+ ay) + ask anéb— hi + bgi + bgk betwogiven vectors and @be the angle between them, Also, let| j= and |b|= ‘Then, by definition, a b = abcos@. b cos = > A) No, we have already seen that ab = (ai ngg + ay) (bi Byf + bd =ayb, + ayby + ayb; and, a= Jo? a} vad b= Sip wh Putting these values in (I), we get aby taabs +axb, Ja vot eat Joes eh cos = 187 IMPORTANT POINT(S) TO NOTE 1F8 is acute, n.b is positive and if @ is obtuse, a.b is negative 9. Geometrical Interpretation of Scalar Product. Explanation LetOA=a, OB=hand A408= in the given figure. Then, by definition a - b = abcos®, where jal = a= Od, |b] 0B. Fig. 18.9 From 4, draw ALL OB. From B, draw BM LO. ‘Then, OM = projection of OB on O4 = OBcosé = bees OL = projection of O4 on OB = Odcos@ = aces. a- b = abcos@= a(bcos6) = OA» (projection of OB on OA) =|al (projection ofb in the direction ofa. Again, ab = ahcos0 = b(acos6) = OB * (projection of O4 on OB) =|b| = (projection of ain the direction of b), ‘Thus, (i) projection of a on b~ ab lal 10. Components of a vector b slong and perpendicular to vector a of bon ab ‘Component of b along a= (3 a a ‘Component of b perpendicular to a= b ~ ( *}s a Explanation ‘OC is component of b along a and CB is component of b perpendicular toa (see figure below). Now, aa You have either reached a page that is unavailable for viewing or reached your viewing limit for this book. aa You have either reached a page that is unavailable for viewing or reached your viewing limit for this book. aa You have either reached a page that is unavailable for viewing or reached your viewing limit for this book. a’, bare said 10 form a reciprocal system of vectors for the vectors a, Ifa, b, eand a,b’, form areciprocal systemof vectors then, () a.a’=bbt (i) abi=ad Gi) [a be] = l iave] Gv) a,b, eare non-coplanar iff so are a’, b’, Vector Triple Product Leta, b,¢ beany three vectors, then the vectors a x (b *) and (a * b) * c are called vector triple product of a, b, Thus, = (b xe) = (ae) b~(a.b)e Properties of Vector Triple Product (The vector triple, product a (b * ¢) is a linear combination of those two vectors which are within brackets. odor Agsbra 10:9 (i) The vectorr=a * (b * ¢) is perpendicularto a and lies intheplaieofbande. (ii) The formula r= a ¥(b ¥e)=(a .¢)b—(a.b)cis true only when the vector outside the bracket is on the left mostside, If itis not, we first shift on left by using the properties of rose product and then appl the same formula, Thus, (b * ¢) « a=—[a* (bx ¢]=—{(a-e) , b-(a-b)c} =(a.b)c-(a- eb (tu) Veco tle product sa vector quantity @ axoedaeuune Rotation of a Vector about an Axis Let a= (a,, a, a,). Ifsystem is rotated about (i) s-axisthroughan angle , then the new componentsof 4 are (a), a, COs + a, Sin &, — a, Sin G+, COS a. (Gi) axis through an angle a, then thenew components of are (a, sin + a, sin 6 a, sin 0.+ a €0S Ga sin @). (iii) z-axis throughan angle o, then the new componentsof a are (a, cos a+ a Sin a, —ay Sin Oey 208 & cy). PROBLEMS FOR PRACTICE SECTION I 1. aand bare mutually perpendicular unit vectors. If ris a vector satisfying r-a=0,r- b= and [rab] = then ris (@) axb+b (©) b+(axb) (b) a+(axb) (@) axbta 2. Ifa, b,e are three non-coplanar vectors and ris any arbitrary vector, then [be r]a + [ear]b+ [ab re =k [ab e]r, where k= @ 1 (b) 2 ©@3 (@) none ofthese 3. Leta, band © be non-zero vectors such that no two 1 are collinear and (a 1 a2 ©3 @ 5 4, Ifthe vectors a and b are perpendicular to each other, then a vector v in terms of a and b satisfying the equations v-a=0,¥- b= 1 and [v.ab] =I, is b_,_axb QUESTIONS WITH ONE CORRECT ANSWER (@) none of these jaxb 5. a:b; ccna theo vectors of magaiinds {3 1,2 euh that a (ax ¢)+ 3b =O. If Gis the angle between a and ¢, then cos?@ is equal to 1 3 ot ot (@) none of these 6 Leta=i+j+kand r be a varisble vector such that reir J and r+ Kare positive integers, Ifr as 12, then the total number of such vectors is, @ Mey-1 &) Po, © "Cy (qd) none of these 7. Given, |a|=|b|=1and|a+b|= J3.Ifebeavecior such that €~_a—2b=3(a xb), thene - bis equal to 1 3 Ms @ 5 s 1 oF @ -5 8. Aveciorahas components 2p and | ws arectangular cartesian system, This system is rotated through a certain angle about the origin in the counter-clockwise aa You have either reached a page that is unavailable for viewing or reached your viewing limit for this book. aa You have either reached a page that is unavailable for viewing or reached your viewing limit for this book. aa You have either reached a page that is unavailable for viewing or reached your viewing limit for this book. aa You have either reached a page that is unavailable for viewing or reached your viewing limit for this book. aa You have either reached a page that is unavailable for viewing or reached your viewing limit for this book. aa You have either reached a page that is unavailable for viewing or reached your viewing limit for this book. aa You have either reached a page that is unavailable for viewing or reached your viewing limit for this book. aa You have either reached a page that is unavailable for viewing or reached your viewing limit for this book. aa You have either reached a page that is unavailable for viewing or reached your viewing limit for this book. aa You have either reached a page that is unavailable for viewing or reached your viewing limit for this book. aa You have either reached a page that is unavailable for viewing or reached your viewing limit for this book. aa You have either reached a page that is unavailable for viewing or reached your viewing limit for this book. aa You have either reached a page that is unavailable for viewing or reached your viewing limit for this book. aa You have either reached a page that is unavailable for viewing or reached your viewing limit for this book. 24, 25. 26. 27. 28. 29, If a, b and ¢ are unit vectors, then ja~ bj? + | b-cf? + |e-al? does not exceed (a) 4 (b) 9 (&) 8 (d) 6 If a, band e are unit coplanar vectors, then scalar tiple product [28 B,2b -€,26 — a] isequal to (a) 0 (b) 1 (© 3 @) VB Ifaand b are two unit vectors such that a + 2b and 5a—4 bare perpendicularto each otherthen the angle between a and bis @ 4" (b) 60° afl (2 owt) © «() Leta =1-k b= adj +(1—x) kande=yh-ta) > (1 +x=y) k. Then, fab €] depends on (a) onlyx (b) only y (©) neitherynory (A) bothx ard y Ifa (+5 +6), ab= 1 anda b=j~é, then bis (@) i-j+k (i ja? [bP ca ch, Sah. jaf Tbe 30. 31. 32, 33. ca oa by then which of the following is a set of mutually orthogonal vectors’? @) fa be) (b) {a,b,c} © {aby.e} (a) fabyey} ‘The unit vector which is orthogonal to the veetor 31+ 2) + 6k andis coplanar with the vectors 21+ j + kand irks atk 23 om OR ack at () ‘Vio (a) Ba Which ofthe following expression is/are meaningful? (@) w.(vew) (b) (u.v).w (© @.vyw (@) wx (vw) Let A be a vector parallel to line of intersection of planes P, and P through origin. P, is parallel to the vectors 2j + 3kand4j + 3k and P, is parallel to j -k and 3i+ 3), then theangle between the vector A.and2i +])-2kis z x @> 5 x an ©O% oF Let a and b be two ron-collinear unit vectors. Ifu=a =(a.b) band y= aD, then | v| is (@) |u) (®) |u| +|u.al (©) [ul +|u.b] (@) |u| + u,(atb) Pa ELS @ 2. ib) 3b) (a) 10. @ 1, } Oo 86O 20 @ . @) 26, tb). re) bo) 5. ib) 6 @) 7 @) & (a) 13, @) 14. @) 15.) 16.) 2.10 0-2 8.) 2. 29. b) 3. 31. O32 (he aa You have either reached a page that is unavailable for viewing or reached your viewing limit for this book. aa You have either reached a page that is unavailable for viewing or reached your viewing limit for this book. aa You have either reached a page that is unavailable for viewing or reached your viewing limit for this book. aa You have either reached a page that is unavailable for viewing or reached your viewing limit for this book. aa You have either reached a page that is unavailable for viewing or reached your viewing limit for this book. aa You have either reached a page that is unavailable for viewing or reached your viewing limit for this book. aa You have either reached a page that is unavailable for viewing or reached your viewing limit for this book. aa You have either reached a page that is unavailable for viewing or reached your viewing limit for this book. aa You have either reached a page that is unavailable for viewing or reached your viewing limit for this book. aa You have either reached a page that is unavailable for viewing or reached your viewing limit for this book. 19.16 Matiematios for is the bisector of the acute or obtuse angle between the fines (1) and (2) according as aja, + ;b, <0 or > 0. Similarly, the equation aie RbEt Lae la? +b ie +o is the bisector of the acute or obtuse angle between the lines (1) and (2) according as aa, ~ by, > 0 or <0. __TRICK(S) FOR PROBLEM SOLVING faa, 6b >0, then the origin lies in obtuse angle and faa, + bib, <0, then the origin lies in acute angle. Equations of Lines Passing through the Point of Intersection of Two Given Lines The equation of any line pessing through the point of intersection of the lines a,x +b,) +c, =Oanda,x+byy +e, =Ois(ax~ byy+e,) +k (Gr + by +e) =0, where & isa parameter, The value of & ean be obtained by using one more condition which the required line satisfies. Explanation Letthe two given lines be axt byte, “ axt byy tc, ~Q) f(a, B) is their point of intersection, then a+ b,pre and, ant byB+e,-0 i.e. (a 8) satisfies ayxt buy te, +Max + byy + ey) =0 0) forall values of k. Heree, the equation of any line (in general) through the point of intersection of (1) and (2) is (ayx+ by y+ e,) + ROX * by y HE) =O. Standard Points of a Triangle Centroid of a Triangle ‘The point of intersection of the medians of the triangle is called the centroid of the triangle. The centroid givides the medians in the ratio 2 : 1 (2 from the vertex and 1 from the opposite side). Ae Com) Bear) Fig. 19.35 The coordinates of the centroid of a triangle with vertices (xy. Ys (tg, 2) and (xy. 5) are ( te te; atin) 3 3 Incentre of a Triangle ‘The point of intersection of the internal bisectors of the angles of a triangle is called the incentre of the trlangle The coordinates of the incentre of a triangle with vertices (x). yy) (p79) and (xy yy) are a) Fig, 19.36 fa tbr, ter; ayy thy, +05) athte E TRICK(S) FOR PROBLEM SOLVING The incentre of the triangle formed by (0,0), (a, 0) ard (bis ab ab freer nacioar ExCentres of a Triangle A circle touches one side outside the triangle and the ‘ other two extended sides then circle is known as excircle. Let ABC be a triangle then there ate three excircles, with three excentres fy, ly fy opposite to vertices 4, B and C, respec- tively. Ifthe vertices of triangle are (2,4) B (>, 99)and CO, ree 3) then P ( thr, +c: as V ~arbre ~atbee @ bie, +085 atta) a=bee a-bee ay thyy—o atbme Circumcentre The circumcentre of a triangle is the point of intersection ofthe perpendicular bisectors ofthe ax thr, ~ery 3 ab aa You have either reached a page that is unavailable for viewing or reached your viewing limit for this book. aa You have either reached a page that is unavailable for viewing or reached your viewing limit for this book. aa You have either reached a page that is unavailable for viewing or reached your viewing limit for this book. 19.20. Mathematics f 2. 22. 23. 24. 28, 27. 28. (a) (4,0) (b) (0a) © (aa) (@) none of these ipa; a8 Well asy,,33.)5 are in G. P. with the same ‘common ratio, then the points (x,.¥).(2,23) and (t5, »3) By (@) licona straight line (©) licon sn ellipse (©) liconacircle (@) are vertices of a triangle ‘Number of equilateral triangles with y= y3(x~1)+2 and y~~ {3.x as two ofits sides, is @o 1 ©@2 (d) none of these Ifthe distance of any point P(x, ») from the origi defined as d(x, »)= Max. (lx, {y]} and dbx, ») = & (non zero constant), then the locus of the point P (a} astraight line (b) acircle (c) a parabola (d) none of these Ifa, b,c forman A. P. with common difference d (#0) and x,y,z form aG. P. with common ratio r(# 1), then the area ofthe triangle with vertices (a, x), (6, ») and (6,2) is independent of (@ 6 () r od @x Aline of fixed length 2 units moves so that its ends are on the positive a-axis and that part ofthe Tine x + 3°= O which lies in the second quadrant. The locus of ‘the mid-point of the line has the equation @) (+ 2yP y= (b) (H-BP +P HL (©) (+29 =F =1 () none of these Asstraight line through the origin O meets the parallel lines 4x + 2y=9 and 2x+y +6 =0 at poims Pand Q. respectively. The point O divides the segment PQin the ratio (@) 1:2 tb) 3:4 © 2:1 @ 4:3 Aline 4x+y~ 1 passes through the point 4(2,~ 7) and meets line BC at B whose equation ts 3x -4y +1 =0, the equation of line AC such that AB = AC is (a) 52x-+89y +519 =0 (b) S2x + 89y-S19=0 (©) 82x-+52y + 519=0(d) 89r + 52y-519=0 Let O be the origin and let 4(2, 0), B(0, 2) be two points. If P(x, ») is a point such that x» > 0 and xty<2.then (a). Plies either inside the triangle O4B orin the third quadrant (b) cannot be inside the triangle O4B (0) Plies inside the triangle OAB (@) none of these 29, 30. a 32. 3 34 38 ‘Consider the equation yy, = m(r—x,). In this equa- tion, ifm and x, are fixed and different lines are drawn, for different values of y,, then, (a) the lines will pass through a single point (b) there will be one possible line only (©) there will be a set of parallel lines (@) none of these D isa point on AC of the triangle with ver A(2, 3), BLL, -3), C-4, -7) and BD divides ABC into two triangles of equal area. The equation of the line drawa through B at right angles to BD is @ y-2+5=0 (b) 2y-x+5=0 (©) yt2e-5=0 (d) Bptx-S ICtwo points A(a, 0) and 6(— a, 0) are stationary and if ZA ~ 2B = Bin ABC, the locus of Cis (a) 2+)? + 2ay tan Oa (b) 2 -y? + 2xy tan (© 2472 +29 c0t (@) 2 -y + xy cot @= a ‘The straight line y=. — 2 rotates about a point where it cuts the x-axis and becomes perpendicular to the straight line av + by +¢~0, Then, its equation is (a) ax +by + 2a () ax—by-2a=0 (c) by tay-2b=0 @) a —by+2b=0 Ifthe point P(a?, a) ies in the region corresponding to the acute angle between the lines 2y = x and Ay ithen (a) ae (2,6) (b) ae (4,6) © a6 @,4) (@) none of these The point (4, 1) undergoes the following three suc- cessive transformations {a} Reflection about the line y= x 1 (b) Translation through a distance 1 unit along the positive x-axis © (6) Rotation through en angle “ about the origin in the anti-clockwise direction, Then, the coordinates of the final point are 11 2) te) (0,3y2) @) 6.4) Ifthe point (1, a) lies in between the lines x + y= 1 and 2(x +y) = 3, them a lies in L (by (4) (4) none of these (a) (4,3) fa) -™, 000. ©) F%,0)U (p= aa You have either reached a page that is unavailable for viewing or reached your viewing limit for this book. aa You have either reached a page that is unavailable for viewing or reached your viewing limit for this book. aa You have either reached a page that is unavailable for viewing or reached your viewing limit for this book. 19.28 Mathematics for IIT-JEE LA point moves sothat it remains the third vertex of a right-angled triangle, the ends of whose hypotenuse always remain as (a, 0) and (b, 0) traces out Ml, The diagonals of a parallclogrom PQRS are along the lines x +3y=4 and 6x—2y= 1. Then, PORS must be a TL If the sum of the distances of @ point from wo perpendicular lines ina plane is 1, then its locus is IN. The ends of arod of length/ move on two mutually perpendicular lines. The locus of the point on the rod which divides it inthe eatio 1: 2is ‘The matching grid: L@OQOO 1@OGO@ L. Ifthe points a(x, 9.2), BQ,2+a)and (x+y) ate such that AB= BC, then x,y, zare in I, Ifa line through the variable point A (k + 1, 24) meets the lines 7x +y—16=0, 5t—y—8=0, x— Sy +8=Oat B, Cand D, respectively, then AC, AB and ADare in IL, The length of the perpenciculars from the poinis (2, 2m), (enn, m+ n) and (?, 20) tothe line x e038 sin?@ ferme +ysin8=p, where p: cos I Ifthe lines ax+ 12y+ 1 =0, bx 13y+1=Oand ce + 14y+ |= Oare concurrent, then @ b, €ate in ‘The matching grid: L@®O® 1 @0O@ (a) thombus, (b) square (©) ellipse (d) circle ceE®@ @ HP &) GP, (©) AGP, @ AP. BoOoO® vV.@® © @ —— E PASSAGE! {In oblique coordinates, the equation y =mx~ ¢ represents. straight line which is inclined at an angle msinw T+ mcosw to the x-axis, where wis the angle between the axes. Ife the angle berween two lines y= m,x+e, and y= ‘mgx-+ ca, w be the angle between the axes, then tang= (i=) sin T£Gm + my) 008 w+ mm ‘The two given lines are parallel if m, = my. The nyo lines are perpendicular if 1+ (m, +m,) cos w + mm, =0. Ifthe straight lines y= m,x +e, and y= mgt + ¢ meke ‘equal angles with the axis of and be not parallel to one another, then m, + my +k mym, cos w= Owhere k= @ (b) 2 ) -1 (d) -2 ‘The axes being inclined at an angle of 30°, the slope ofthe line which passes through the point (2,3) and is perpendicular tothe straight line y+ 31 ~ Gis sy3-2 ( 293% Yow -6 3y3-2 ES (q) none of these Nar6 (a) ©) aa You have either reached a page that is unavailable for viewing or reached your viewing limit for this book. aa You have either reached a page that is unavailable for viewing or reached your viewing limit for this book. aa You have either reached a page that is unavailable for viewing or reached your viewing limit for this book. Let PAB = and ZPBA = B. inarcd, wn = FE - CA a-x i POT. and in APBC, tan B= = = 7 Given, ZPAR— PRA = 26 (constant) => @-B=20= tan(a— B) = tan 20 tanor—ta a me tan 20 T+tana tan B @ Poy t Day cot 20-0, which is the required locus. 41 (a). Since p, q are the roots of the equation a? + 2x + b= 0 2h p+q~— 7 and pg 4 ol) Also, r,s are the roots of the equation a’ +2h'x+b'=0 rts—- > and rs— on) Let C divides AB in the ratio a: 1. Then, r= S242, Bor al r=@ Let D divides AB in the ratio B : 1 Ba+p pos Then, 3 ~ FP po Bo Given, a+ B=0 > it 0=(p +9) (r+s)—2pq- Irs =0 2H i (-2\- \-2 24" <9 (Using (1) and (2)) => ab!+a'b=2hh'. 42(c). LetP=(x,,y,)and ZAPB= 6 Psy) A(4,2) ‘B(2,4) cg (PAY +(PBY ~( AB Then, cos 9= C28 CO Coordinates and Straight Lines 19.33, Since cos OS 1 (Pay-+ (PB) (AB? 2PA. PB (PAY + (PBY (AB) S2PA- PB (eA—PBy< (4B? | PA-PB|s| AB |= (4-2)? +(2-4)? | PA-PB| S242. Maximum value of | PA ~ PB | is 2/2 when @=0 P lies on the line AB as well as on the given line. “equation of AB is Zo =2eos B+ sin AG 72008 Otsin @ and, (~5+AD cos 6) ~(-4+ AD sin @)-5=0 6 i © = cos o-sing aD 15 10 Y _( 6 sine (15) (12) (so) wees (cos +3 sin 6 + (2.cos + sin = (cos @~sin 0? = 4080+ 9sin?@+ 12 sin @cos 0=0 => (2c0s +3 sin 6)"=0 = 2cos 9=-3 sin @ aa You have either reached a page that is unavailable for viewing or reached your viewing limit for this book. aa You have either reached a page that is unavailable for viewing or reached your viewing limit for this book. aa You have either reached a page that is unavailable for viewing or reached your viewing limit for this book. 19.38 Mathematics for IT—JEE (Ryn 4 OnE” By Wa 4, a,= ol mel" aay" "ayn! 32" 200227 > n>7 F089 10, 3 (@, d, c,d). The equation of the line in parametric formis SOS 3 4 cord sind Any point on this line is (3 +r cos @ 4+ sin 6) lies on.x=6if3 + rcos 0= 6 => r=3 sec 0 2 PR= ‘Again, the point lies ony =8 if +r sin @=8 r= 4 cosec @ or PS= 4 cose 6 4 (b, c). Let the equations of the lines AB and AC he Sxt4y=5 aA) and, dey 1S of) see @ Fa. 8 © (1) and (2) intersect at 4 = (3, ~ 1). [obtained by solving (1) and (2)] Equation of the line BC, passing through P (1, 2)is yo2e=mG@-Iory=m(ae-042..Q) 213m 8+ 1m slvins and (3) 6 Solving @) and 3), we get C= (224, Ste) cng ath og 48-3, 21) Am+3' 443. (946m)? +(12+ 8m)? (4—3my 8m =12)? + (6m 9)? (m+3)? => (100m? + 300m +225) (4m +3)? = (100m? + 300m+ 225) (4— 3m)? => -25(2m +3) [4m +32 -(4-3my]=0 = 2S(Qm +37 (m7) (Fm—1)=0 3 When Sor) or 3x 2y7=O which passes through 4 (3, -1) and hence it cannot be line BC. When m =~ 7, equation of BCis y= —T@x—1) =2 on Tyty-9=0 3 equation (3) becomes y and when m=, equation of BCis gan 1 =F WN) 2orx—Iy+ 13=0 Hence, equation of BCisx~ Ty + 13=Oor T+ y-9=0. 5 (a,c). The equation ofthe bisectors of the angles between the given lines are 3e-4 12x Se +6 {any +E52 4-4? > BSx—52y-91) = + (604 = 25y +30), Taking the negative sign, weget 99r—71y-61 Taking the positive sign, weget 2Ir+27y4121=0 22) ‘The required lines are the lines through (4, 5) and paral- lel o the bisectors of the angles between the given lines ‘The equation of a line parallel to (1), is 99r—TIy + ky'= 0 3) This passes through (4, 5), 99 (4)—77(5) + & = 0-94 =— 11, Putting k, =~ 11 in (3), we get 99: Ty-11=0 or 9x-Ty-1=0. Thisis one of the required lines. The equation of a line parallel to (2), is 2s + Iy+k,=0 24) This passes through (4, 5) 844135 +k, =O ky Putting k =~ 219 in (4), we get Die + My-219=0 or txt W-73=0, which is the other required line Ay aa You have either reached a page that is unavailable for viewing or reached your viewing limit for this book. aa You have either reached a page that is unavailable for viewing or reached your viewing limit for this book. aa You have either reached a page that is unavailable for viewing or reached your viewing limit for this book. aa You have either reached a page that is unavailable for viewing or reached your viewing limit for this book. aa You have either reached a page that is unavailable for viewing or reached your viewing limit for this book. aa You have either reached a page that is unavailable for viewing or reached your viewing limit for this book. 28, 26. 2. 28. 29. 30. a} 2h yy +o? | ex? -2he, , +57? | ‘The area of the triangle formed by the lines ax? + hay + by" = 0 and lx +my + n= Ois an? = 2hin + 61 2 fae m? fi? = ab > (d) none of these am? = 2hln + BI © @) (@) am? + Zhim + br © Iftwo of the straight lines represented by ax° + bxy + exy? + dy? = Oare at right angles, then @ @+actad-é=0 () aP+achd+P=0 © @-act bd+. (@ P+actbd+d=0 fone of the two lines given by ax? + 2hxy + by? =9 may be perpendicular to one of the lines given by a'x? + Dhixy + b'y?=0, then (@) 2 (ha"s hb) (Ifa + Hb") + (bb'— aa") (©) 4 (ha'= hb) (hva hb’) (bb"~aa) (©) 4 (ha’=h') (hia hb" )+ (bb'~ aa (@) none of these If the distance of @ point (x,, y) from each of wo straight lines, which pass through the origin of coordinates, is 6, then the two lines are given by @ (yy-yPaF OF+y7) ©) (yy tay) P= F OAH) © yo 8 OF +9) (d) none of these Ifthe lines ax? + 2ixy + by! = 0 be two sides of parallelogram and the line ++ my be one of its 4disgonals, then the equation of tho other diagonal is (@) »(am—hd)=x (bh) . (b) y (bh) =x (am—Al) . (6) ¥(am+ hl) =x (61+ hm) (@) mone of these The orthocenire(x’,»") ofthe triangle formed by the straight lines ax? + 2ixy + by? = 0 and the line Txt my= Lis givenby ath 2him + oF ath @= y Meret OT ante Dh FoF 31. 32. M4. 35. 36. mam? —2him+ bE (none of these Ifa triangle has two ofits sides given by the equa- tion ax? + 2hxy + by? =0 and the orthocentre given by (c,d), then the equation of the third side is (a) (a+b) (ex+ dy) = ad —2hed + be? (b) (a+b) (cx+ dh) = ad? +2hed + be? (©) (a+ b)(ate* cy) = ad? + 2hed + be (© none of these ‘The equation of two stright lines throug the point (y.¥,) and 1 to the lines given by ax? + 2ixv + by?= os @ bern)? -2hex—m)O—H) Fa—yF () b=x,P +2hE=n)O=y) FaG=yP=O (© a(e=x))2-2h(e—x)O~ 3) +bO-yP=0 (@) none of these ‘The equation of two straight lines through the point (yy) and parallel to the lines given by ar? + 2hxy + by =dis (©) aCynyy Pt 2h Geary) + bG =a PO ) ayy, P= 2h =x) (y=¥) + bP =O © b(y-y P+ 2h x) y-y) tax P=0 (6) none of these A point moves so that the distance between the feet, ‘ofthe perpendiculars from iton the lines ar= + 2haxy-+ Jy? Die a constant "20, The equs (@) GP +P) GP ~ ab) = [la bP +40] () (2+)? (2 +ab)=2 [la—b? +482] (©), @? +97)? -ab) =F [a5 49] (@) none of these ofits locus is The conditions that two of the lines represented by the equation ay4 + bay? + ex'y2 + dtp + ex 0 may be at right angles, is (a) (¢+ate)(e~a)’+(b+d) (ad +be)=0 () (cat e)(e—a-(b +d) (ad +be)=0 (© CHate)leray+(o+a)ad+be)=0 (© none of these Ifthe equation ax? + 2iay + by? + 2gx + 267 +e =0 represents two straight lines, then the square of the distance oftheir point ofintersection from the orginis. clath)+ fre ab— @ aa You have either reached a page that is unavailable for viewing or reached your viewing limit for this book. aa You have either reached a page that is unavailable for viewing or reached your viewing limit for this book. aa You have either reached a page that is unavailable for viewing or reached your viewing limit for this book. aa You have either reached a page that is unavailable for viewing or reached your viewing limit for this book. aa You have either reached a page that is unavailable for viewing or reached your viewing limit for this book. aa You have either reached a page that is unavailable for viewing or reached your viewing limit for this book. aa You have either reached a page that is unavailable for viewing or reached your viewing limit for this book. aa You have either reached a page that is unavailable for viewing or reached your viewing limit for this book. aa You have either reached a page that is unavailable for viewing or reached your viewing limit for this book. aa You have either reached a page that is unavailable for viewing or reached your viewing limit for this book. aa You have either reached a page that is unavailable for viewing or reached your viewing limit for this book. aa You have either reached a page that is unavailable for viewing or reached your viewing limit for this book. 21.8 Mathematics for lIT-JEE Explanation Equation of the fine is y= me +e -) Equation of the cirele is? ~ 2 = @ 2) Suisttuting the value of y from (1) in (2), wehave x +(me+ oP =a" oy athe Imex tm oP oy (1+ m?)x? + 2mex +e? a =0 (3) which is a quadratic ins. the ine (1) iva tangent to the etele (2), then (3) must have coincident roots. Discriminant of ( 41 tn?) fc? a) = 0, of, =P (tm) or c= tayl +a ‘which is the required condition for the line (1) to be a tan- gent tocitele (2), * A line will touch a circle ifand only if the length of the perpendicular from the centre of the circle to the line is equal wo the radius of thee © The condition thatthe line fv + my + n= O touches the circlex? +y? +2gr+ 2fr+e=0is (ig + mf —myP=(P 4 m2Vg2 =f? —c). # Equation of tangent tothe cirele x? =)? + 2gv +3f in terms of slope is y = mx + me — f* gh +f?) Jaen) © Ifthe line fe + my + m= OP + 0A Tangents from a Point Outside the Circle Working Rule 1. Let the pointe (x), 9). Write the equation of a straight line passing through the point (¥),»4) and having slope mic, (yy) =m =x) =) 2. Find the length of the perpendicular from the centre of the circle tothe line(1) and equate ito the radius of the circle. Call this equation as (2). 3. Obtain the value of m from the Eq. (2). 4. Substitute this value of min equation (1) toobisin the required equation of tangent. Length of the Tangent from a Point to a Circle ‘The length of the tangent that can be drawn from the point P(xy.¥)) to the circle S=Ois /5, , where the coeificients. ‘of x? and )? in the equation of the circle are unity. Thus, the length of the tangent from the point P (x,,.,)to circle x? + y+ Dget2fy> iven by gy t2fy te = fS, Explanation the equation of the circe is tye tay te=0 ‘The centre Cis (-g, =f) and ndius= T= fg? + FP -e Also, CP= JG, + ° +O) +/)* But CT LPT ++ Radius 1 tangent] ZCTP~ 942 By Pythagoras’ theorem, CP = Cr+ Pr a, | PP =CP cP xy taht (a th? ey = sf ey? +2ee; +2 +6 o, P= yxp +07 +2gx +2, +e. which is the required length of the angen. | TRICK(S) FOR PROBLEM SOLVING + While calculating the length of tangent using the above formula, itmust be noted tha the coefficients of x? and »? mustbe unity © Ifthe point lies insidethe circle, then S, < Oand\7'| is imaginary therefore we cannot have any tangent froma point inside the circle. # Ifthe point lies on the circle, then S, = 0 and hence || = 0. So, ifthe point lies on the eicle, then we have only one tangent having zero length. + Ite point lies ouside thecitel, then 5 isfinite, ‘ cauTiON EE No tangent ean be draven to a circle from a point lying within the circle Jand|P | Normal to the Circle at a Given Point ‘The norinal 10 a circle at any point is a straight line perpendicular to the tangent at the point and always passes through the centre of te circle 1. Equationof normal ‘* The equation of normal to the circle x° +)? + 2ex + 2p +e = 0 at any point (xy, y)) is yy, = BT gop gp Ae SM ate ate outst + The equation of normal to the eizcle x2-+y2 aay point (xy.34) isa) —",7 aa You have either reached a page that is unavailable for viewing or reached your viewing limit for this book. aa You have either reached a page that is unavailable for viewing or reached your viewing limit for this book. aa You have either reached a page that is unavailable for viewing or reached your viewing limit for this book. aa You have either reached a page that is unavailable for viewing or reached your viewing limit for this book. aa You have either reached a page that is unavailable for viewing or reached your viewing limit for this book. aa You have either reached a page that is unavailable for viewing or reached your viewing limit for this book. aa You have either reached a page that is unavailable for viewing or reached your viewing limit for this book. aa You have either reached a page that is unavailable for viewing or reached your viewing limit for this book. aa You have either reached a page that is unavailable for viewing or reached your viewing limit for this book. aa You have either reached a page that is unavailable for viewing or reached your viewing limit for this book. aa You have either reached a page that is unavailable for viewing or reached your viewing limit for this book. aa You have either reached a page that is unavailable for viewing or reached your viewing limit for this book. OM=OP=a. +s equation of the circle is 1? —2r Reos (@- a) + R= a? = ol) From the right-angled triangle OMO’, Reosa=e a) and, Rsina=a 0) Recta wd) Substituting these values in (1), we have P—2r (cos 0+ asin 8) +0? =0 => P—2ercos 8+ 2 =2ar sin @ 43) “> itpasses through P (ry, 6,), 2. F-2er, cos 8, + = 2ar, sin B, (6) Dividing (5)by (6), we have 2 ‘i cr cos +e 27, C088, +e" 2ercos@ +e rsin€ ‘which isthe equation ofthe circle in polar form. 1. Oisa fixed point, Pis any point on a givencircle. OP is joined and on it is taken a point Q such that (OP.00 = k, where kis aconstant. The locus of Q isa (a) circle (b) straight line (©) parabola (@) none of these 2. Inthe above question, if the point O is taken on the ‘original circle, the locus of Qis a (@) circle (b) straight line (©) parabola (A) none of these 3. Oisa fixed point and P any point ona given straight Tine; OP is gined and ont istaken a point Q such that OP. 00=P. The lecus of Qisa {a)_ circle passing through O (b) citele which does not pess through O (©) a straight line through (@) a straight line not passing through O E PASSAGE II Leta straight line be drawn from a point P tomeet thecircle in Qiand R. Let the tangents at Qand R mectat 7. The locus of Tis called the polat of P with respect to the circle, ‘The given point Pis called the pole of the polar line. y Y “ih, 8) OTT Pony) Bs Q Ply ri) x a x x rr 3 Circles 21.28 Let P(x),3) be the given point lying outside the circle in ig. ()) and inside the cirele in fig. (i) Through P, draw a line tomeet the circle in Qand R. Let the tangents to the ciccle at Qand R meet in 7,4). Itis required to find the polarof P, .¢.,the locus of 7. Equation of QR, the chord of contact of the tangents, drawn from Ttothe circle x* ° is iy “= (1) passes through P(,9)), phy k= a, the locus of (A, A) is xr, + yy, = a2, Which is the equation of polar of P. 4. Ifthe polar of P with respect to the citcle x? +3? =a? touches the circle (x = f° + (y~ g} = #, then its locus is given by the equation @) (fx tara mai 24s) ) (fxtey-@ P= be +s) © (fr-gy-a=a (P47) (@) none of these S. The pole of the line 3x + 4y= 45 with respect to the cirelex? +y? 6x8) +5 =0is (2) 68) (b) (6,-8) © 6%) (d) 6-8) 6. The pole of the chord of the circle x? + y?= 16 which is bisected at the point (~2, 3), with respect to the circles a2 48 w (2.8 © 3) 2. o (24) © (FB) nore ormae BB 7. The coordinates of the poles of the common chord of the citoles +)? = 12and x°+y2—Sx+ 2y-2=0with respect to the circle x2 ~y?= 12 are 12 @ (s ) (- 2) (d) none of these PASSAGE A system of circles, every two of which have the same radical axis, is said to be coaral. The simplest equation ofa coaxal system of eitcles is 2 0 where g is different for different circles of the system and ¢ is the same forall the circles, the common radical axis being the axis of y and the line of centres, the axis of x. The equation of a system of coaxal circles, having given the equation of radical axis and one circle ofthe system S = Gis, Payts2gr aa You have either reached a page that is unavailable for viewing or reached your viewing limit for this book. aa You have either reached a page that is unavailable for viewing or reached your viewing limit for this book. aa You have either reached a page that is unavailable for viewing or reached your viewing limit for this book. aa You have either reached a page that is unavailable for viewing or reached your viewing limit for this book. aa You have either reached a page that is unavailable for viewing or reached your viewing limit for this book. aa You have either reached a page that is unavailable for viewing or reached your viewing limit for this book. aa You have either reached a page that is unavailable for viewing or reached your viewing limit for this book. 1 5 86 4 (=384) 20 (a, b,c). Mukiplying by r,:he given equation becomes A= |reos 0 0) In Cartesian form equation (1) can be written 2s Paty=|x] = Pty strifred and Py? =-xife<0 Equation (2) represents circle of ra and centered w ($0) sain namaste te! L cenered at |. 0 Herc. svenequtionepeses two circles touching each other atthe origin, 4.(13). The equations ofthe circles are tya4 A) and, x24)? 2de— My ta2=0 Q Centre of (1) is = (0,0) and radius r, =2 Centre of 2) is C, =(12, 5) and radius ‘d= distance between centres = VIF ES = 13, the two circles have exactly two common tangents then 169-@?>Oandr, +r, >d (a-13)(a+ 13)<0 and 2 + ¥169-a? > 13 ~13 121 = 13 |1/<8. afr. 1 3(4), ‘The centroid of an equilateral triangleis the centre of its ciroum cirele and the radius of the circle isthe distance of any vertex from the centroid, i.e.,radiusof the circle = distance of centroid from any vertex 2 -3 (Median) = 5 Ga) =2a, 3 Hence, equation of circle whose centre is (0. 0) and radius 2a is (x-0F +(y-0) =Qay oor at ty = 4a", 4.(2), Let the lines cuts the x-axisat 4 and B, then 1 0. and OB a Also, if the lines cut the y-axis at Cand D, then 3 OC=1 and OB= 2 Now, if the circle passes through A,B, Cand D then. =ocxop 3 {-+)-3=1«3 04% 08=0C*0D (-4)« yeixd =A=2, 5 (22). AB?= 44a = {a B Since the traingle is equilateral, therefore centroid, orthocentre, circumeentre and ineentre all coincide. aa You have either reached a page that is unavailable for viewing or reached your viewing limit for this book. aa You have either reached a page that is unavailable for viewing or reached your viewing limit for this book. aa You have either reached a page that is unavailable for viewing or reached your viewing limit for this book. aa You have either reached a page that is unavailable for viewing or reached your viewing limit for this book. aa You have either reached a page that is unavailable for viewing or reached your viewing limit for this book. aa You have either reached a page that is unavailable for viewing or reached your viewing limit for this book. aa You have either reached a page that is unavailable for viewing or reached your viewing limit for this book. aa You have either reached a page that is unavailable for viewing or reached your viewing limit for this book. aa You have either reached a page that is unavailable for viewing or reached your viewing limit for this book. aa You have either reached a page that is unavailable for viewing or reached your viewing limit for this book. aa You have either reached a page that is unavailable for viewing or reached your viewing limit for this book. aa You have either reached a page that is unavailable for viewing or reached your viewing limit for this book. aa You have either reached a page that is unavailable for viewing or reached your viewing limit for this book. aa You have either reached a page that is unavailable for viewing or reached your viewing limit for this book. aa You have either reached a page that is unavailable for viewing or reached your viewing limit for this book. aa You have either reached a page that is unavailable for viewing or reached your viewing limit for this book. aa You have either reached a page that is unavailable for viewing or reached your viewing limit for this book. aa You have either reached a page that is unavailable for viewing or reached your viewing limit for this book. aa You have either reached a page that is unavailable for viewing or reached your viewing limit for this book. aa You have either reached a page that is unavailable for viewing or reached your viewing limit for this book. aa You have either reached a page that is unavailable for viewing or reached your viewing limit for this book. aa You have either reached a page that is unavailable for viewing or reached your viewing limit for this book. aa You have either reached a page that is unavailable for viewing or reached your viewing limit for this book. aa You have either reached a page that is unavailable for viewing or reached your viewing limit for this book. aa You have either reached a page that is unavailable for viewing or reached your viewing limit for this book. aa You have either reached a page that is unavailable for viewing or reached your viewing limit for this book. aa You have either reached a page that is unavailable for viewing or reached your viewing limit for this book. aa You have either reached a page that is unavailable for viewing or reached your viewing limit for this book. aa You have either reached a page that is unavailable for viewing or reached your viewing limit for this book. aa You have either reached a page that is unavailable for viewing or reached your viewing limit for this book. aa You have either reached a page that is unavailable for viewing or reached your viewing limit for this book. aa You have either reached a page that is unavailable for viewing or reached your viewing limit for this book. aa You have either reached a page that is unavailable for viewing or reached your viewing limit for this book. 22.50 Mathematics for IIT-JEE ML. 2 13, 14, 15, 16. Ifa>2b>0, then positive velue of m for which y= mx ~by1+m? isa common tangent to x? +y? = 6? and (ah +P =e is 2b ® o la? — 45> oS @ If P= (xo), F, = G, 0), Fy = (3, O) and 16x? + 25)? 100, then PF, +PF, equals @s () 16 © 10 @ 2 ‘The number of values ofc such that the straight line y 2 = 4x etouches the cue +) = Lis @ 0 (b) 2 @1 (@) infinite ‘Tangent is drawn toellipse $= + y? =1 at(3 V5 cos 8, a sin (where @e (0, /2)). Then, the value of @ such that the sum of intercepts ‘on axes made by this tangent is minimum, is z x @ F OF z z © 8 @ a a 18, 19. Ifthe circle x + y* = a? intersects the hyperbola xy =c? | 2x in four points P (x,,¥4).O Ca.¥3)s R ys)» Sky) then @ xtytyty=0 © dityarystyn0 © 23% @ yyrIs¥e 2 oe Ire, istheeccentrcity of the ellipse == +3 is the eccentricity ofthe hyperbola passing through. the foci ofthe ellipse and e,¢, = 1, then equation of the hyperbola is 2 and : 2 @ 3. 4 @) %@ 10 @b) 1. @ 120 7) 18) 19, 20.) | | | a 5 2B. 21. o 14 © 922 9 25 CLEC SERGE ERE HS (© none of these 9+ 6=5 , be two points on the hyperbola = = 1.16 Gy Bt the pot of tho ination ofthe normals at P and O, then kis equal 10 © ae @ (24) isthe chord of contact ofthe hyperbola? —y? 9, then the equation of the corresponding pair of tangents is (@) 9x? -8y7+ 18r-9=0 (b) 9x28)? 18e+9=0 (©) 9x?- 8)? 18r-9=0 (d) 9x78)? 18r+9=0 Ifthe line2x-+ VJ6y = 2 touches the hyperbolax? —2y2 = 4, then the point of contact is (@) ©2,¥6) (b) 5,26) © (3 - ) @ Wy The equation ae: Foren lt represents (@) anellipse ©) ahyperbola (c) acircle (@)_ none of these For hyperbola == |. which of the z following remains constant with chengein “a"? a) abscissae of vertices (b) abscissae of foci (©) eccentricity ) 6 © sin?@ @ directx © @ (b) 15. (bed) 16. tb) aa You have either reached a page that is unavailable for viewing or reached your viewing limit for this book. aa You have either reached a page that is unavailable for viewing or reached your viewing limit for this book. aa You have either reached a page that is unavailable for viewing or reached your viewing limit for this book. aa You have either reached a page that is unavailable for viewing or reached your viewing limit for this book. aa You have either reached a page that is unavailable for viewing or reached your viewing limit for this book. aa You have either reached a page that is unavailable for viewing or reached your viewing limit for this book. aa You have either reached a page that is unavailable for viewing or reached your viewing limit for this book. aa You have either reached a page that is unavailable for viewing or reached your viewing limit for this book. aa You have either reached a page that is unavailable for viewing or reached your viewing limit for this book. aa You have either reached a page that is unavailable for viewing or reached your viewing limit for this book. aa You have either reached a page that is unavailable for viewing or reached your viewing limit for this book. aa You have either reached a page that is unavailable for viewing or reached your viewing limit for this book. aa You have either reached a page that is unavailable for viewing or reached your viewing limit for this book. aa You have either reached a page that is unavailable for viewing or reached your viewing limit for this book. aa You have either reached a page that is unavailable for viewing or reached your viewing limit for this book. aa You have either reached a page that is unavailable for viewing or reached your viewing limit for this book. aa You have either reached a page that is unavailable for viewing or reached your viewing limit for this book. aa You have either reached a page that is unavailable for viewing or reached your viewing limit for this book. aa You have either reached a page that is unavailable for viewing or reached your viewing limit for this book. aa You have either reached a page that is unavailable for viewing or reached your viewing limit for this book. aa You have either reached a page that is unavailable for viewing or reached your viewing limit for this book. aa You have either reached a page that is unavailable for viewing or reached your viewing limit for this book. aa You have either reached a page that is unavailable for viewing or reached your viewing limit for this book. aa You have either reached a page that is unavailable for viewing or reached your viewing limit for this book. 248 Mathematics for IT=JEE Trigonometric Ratios of Submultiple Angles where, A¢ (n+ Ie (An Angle of the form 2, ne 2) w -2 ” os 1. sin 6 = 2sin& gos 2 = 2002 I+c0s. . 20° 2 1+tan?@/2 » Ae sind where 4 # 2nt The ambiguities of igns are removed by locating the quadrant sgt A in which + lies ot cot? S-1 you can follow the 4 cot= —2— Adjoining figure. ne coe = 140050 "2 10088 10, 16080 = ogy 2 sind 2 +0) = Vivo{ 03 TRICK(S) FOR PROBLEM SOLVING «fein seo] cort unt > a,>a, a) For I >uan E>0 ay < land a; >1 2.4, x (16 cos®8+9.cos*@— 24 cost) +» cos'd =9e0s70+ 16 cose = 9re0s?8+ (y~ 2dr) cos*8+ Lor cos’ =9 0370+ 1660380 On comparing, we get 9x=9 and y—24r=0 x= Land y=24, 4 (a). | tan @ + sec 6] =|tan 6| + | see 8 | only if sec @ and tan6 both have same sign sind ie, sec@:tan@>0 or 20 cos 6 = sin @>0 and cos 60 = ve wni-{2} : 5 (a). We have, sin8, sind and cos@ are in G.P. a 160820 _ sin26 = sin?g~sin cos =» S520 - ‘o> sin Bcos @ 7 . = 0529 1-sin 2620. Now, the discriminant of the given equation is sooty See sae 2) =4c0s26~cosee?920 ( 2 Roots of the given equation are always real. of (i) 8 (a). We have, cos 50° = cos?25° ~ sin?25° = (cos 25° + sin 25°) (cos25° sin 25°) = k(c0s25°—sin 25°), But (cos 25° + sin 25°)?+ (cos25° —sin 25°)? = 2 bP = cos 25° sin 25 aa You have either reached a page that is unavailable for viewing or reached your viewing limit for this book. aa You have either reached a page that is unavailable for viewing or reached your viewing limit for this book. Trigonometic Ratios and Identities 24.23 4 (1). Since sin A, cos 4 and tan 4 are in GP. we have cos? A=sin A tan 4 = cos A an 2 (1448). We have, cos 1 [psi Eeos% van Zeort® 2sin—| + 2sin EZ cos % 785 3 (648). We have, sin (1 €0s @)= cos (7 sin 6) ~=sin (Frasne) e010 F + wsin 09 n(cor O-sin d= F 1 > cos a-sino= 5 = cos 0- cos F—sin @-sin ts (o5 4 (4). We have, (ean pve? =i ana) 4 (en -fe?t} 2 +(omny-iF Fitna) 20 3 tLe +1) (un? a an? B+ tn? —fetanocs Ja? 1 tan + + ttany} 20 5 raed do? => tan? aban? B+ tan? y= Ba 5 (966). We can write the given equations as sin a—8 sin d= 4sin ¢~7sin 6 cos a— Neos d= 4 cos e—7 cos b Squaring and adding, we get 1+ 64-16 [sin asin d+ cos a cos d] = 16 +49- 36 [sinb sinc~ cos c0s c] = 65-16 c0s (a-d)= 65-56 cos (6-2) > 2eos(a-d)=7 cos (5-0) Teos(b-0) ‘cosa—d) 6 (1). We have, cosec 6 ~ sin 8 = a? ay and, sec 0 cos 6= 5 2) l From(1), => O: Sa 2, ~ el ee ere] +3) sind 1 5 From(2), <=> —cos 6= 8 cosé 1=cos?@ = 030 = sin?@= 5 cos @ (4) From (4), cos 8= = ‘ From (3), EP sin 0 anh e=a oF sin 9= ab? (3) 2 cos = 28% 16) Squaring and adding (5) and (6), we get sin?@~ cos'@ = a0! + ao? ~ 1 = a8 +BY), 7 (0). We have, tan 6 tan (9 + 60°) + tan O tan (9~ 60°) + tan (0 +60°)tan (9-609) +3 = [tan tan (0+ 60") +1] [ten Bian (060°) + 1] + {tan (8+ 60") tan (860°) ~ 1] _ £0810 -8-60) | c0s(0-8 +60) £€050.c0s(+ 60°)" c0s6 cos 6-60") |, 2080 +6086 +60") ©0s(0+ 60°) cos(@ = 60°) 24.24 Mathematics for IT-JEE cas6?c0s (8 ~ 60%) + cas 6lc0s (9 +60") + 208120%eas 4 056 cos(6 + 60" )oos(@ 60°) [cos (8 60") 4 cos(@ +60°}] sin 30°cos8 (C0880 (0 + 60") C05 (8 = 60°) 11 fa). Wehave, $ a, cosko = azo = g( 280 = sind \ = 6sin Osin 38-2 sin?34 =3icos 26- cos 48)~ (I ~c0s 66) =1+3.0s 28-3 cos 48+ cos 68 5 16 (a). We have, aves d= poo 4432 | =you( 4+ Dividing by oxBy, we get B+ By+ yee 442 cos( tet 3 3 |sn9 1 sansa devs +2eos(x + A) cost] kl 5 l pleas ~c08-4] =O, I (b). Since the equation a, +a, sin x+a,cos.x~a, sin 2 +a,c0s 2 =0 holds for all valuesof x, atat (on putting x=0) aja, +as=0 (on putting x=) = a,=Oanda, +a,=0 ay Putting x= 1/2 and3/2, we get a, +4,—a,=Oanda,~a,~a,=0 = a,=Oanda,-a,=0 Q) (and (2) give | a a a ‘The given equation reduces to a, sin 2x = 0, This is true for all values of x therefore. a,=0 Hence, a,= ay 0 n= a= a ‘Thus, the number of S-tuples is one IQ). TH (sin2px— 10052 pa) Bei = 0!" TH e082, iN!" f} (costpe + isin2pm) A : 2e0s0 0960" —sin 30° e030 038 cos (0+ 61°) eos (O60) J (eos -c036) ‘cos@ e0s(0+ 61" jeos(6- 60") =aglrt =-1(cos 10m! sin 10m) =—1 21(@). Given, sin 8= 3 sin(9= 227) = sin(0+a~a@)—3sin(@+a+a) => sin(@+ a) cos @—cos (8+ a) sin & =3sin (6+ @) cos @+3 cos (0+ a) sin a. => —2sin (9+ @) cos = 4 cos (@> a) sin @ in (8 + oF} in cos(O +a) > cose => —tan(6+a)=Jtane@ => tan(O+ a)+2tan a= 0. M(b). We have, psin +a cos 0) and, pcos @-qsinO=b Q) Squaring (1) and (2), and then adding, we get [psin 0+ cos 8 +(p cos 6 sin 6? = PU+@(l)—a_B=0 = (Pa) (F-F)=0 = (pta)(p—e)+(g+5)(q-b)=0 pita g-b qth p-a moe. St UI (a). sin sin sin (sing eT Tg % ~enlsfymnf)n*) Wr _ dt Also, cosT7 = cost Sr oe 30, c0s%/ c0s28/ cos!04/ ~ sin sin = sin ween ee GF =i de aa You have either reached a page that is unavailable for viewing or reached your viewing limit for this book. aa You have either reached a page that is unavailable for viewing or reached your viewing limit for this book. aa You have either reached a page that is unavailable for viewing or reached your viewing limit for this book. 25.4 Mathematics ‘or | EE f VIN © Make sure that the answer should not contain any |___TRICK(S) FOR PROBLEM SOLVING SUS EGE ERUEEEMSOIVING value of unknown “6° which makes any of the Solutions of Basic Trigonometric Inequa ‘© Squaring should be avoided as far as possible. If squaring is done, then check for extra solutions For example, consider the equation sin 6+ cos @=1 ‘On squaring, we get 1 +sin20 =1 or sin20=0 > 6 alee 3n “ee The values of the angle, @ = and @ not satisfy the given equation, So, we get extra solutions. Thus, if squaring is must, verify each of the solutions. Never cancel a common factor containing "6" irom the two sides of an equation. For example, consider the equation tan @= V2 sin 4, 1 If we divide both sides by sin 8, we get cos 8= which is clearly not equivalent to the given equation as the solutions obtained by sin x = 0 are lost. Thus, instead of dividing an equation bya common factor, ‘take this factor out asa common factor fromail erms ofthe equation, terms undefined, * If tan@ or sec @ is involved in the equation, 4 should not be an odd multiple of 72, * If cot@ or cosec @ is involved in the equation, @ should not be a multiple of = or 0. # Thevalueot J7(B) is always postive. Forexample, Voos?@ =| cos @) and not + cos @ ‘Al the solutions should satisfy the given equation, and lie in the domain of the variable of the given equation, ‘© Check the validity ofthe given equation, e.., 2c0s6 + siné = 3 can never be true for any @ as the valve (20s 6+ sin 6) can never exceed 2? +1* = v3, so there is no solution for this equation. caution Ithe argument in inequalities is not x but.x+ aor ax cor * then always write general solution first in terms of this argument and then find x ‘Table 25.2 Inequality sin v> where |r| < 1 nx< a whore || <1 cos.x> a where (04| <1 cos x < 0, where | or) <1 tan x> ez, where co < a < co tan x < @, where -co @, where -20 < or < co cot x< a where 20 < a <2 Solution (u ¢ Z) sin a+ Ving sin’2r=2(1-a)=2(1-a)S1 and, 2(1-a)20= 12Sasi. 3.(¢,d). The equation holds if | cos x ie, ife=nanel 23 If] cos x # 1,then sin? x ->sinx as 1 = sinx=lor = 2 sin x# 1,as in that case cos.x=0 sin. ed =nn+ cay" = 2 rears 2 4 (a, b). We have, spinzette e .gfsinzeszoet we Binetteoe 4 Ben 2ee2e08?n) Lety= 3i02s2e0t« . yt Pars v => -2Byt2I=0Sy=27 oF 1 Ity=27sthen e220 33 => sin 2x+(2cos*x—1)=2 = sin2x+cos2x=2(1) Dividing both sides by V2, we have sin(20+2) 2 which is not possible Now if = FPSO | 9 sin dx +2 cox =0 > Zoos x(sinx + c0s.x)=0 cosx=0 or tan x 5 (a,c). We have, 2cos 26+ J2sin@ =2 => Visin@ =2(1-cos26) = Visiné =4sin'@ = JIsind ~4 sin’ = Visin@ [1-22 sin?’29] =0 Ww Jisind =0, sin @=0 = O=nmnel. 3 2 Ww 1-2 V2 sin? @ =0, sin? @ [VE =052=0) 22 i =sin = = O-nmt oly Fane. 8=nmnn> CI)" = net 6 (a, b). ‘The given equation can be writien as Tc03?@ sin? @ Asin 40 -! 1 0 }=o 0 at [Applying R, > R, -R, and R,—> Ry~ By) 2 sin? 4sin 40 (Applying ©, > C+ Ca) ane = 2+ 4sin40=0= sina = 40=nx+(-1)" (-E]o- Feeney. We have to choose values of 68.1.0 <0 < = 7 (a, d). We have, 2577 +. = sinxtcosy=0 A) and, 16st 00s! sin?s + eos?y Eliminating cos y from (1) and(2), we get +2) 1 When sin.x= 5. cos y 008 (7/3) = cos (wm!) x= amt (- 1)" (26), y= Inet (213) J coss=4 7 2 c= n= (- 1)" (w16) and y= Ines (23), When sin x: aa You have either reached a page that is unavailable for viewing or reached your viewing limit for this book. aa You have either reached a page that is unavailable for viewing or reached your viewing limit for this book. aa You have either reached a page that is unavailable for viewing or reached your viewing limit for this book. aa You have either reached a page that is unavailable for viewing or reached your viewing limit for this book. aa You have either reached a page that is unavailable for viewing or reached your viewing limit for this book. aa You have either reached a page that is unavailable for viewing or reached your viewing limit for this book. aa You have either reached a page that is unavailable for viewing or reached your viewing limit for this book. aa You have either reached a page that is unavailable for viewing or reached your viewing limit for this book. aa You have either reached a page that is unavailable for viewing or reached your viewing limit for this book. aa You have either reached a page that is unavailable for viewing or reached your viewing limit for this book. aa You have either reached a page that is unavailable for viewing or reached your viewing limit for this book. aa You have either reached a page that is unavailable for viewing or reached your viewing limit for this book. aa You have either reached a page that is unavailable for viewing or reached your viewing limit for this book. aa You have either reached a page that is unavailable for viewing or reached your viewing limit for this book. aa You have either reached a page that is unavailable for viewing or reached your viewing limit for this book. aa You have either reached a page that is unavailable for viewing or reached your viewing limit for this book. aa You have either reached a page that is unavailable for viewing or reached your viewing limit for this book. aa You have either reached a page that is unavailable for viewing or reached your viewing limit for this book. aa You have either reached a page that is unavailable for viewing or reached your viewing limit for this book. aa You have either reached a page that is unavailable for viewing or reached your viewing limit for this book. ESS gnoonud sitarnonoghT sexe! sab ch tes ts) 20s (0) © LY) aah niealaiaryal He ster aAT out ta aie a 1h tw eG ie! yr in Ow tpt 4 denyrn. wu i ° " ow fw ae i i ; a “3 sisi osamity (bi 2 iinet ca ana Ea we f Copyrighted material aa You have either reached a page that is unavailable for viewing or reached your viewing limit for this book. aa You have either reached a page that is unavailable for viewing or reached your viewing limit for this book. aa You have either reached a page that is unavailable for viewing or reached your viewing limit for this book. aa You have either reached a page that is unavailable for viewing or reached your viewing limit for this book. aa You have either reached a page that is unavailable for viewing or reached your viewing limit for this book. aa You have either reached a page that is unavailable for viewing or reached your viewing limit for this book. aa You have either reached a page that is unavailable for viewing or reached your viewing limit for this book. aa You have either reached a page that is unavailable for viewing or reached your viewing limit for this book. Properties and Solutions of Tangles 27.19 21 (a). We know that, 24 (c), HD =BD tan Z EBC (2-H +a ate b+ ct 2(@R +P2-Aa) = @ B+ cP = 2a + 2B — 2a = abies nate? @ B+) = 20 eed PP ot ae 2 2Reos Acos BeasC = B= 45° or 135° > BR cos Boos C= SATE 22 (b). We have, atan + bee 8= 6 = ccos 0 —asin@= 6 simitry, HE = *Re0s-Asos Boose 4 Let a and B be other two angles of the triangle cos B = ceos@~a sin a~ coos - asinB 2Reos Acos BeosC => (cos a - cos B) = alsin a — HF= coe > resin B 5g BHO 2 = 2acos _ ss unttB a 2sin(ZBOC) ~ 2sin2A 2 4 oD =2sin 24, ia 3 : c roe . 4 . & Ber A —— ar Se evenae Similarly, — = 2 sin2B and <= 2sin 2c 1 % a é a c 23 (b), For a right-angled triangle inscribed in a circle of So, 2+ RRR (sin 24 + sin 2B + sin 2C) -Asind sin B sin C [v A+B+C=n] “4 radius R, the length of the hypotenuse is 28. the area is maximum when the triangle is isosceles with each side = V2R c 1 = 7QN2+28 = (W2+DR = (sin A) 2 sin B) (sin ©) “a F So fRER =a LL. s-a,s-b 1 sin’ a4 ppsin naa 4 26 (a). A= —pesin= +— pbsin= : (a). ppesing +5 pbsins "ae Also, a= Locsin = besin4cos4 z 28a aa You have either reached a page that is unavailable for viewing or reached your viewing limit for this book. aa You have either reached a page that is unavailable for viewing or reached your viewing limit for this book. aa You have either reached a page that is unavailable for viewing or reached your viewing limit for this book. aa You have either reached a page that is unavailable for viewing or reached your viewing limit for this book. aa You have either reached a page that is unavailable for viewing or reached your viewing limit for this book. aa You have either reached a page that is unavailable for viewing or reached your viewing limit for this book. aa You have either reached a page that is unavailable for viewing or reached your viewing limit for this book. aa You have either reached a page that is unavailable for viewing or reached your viewing limit for this book. aa You have either reached a page that is unavailable for viewing or reached your viewing limit for this book. aa You have either reached a page that is unavailable for viewing or reached your viewing limit for this book. aa You have either reached a page that is unavailable for viewing or reached your viewing limit for this book. aa You have either reached a page that is unavailable for viewing or reached your viewing limit for this book. aa You have either reached a page that is unavailable for viewing or reached your viewing limit for this book. aa You have either reached a page that is unavailable for viewing or reached your viewing limit for this book. 288 Mathematics for IT-JEE 4 A bird is perched on thetop ofa tree 20 mighandits elevation froma point on the ground is 45°, It fies off hotizontally straight away from the observer and in ‘one second the elevation of the bird is reduced to 30°, ‘The speed of the bird is approximately... m/s ‘The angle of elevation of the top ofa tree at point B due south of itis 60° and ata point C due north of itis 30°. D is a point due north of C where the angle of 5. elevationis 15°, If V3 = 1— and BCx CD= 2? «x i 19 * 11, the height of the tree is.. The angle of elevation of a stationary cloud from a point 2,500 m above a lake is 15° and the angle of depression of its rellection in the lakeis 45°. The height of the cloud abovethe lake level is ky/3 m where k=... 1. AB is avertical pole and Cis ts middle point. The fend A is on the level ground and P is any point on the level ground other than 4. The portion CB sub- tendsan angle Bat P, IAP: AB=2 | then tan= IL The angle of elevation ofthe top ofa vertical pole when observed from each vertex of a regular hexagon is. e Ifthe areaof the circle circumserib- ing the hexagon be square metre, then the area ofthe ka hexagon is “> square meres, where k= I, The angular elevation of tower CD at apoint A cue south of itis 60” andata point B due west ofA, the is 30°. If 4B = 3 kam, the height of the tower is V6 km, wherek = IN. Irwo vertical poles of heights h, and fy standing atthe middle point ofthe side BC nd ate centroid ofthe triangle A4BC respectively, make the same elevati angle atthe vertex 4, then a b The matching grid: 1. ABC is wiangular park with 4B —AC~ 100 m. A clock tower is situated at the mid-point of BC. The angles of elevation of the top ofthe tower at and B are cot !3.2 and cosee !26 respectively. The height ofthe tower is IL Each side of a square subtends an angle of 60° at the top of a tower # metre high standing in the centre of the square. If ais the length of each side ofthe square, then o = Ki? where k = @ (C) o @ wie 2 8 (HS aa You have either reached a page that is unavailable for viewing or reached your viewing limit for this book. aa You have either reached a page that is unavailable for viewing or reached your viewing limit for this book. aa You have either reached a page that is unavailable for viewing or reached your viewing limit for this book. 28.12 Mathematics for IT-JEE ‘Now, 4B =24N 2Aosin$ ‘Also, AM=MP= MC=r 2 PC=2r Now, fiom SACP, we have PC = PA? + AC = Papen reriparth= ni = h=r(/2-1 rheapunaeerche (ome = Vth) =(r+h)+d= = d= (V3-1rv2 a Vis 1) 10 (c). Let AP be the lamp post of height hat a point on a circular path of radius + and centre C. Let B be the point on this path such that ZPBA = 0, AB=heot a. Since 4B subtends an angle 45°at another point of | the path, itsubtends an engle of 90° at the centre C so that ZBCA=90 Also, CA=CB=r=34B= Wir and then Arcot a= V2 r => h= Jirtana, 11 (a). 40=80=CO=hcot 2+ Qibcircumsentrs of SABC = Bo=co=R= 40=09=co=n= 2 r+ AX3 1) == jm abe tan 12 (a). Let C be the cloud Cy and D itsreflection, PD is the surface of lake Gt iycoro Hed sorte) =(hsin(6+ 9) sing- 9 13 (a). Let Obe the fot ofthe tower p 4 ae 5 1S 0 “a Sa 3 2, AB= 4a: OB=Sa * Insdor, h tan@=—— ., h=3atana 3a Ke iim 2e rm Sere % Thus, h=Ja tan aor Sa tan Bh. 14 (2). Let PO be vertical and at its end Q there isa balloon. Pis on the ground = PC= heot Pis circumcentre of AABC and its citcumradius (R)=A cot a = R= = 2h cota aa You have either reached a page that is unavailable for viewing or reached your viewing limit for this book. aa You have either reached a page that is unavailable for viewing or reached your viewing limit for this book. aa You have either reached a page that is unavailable for viewing or reached your viewing limit for this book. 28.16 Mathematics for IIT—JEE 2(a,b). Let PO be the tower ofheight h and 4, B,C be the positions of the person from initial to final (fig). Then, ZPAQ=2 PBQ=30" and ZPCO= 60° = AQ=BO=heor30=h V3 and O0=heo60r= 4 Let QMbe perpendicular to AB and CL be perpendicular toOM Now, 4B =a and BC= # and ZABC=90° = ML=BC= & andce=aM= 2 From rightangled triangle BMQ, 2 OM= Bo? BAe = 312 al ee nd from right angled vingle CLQ, 2, 28 QL? = OC? CL? <7 Since,OM=OL+LM = z. 4 ori a is jis sang Secs yg lepslt = 6art—400 625 4 ~ 190 2, = 5T6H*—1500<*H* + 850a'=0 = reel 7500? + 42504 = (ABP 850°) (617-52) = Sa or ff Sle 8 6 £-(@),(b) holds. 1(2), LetABCbe an equilateral triangle with each side= a, AP bethe pillar of height, making angle of 45° at C,then AC =htan 45°=h= Ifthe elevation ofthe pillar at D { is@. (see figure) 4 ha ‘Then, tan O= = OD Fiala a be 2(40). Let BC be the declivity and BA be the ower. BC___4B 4 Sin 75° ~ Sin 30° £0 80sin30° SAB Sn TS 40x 2y2 B41 k = 40(V6 - V2) 345). Let the bird be perched at B, the top of the tree BD, and Obe the observer. Then, ZBOD= 45° and BD = 20 metres. ‘Now, the bird flying horizontally reaches M in 1 second. a au 20m oe og 08 D ” Then, ZMON= 30°, where MN LON. ‘Now, BD= MN=20 metres From triangle BOD, RD _ 20 ase= BB. 20 §, op=20 tan as*= 55 = Gy 2 OD=20metres ‘Againfrom AMON, tan30°= AN. _20 ON 20+ DN 120 ze 0¢20+ DN= 203 WB 20+DN DN= 20(¥3 ~1) =20%0.732= 14.64m smtee BM _ DN _s64m time I 1 = 1S ms approx, aa You have either reached a page that is unavailable for viewing or reached your viewing limit for this book. aa You have either reached a page that is unavailable for viewing or reached your viewing limit for this book. aa You have either reached a page that is unavailable for viewing or reached your viewing limit for this book. aa You have either reached a page that is unavailable for viewing or reached your viewing limit for this book. aa You have either reached a page that is unavailable for viewing or reached your viewing limit for this book. aa You have either reached a page that is unavailable for viewing or reached your viewing limit for this book. aa You have either reached a page that is unavailable for viewing or reached your viewing limit for this book. P6 Mathematics for IT-JEE Laine prk ee) =e fod Ce] ES 1(d). Equation of directrix is + y=0 Equation of parabola is (x—2)* + (y-2)? = ( = (r-yP Bir ty—2) 2 (d). Given integral J) (a). We have, 2sin'0~5 sind +2>0 => 2sin’0—4 sind sind +2>0 = (sind—2)2sind—1)>0 => 2sind-1<0 uc tee Lanne (08) (82) 5 (¢). 7 canbe made as (1,77), (7, 1), 07,7), (7, 7), and (7,7), Total ways = 5 can be made as, 4), (A), (6) (ADP OEP), (6,0), (8,0), (B.A). Total ways=9 Similarly? can be mede in ways. ‘Total number of ordered pairs=5 9x 5=225 6 (a). Sincex, is real, therefore, 4 (a+ b+ c))—4™ 3A(ab+ be+ac)<0 (atbte) 4 guerre A 2 FS Sab be + ea) 3 Since forthe existence of triangle @ (tanoy"> (tan 6)" = P,> P, Again, consider (cot 8) where (cot 6 > 1) which is increasing = P, (tn Of 4 P,> Py = Py>P)>P|>Py aa You have either reached a page that is unavailable for viewing or reached your viewing limit for this book. aa You have either reached a page that is unavailable for viewing or reached your viewing limit for this book. aa You have either reached a page that is unavailable for viewing or reached your viewing limit for this book. P42 Mathematics for WT-JEE 1 391 (a). Required area= 2{[(1-5, > 2 We). f Ginay*** . (cosx, cotx—In (sin x)™)de = dt a Put siny"*=1 = (sin. (cos x. cotx—log(sin x4) de = at A = fiede= th =a a Mb). y=3" Ine y=e Point of intersection (1,0) Slope at(1, 0) ofy=3"~! inxis 1. Slope at(1, 0) of y=x'— tis 1 40 1 (d). If (, y) is the point of intersection, then lol asl >O=9ae (~i,eo) ‘ 1 y=lal D 9 a€ (1) ay 1. Ha). Kxtkxa) =0 = (ka@i-§ba=0 = 7a- = ai+ pj = ob B=0 = a=0,f Sos pay=23i=2 MW (b). {fa—y7) ad} +]? -D a 0 ig ay 2) | -2(2)-4 .-]]-2@)-4 IV), sin c= ERASE _ Lee0s d cos B 2 Asin B Siang =P! evs Acos Bssin = 1S cos (48) =4-B=0 aa You have either reached a page that is unavailable for viewing or reached your viewing limit for this book. aa You have either reached a page that is unavailable for viewing or reached your viewing limit for this book. aa You have either reached a page that is unavailable for viewing or reached your viewing limit for this book. Mathematics for 1IT-JEE 20. 16 (a), I> (A), I (a), Ver ie) 21. 1 (@,b), 1 (a, d), IN (c,d), IV © (a, b) 2219 (Ne ()ed, Vel : HINTS TO SELECTED PROBLEMS 1(a). Differentiating we get # y tim LO0e2 x). 280? wtan x _ f2).2.2.1 > Find costo gue oy 72.(#/4) 4(d). Taking rotation about 4 : 2 2-34 4 ina Je) ent 3 2 (c). Let E, be the event that each American is seated ga 3dtt = aig Bt adjacent to his wife = z=Qrane™ st16 5 (a). Slope of tangent = = PE) ‘And let(E, 7 E>) be theevent that each American with Indian are seated adjacent to their wives = Cutson the left ofr=e. 6 (a). Differentiating. 4. 421/)—2 (=I => lim 2xf@)-Pf(=1 ~) op=r Ail) oB=r Copyrighted m aa You have either reached a page that is unavailable for viewing or reached your viewing limit for this book. aa You have either reached a page that is unavailable for viewing or reached your viewing limit for this book. aa You have either reached a page that is unavailable for viewing or reached your viewing limit for this book. aa You have either reached a page that is unavailable for viewing or reached your viewing limit for this book. P28 Mathematics for IIT-JEE WSs 7 abed 8 @d 9% bd 10. bd wo 2 Be wo 3 16) I 1B) 19% ew 2 HINTS TO SELECTED PROBLEMS 1 (b). ay i) and, x?+y?—6x+1=0 Ai) ‘Solving (i) and (ii) a a re Forthe limit o exist »-2=0— n=2and m=2 = 1-0 4 (6). The given expression ~ yo. 4 Cand C, touch each ather exactly a two points. 2 (b). «Sey +6y2=0 => Phyto =0 = xG-3y)-2y(-39)=0 > (3) (29) <0 =stwostraight lines and, when a= b and sign of cis opposite to that of a, the equation ax? + by? +.¢= Orepresents circle. B(c). - p=-l, : im =D" =-1=9 im —2-) Ip BT Fog cos" =D) Sea). et = im — 2 ___ 10 2x meas (x=) x(-sin (2-1) 1 ai cos" (x =1) Volume ~ ae oO we ge 11 tim MD te k oe oF wl —sin (x1) 603 (x= 1) aa You have either reached a page that is unavailable for viewing or reached your viewing limit for this book. aa You have either reached a page that is unavailable for viewing or reached your viewing limit for this book. aa You have either reached a page that is unavailable for viewing or reached your viewing limit for this book. aa You have either reached a page that is unavailable for viewing or reached your viewing limit for this book. aa You have either reached a page that is unavailable for viewing or reached your viewing limit for this book. aa You have either reached a page that is unavailable for viewing or reached your viewing limit for this book. aa You have either reached a page that is unavailable for viewing or reached your viewing limit for this book. aa You have either reached a page that is unavailable for viewing or reached your viewing limit for this book. aa You have either reached a page that is unavailable for viewing or reached your viewing limit for this book. aa You have either reached a page that is unavailable for viewing or reached your viewing limit for this book. aa You have either reached a page that is unavailable for viewing or reached your viewing limit for this book. aa You have either reached a page that is unavailable for viewing or reached your viewing limit for this book. aa You have either reached a page that is unavailable for viewing or reached your viewing limit for this book. Solved Paper IIT-JEE 2010 25. Theline 2x-+ y= istangent tothe hyperbola X= _fthis ine passes through the pint of ntersectonof the nearest ier and the x-axis, then the eccentricity ofthe hypebolais Sol. (2) substi 2, ofin y= -2x +1,we get e 4x) is periodic with period 2 a= ft (xjeosax de a Also, 1 = Va?m? — b? rn 2 41s atm? =o = 2] £(x)oosnx dx = 2%5f F[x)covne dx dy = ofa x)cos ax avs flower 0 (+12) ‘ " 1, = f(x =1)cos ax dx putx—1=¢ 26. Ifthe distance between the plane Ax ~ 2y + z= d and the plane containing the lines Sol (6) We have, 21+ 3m + n=O 31+ 4m +5n=0 -. Equation of plane will be 28, Let w be the complex number cos % ++isin 22 then “1 (x=1)+2(y-2)-1 (2-3) -0 the number of distinct complex numbers z satisfying, Sax41+2y—4—-743=0 ett ow Soxtly—z wo zt 1 isequal to re 1 t+u + x-2y+z=0 ‘Sol. (1) We have, I) ve waxes. 27, For any real number x, lt|3| denote the largest integer Jess than or equal to x. Let f be a real valued function defined on the interval {-10, 10]by x-|[x] if [xJisodd f(x)= 1 i (x) free x if [xfiseven q Salut) (z+0)—1-u(z+u-1) Tenfevaeot sf £(x)eosradei ion to Sol. (4).We have + 2 = Ois only solution. ARSON GUID UGE tas a aaa Sg eee era Sar aL Meee eee RU LS ed eC Re Cuca eevee ta easier ciated of the syllabus. The clear and concise presentation of the fundamental principles makes The Pearson Guide to Dee eeu ee ect Maruca ies fete ee Ret Cin areata CP stele illustrative examples to clarify concepts and applications Tricks to help students solve the maximum number of questions in the least amount of time. ee eco Ee eee Et anne en enn alates Pee Ee ae ete uae tke er ae tet sees the University of Delhi and isa gold medalist. He has been teaching undergraduate mathematics at Kirori Mal College, University of Delhi, for the past 18 years. He also teaches postgraduate students at the Faculty of Mathematics, University of Delhi. Dr Khattarhas authored ten books on mathematics and his main area Bae eee ey TM PEARSON

You might also like